恐ろしく難解な問題をだせ!

このエントリーをはてなブックマークに追加
1 ◆C27utN2bh.
さぁ、頭の良く出来てるキミ!
本気で問題だしてみたまえ!
2132人目の素数さん:03/04/07 03:04
坊や、糞スレはこっちで立てておくれ
http://www.bs1.net/math/ 
3132人目の素数さん:03/04/07 03:27
●未解決問題は禁止
●宿題は自分で解け(自分の解ける問題だけ)
4132人目の素数さん:03/04/07 06:57
Σ[n>0]1/(2^n+3^n)を求めよ。
5132人目の素数さん:03/04/07 07:12
>>4
とけるの?
6132人目の素数さん:03/04/07 08:31
>>5
ふつうに解けるだろ。
つか、こんなのも解けないのに難解な問題を出せとかいってんの?
もう見てらんない。
7132人目の素数さん:03/04/07 08:36
>>6
おれ>>1じゃない。>>4ふつうにとけるの?どうやんの?
8132人目の素数さん:03/04/07 08:43
とけるのage
9132人目の素数さん:03/04/07 08:53
>>7
ここはあなたのような真面目な厨房が来るスレではあをませぬ。
立ち去るが吉かと。
104:03/04/07 08:53
すまん。未解決問題出しちゃ駄目だったのね。
4はSpringerの未解決問題集に載ってる奴。
なんだ。とけないのか。
問題を出して、それで、どうするの?
>>1が答えるの?
>>12
> 問題を出して、それで、どうするの?
> >>1が答えるの?

>>6が答えてくれます。
14132人目の素数さん:03/04/07 17:51
Σ[k=1→n](1/k)
ぷりーず
>自分の解ける問題だけ
16132人目の素数さん:03/04/07 18:31
n≧240が整数ならばn^2+1は13より大きい素因数を持つことを示せ。
ζ(3)は無理数である。
18あぼーん:あぼーん
あぼーん
19132人目の素数さん:03/04/08 00:27
Σ[k=1→∞](1/k^2)
>>19
さんざん外出
lim[n→∞]n(-3/4+納r=1,n]納k=1,r,(k,r)=1]1/r^2(r+k))
23132人目の素数さん:03/04/20 02:32
  
>>19
π/6?
>>24
S>1>π/6=0.52....
π^2/6か?
>>26
精解
28山崎渉:03/04/20 03:57
   ∧_∧
  (  ^^ )< ぬるぽ(^^)
30132人目の素数さん:03/05/14 19:24
ほしゅったらageろ!
31翔太@中3 ◆////qfAzXY :03/05/14 19:25
人間はうんこ3dを何分でくえるでしょーか
32132人目の素数さん:03/05/14 20:50
リーマン予想を証明せよ
>>14
n->∞なら

与式=1/1+1/2+1/3+1/4+1/5+...
>1/2+1/2+1/4+1/4+1/8+...=>∞に発散ってやつだけど
34あぼーん:あぼーん
あぼーん
σ集合体の列B[1],B[2],… と事象列A[1],A[2],… があり、∀n, A[n]∈B[n]とする。
σ集合体 ∩[n=1〜∞]σ(∪[k≧n]B[k]) に含まれる集合を末尾事象と呼ぶが、
limsup[n→∞]A[n] = ∩[n=1〜∞]∪[k≧n]A[k], およびliminf[n→∞]A[n]=∪[n=1〜∞]∩[k≧n]A[k]
が末尾事象であることを示せ。

末尾事象という概念が載っているたいていの本では、この証明は省かれているか、練習問題になっている。
純粋に論理だけの問題だし。
しかし、議論のコツに気づかない学生には、「恐ろしく難解」かもしれない。
もっとも、それ以前に∀とか∃とか論理が使いこなせないレベルだと、問題の意味自体が恐ろしく難解かも
しれない。(藁

>>35
確かに集合の極限の扱いに慣れてないとできないかも。
多少なりとも慣れていればほとんど自明だけどね。
16の問題が気になる
3835:03/05/15 00:13
>>36
liminfのほうは自明というほどではないと思ふ。
∩[k≧n]A[k]∈σ(∪[k≧n]B[k])だが、 !∈ ∩[n=1〜∞]σ(∪[k≧n]B[k])なので。
“多少慣れている”だけだと、ここでつまづくと思われ。

それより、スレタイの「恐ろしく難解」というのは「難問」という意味じゃないのでは、
と思ってこの例を出した。
39132人目の素数さん:03/05/15 20:17
40あぼーん:あぼーん
あぼーん
41132人目の素数さん:03/05/15 20:47
問題:点1と点2+@を結ぶ各曲線に沿っての線積分の値をそれぞれ求めよ。(式もおねがい)
   
   ∫[Ca]e^(z)dz   (a=1,2,3)

   ここで、C1:z(t)=1+t+@*t (0≦t≦1)、
       C2:z(t)=1+t+@*t^2  (0≦t≦1)
   @   C3:z(t)=1+2*t   (0≦t≦1/2)
       z(t)=2+2@*(t−1/2)  (1/2≦t≦1)
   とする。
●宿題は自分で解け(自分の解ける問題だけ)
●宿題は自分で解け(自分の解ける問題だけ)
●宿題は自分で解け(自分の解ける問題だけ)
●宿題は自分で解け(自分の解ける問題だけ)
●宿題は自分で解け(自分の解ける問題だけ)
4316:03/05/15 21:00
>>37
Pell方程式の理論

{a_i}(i>=0)を整数列とし、十分大きな全ての整数がa_i+j^2(i, j>=0)と表されるとする。
この時、a_i<(1-c)i^2を満たす定数c>0が存在することを示せ。
44132人目の素数さん:03/05/15 22:05
2400π×433=
>>43
>{a_i}(i>=0)を整数列とし、十分大きな全ての整数がa_i+j^2(i, j>=0)と表されるとする。
>この時、a_i<(1-c)i^2を満たす定数c>0が存在することを示せ。

単調増加を仮定しないと嘘だな。
項を加えて全ての自然数が表されるとして、f_n:=#{i|a_i<n}とおいて
n*f_{n^2}=#{i|i^2<n^2}*#{i|a_i<n^2}>=n^2よりf_{n^2}>=nよりa_n<n^2てな感じだろうな。
46132人目の素数さん:03/05/16 20:58
up
4743:03/05/16 21:58
>>45
>単調増加を仮定しないと嘘だな。

そうだったスマソ。

>n*f_{n^2}=#{i|i^2<n^2}*#{i|a_i<n^2}>=n^2よりf_{n^2}>=nよりa_n<n^2てな感じだろうな。

確かにa_n<n^2は簡単にいえるが
a_n<(1-c)n^2(c>0)はそう簡単にはいえない。そこがポイント。
48あの人:03/05/16 22:01
AさんとBさんとCさんがいます。
Bさんはいつも笑って100円を片手にもってあるいてます。
200メートル歩いていくと、Bさんに出会いましたが、かれは走って逃げました。
Bさんは時速12kmで3分走り、10円を見つけて拾いました。
すると、偶然Cさんに会い、カラオケへ行く事になりました。

さて、Aさんの地点からカラオケ屋まで何kmでしょう。
また、いまAさんはいくら持っているでしょう。
110円
イk
50あぼーん:あぼーん
あぼーん
51山崎渉:03/05/22 00:28
━―━―━―━―━―━―━―━―━[JR山崎駅(^^)]━―━―━―━―━―━―━―━―━―
52山崎渉:03/05/28 14:59
     ∧_∧
ピュ.ー (  ^^ ) <これからも僕を応援して下さいね(^^)。
  =〔~∪ ̄ ̄〕
  = ◎――◎                      山崎渉
53132人目の素数さん:03/06/07 07:57
11
age
55132人目の素数さん:03/06/07 15:10
円周率が無理数であることを示せ。
56aaad:03/06/07 16:19
2^50はいくら?
57Agent_Smith◇:03/06/07 16:22
           ,..-‐−- 、、
         ,ィ":::::::::::::::::::;;;;;:ii>;,、
       /:::::::::::::::;;;;;;;;iii彡" :ヤi、
       i::::::::::::;:"~ ̄     ::i||li
       |:::::::::j'_,.ィ^' ‐、 _,,. ::iii》
        |:::i´`  `‐-‐"^{" `リ"
        ヾ;Y     ,.,li`~~i
         `i、   ・=-_、, .:/   自明だ。
          ヽ    ''  .:/
           > ‐- 、、ノ
58Agent_Smith◇:03/06/07 16:24
           ,..-‐−- 、、
         ,ィ":::::::::::::::::::;;;;;:ii>;,、
       /:::::::::::::::;;;;;;;;iii彡" :ヤi、
       i::::::::::::;:"~ ̄     ::i||li
       |:::::::::j'_,.ィ^' ‐、 _,,. ::iii》
        |:::i´`  `‐-‐"^{" `リ"
        ヾ;Y     ,.,li`~~i
         `i、   ・=-_、, .:/   1125899906842624だ。
          ヽ    ''  .:/
           > ‐- 、、ノ
(x-a)(x-b)・・・(x-y)(x-z)は?
>>59
0
61あぼーん:あぼーん
あぼーん
62132人目の素数さん:03/06/07 20:29

>>59
ゆうめりだからじょたえしってる
63132人目の素数さん:03/06/07 21:16
NP完全問題集
64132人目の素数さん:03/06/08 10:32
>>41
ヒント
∫f(z)dz=∫f(C(t))dz,dz=dc(t)=c'(t)dt->∫f(C(t))c'(t)d(t)
65132人目の素数さん:03/06/08 10:38
久しぶりにムスカを見た
いや・・・、エージェントスミスだったか
A君は自宅から駅まで行こうとしています
さて 自宅から駅の間にあるスーパーから駅までは何kmでしょう
また B君はA君をA君が出発してから3分後に自転車で追いかけました
駅の電車が500mのトンネルに入りはじめてからトンネルを完全に抜けるまで何秒かかるでしょう
>>31
うんこを食う事によって生成されるうんこも食べなければならないのですか?
68132人目の素数さん:03/06/08 14:57
 うんこ食べてまたうんこ出すのは循環論法か?
真偽をチェックせよ。
69Agent_Smith◇:03/06/08 14:58
           ,..-‐−- 、、
         ,ィ":::::::::::::::::::;;;;;:ii>;,、
       /:::::::::::::::;;;;;;;;iii彡" :ヤi、
       i::::::::::::;:"~ ̄     ::i||li
       |:::::::::j'_,.ィ^' ‐、 _,,. ::iii》
        |:::i´`  `‐-‐"^{" `リ"
        ヾ;Y     ,.,li`~~i >>65
         `i、   ・=-_、, .:/   ムスカで悪かったな。いいのがなかったのだ。
          ヽ    ''  .:/    ちかじか、本物のスミスをつくるかもしれない。
           > ‐- 、、ノ     
70mathmania ◆uvIGneQQBs :03/06/08 16:14
∫_{0}^{∞}sin(x)/xdx=π/2を示せ。
b(n) を、nを二進数表示したときに現れる 1 の個数とするとき、

∞   b(i)
Σ ───
i=1  i(i+1)

はどんな値に収束するか。
72藤原一宏:03/06/08 16:46
 (^^)
73132人目の素数さん:03/06/08 16:50
>>70
おやまあmathmaniaとは思えない古典的な問題。
コーシーの定理使うんだよな。
確か学部2年後期の複素解析で習ったなぁ。
74mathmania ◆uvIGneQQBs :03/06/08 17:01
複素関連をもう一つ。コーシーの積分公式を知らないと難しい。
∫_{C}exp(z)/zdz=2πiを示せ。但し、Cは原点の回りを正の向きに回る円周とする。

さて、もう一つ問題を出そう。(少し難しい程度かな?)
f(0,x)=1,f(1,x)=x,f(n+2,x)=xf(n+1,x)+f(n,x)で定義されるfについて、
kを非負整数とするとき、f(2k+1,x)はただ一つの根を持ち、f(2k,x)は根を持たないことを示せ。
(xは実数とせよ。xがfの根であるとは、f(x)=0のことをいう。)
>>74
expをテイラー展開すればコーシーの積分公式使わなくても余裕なのだが
76mathmania ◆uvIGneQQBs :03/06/09 12:52
Re:>75
項別積分が可能であることを示して欲しいものだ。(まぁ、それでも余裕というかな?)
77132人目の素数さん:03/06/09 21:31
f(x)=x^n (n≧3:整数) x=1で連続をε-δ論法で示せ。

∀ε>0 , δ= とおくと、

δは・・・。
宿題か?
学部1年程度の問題
区間[0,1]から区間[0,1)への全単射写像は存在するか?
存在するか存在しないかを選択し、証明をせよ。

・・・「恐ろしく難解」ではないと思うけど、ま、基礎の中では難しめってことで
81132人目の素数さん:03/06/09 22:34
宿題じゃねーが
宿題か
83132人目の素数さん:03/06/09 22:42
>>80
なかなか面白い問題だね。
f:[0,1]→[0,1)を全単射として構成するとして、例えば
f(0)=0, f(1)=1/2
f|(1/2,1)=id (fの(1/2,1)への制限を恒等写像にする)
f(1/2)=1/4
f|(1/4,1/2)=id
………
って構成していけば、全単射になるね。
このスレ、なんかいい〜
85132人目の素数さん:03/06/09 22:48
>1
三次元空間に、同じ大きさの球が、一つの球に対して接する球の数は幾つか?
86132人目の素数さん:03/06/09 22:51
  ↑
意味不明
8773:03/06/09 23:02
>>70 コーシーの定理を使わない解も思い出した。
F(t):=∫_[0,∞]exp(-tx)sin(x)/xdx
とおくと、
F'(t)=-∫_[0,∞]exp(-tx)sin(x)dx=[exp(-tx)cos(x)]_[0,∞] -(-t)∫_[0,∞]exp(-tx)cos(x)dx
= -1+t{[exp(-tx)sin(x)]_[0,∞]-(-t)∫_[0,∞]exp(-tx)sin(x)dx} = -1-t^2*F'(t)
∴dF(t)/dt = -1/(1+t^2)
t=tan(y)(0≦y<π/2)と変数変換すると、
d(F。tan)/dy = dF/dy。tan(y)*sec^2(y) = -1/{1+tan^2(y)}*sec^2(y) = -1
ここに「。」は関数の合成。
∴ F。tan(y) = C - y (C:定数)
∴ F(t) = F。tan。Arctan(t) = C - Arctan(t)
境界条件lim_[t→∞]F(t)=0により、
C = lim_[t→∞]{F(t)+Arctan(t)]=π/2
∴∫_[0,∞]exp(-tx)sin(x)/xdx = F(t) = π/2 - Arctan(t)
特にt=0とおいて、
∫_[0,∞]sin(x)/xdx = π/2
88132人目の素数さん:03/06/09 23:03
最低1個
89132人目の素数さん:03/06/09 23:08
教養学部一年レベルだけど、
∫_[0,∞]exp(-x²)cos(ax)dx = √π/2 * exp(-a²/4)
を証明してください。
あるひとつの球Sと、他の球Si(i=1,2,...)とがそれぞれ接するというだけで
Siどうしの位置関係については何も言っていないな。
91スミス91:03/06/09 23:23
すべての因果法則はアーキテクトである私のプログラムなのだよ、アンダーソン君。
92132人目の素数さん:03/06/09 23:26
↑訳のわからないヤツが出てきたなあ。
新手の荒らしか?
>>85
ケプラー予想を指してるのか?それは
94132人目の素数さん:03/06/10 00:06
四角形ABCDにおいて
AB=28,BC=21,CD=5
∠ABC=∠ACD=∠Rのとき
∠BADを求めてください。
96132人目の素数さん:03/06/10 00:14
>>94
他スレで高圧的な態度取って答えてもらえなかったからって、マルチするなよ。
皆に嫌われるぞ。
>>85
hcp
>>71
2log2
二進数の各位ごとの和に分解すると
Σ[n=0,∞]Σ[k=0,∞]Σ[j=0,2^n-1] 1/((k*2^(n+1)+2^n+j)*(k*2^(n+1)+2^n+j+1))
99魔人Arithmetica:03/06/11 17:46
x^3+y^3=mの正の整数解の個数をr(m)とするとき、

r(m)>(logm)^1/4となるmが無限に多く存在することを示せ。
100get
101132人目の素数さん:03/06/11 18:09
フェルマーさんがいつも起きる時間の平均値をもとめよ。
102あぼーん:あぼーん
あぼーん
103132人目の素数さん:03/06/12 01:26
1,2,3,…12と書かれたカードが1枚ずつある。(つまり計12枚)
これらのカードを、6角形の各頂点と各辺の中点(つまり12箇所)に1枚ずつ置く。
このとき、各辺の和が全て等しくなるようにカードを置くことは可能か?

2時間考えてもわかりませんでした…

104132人目の素数さん:03/06/12 01:33
オドリケルさんの♂の長さの平均値を求めよ
105132人目の素数さん:03/06/12 14:45
ある建築物を作るのに3メートルの部材Aが40本、2.5メートルの部材Bが30本、
1.3メートルの部材Cが50本、0.7メートルの部材Dが20本必要になりました。
部材の元になる資材は5メートル、6メートル、7メートル単位でないと
購入できません。
余りを少なく部材を切り出すには何メートルの資材をそれぞれ何本ずつ
購入すれば最も効率がいいでしょうか?
(必要本数以上の部材は必要ありません)

また、これを数式化することは可能でしょうか?
どなたか解の求め方を説明していただけるとありがたいです。
>>105
ばか者!
質問する場所が違うぞ!

ちゃんと質問スレに書き直せ!
簡単なルールも守れない奴は、学校など辞めて働け!
3 :132人目の素数さん :03/04/07 03:27
●未解決問題は禁止
●宿題は自分で解け(自分の解ける問題だけ)
108132人目の素数さん:03/06/12 15:04
>>106
すみません、この板初めてでしたので許してください。
出直します。
109132人目の素数さん:03/06/21 21:56
wz+h+j-q=0
(gk+2g+k+1)(h+j)+h-z=0
16(k+1)^3(k+2)(n+1)^2+1-f^2=0
2n+p+q+z-e=0
e^3(e+2)(a+1)^2+1-o^2=0
(a^2-1)y^2+1-x^2=0
16r^2y^4(a^2-1)+1-u^2=0
n+l+v-y=0
(a^2-1)l^2+1-m^2=0
ai+k+1-l-i=0
((a+u^2(u^2-a))^2-1)(n+4dy)^2+1-(x+cu)^2=0
p+l(a-n-1)+b(2an+2a-n^2-2n-2)-m=0
q+y(a-p-1)+s(2ap+2a-p^2-2p-2)-x=0
z+pl(a-p)+t(2ap-p^2-1)-pm=0

が自然数解を持つためのk+2の値の必要十分条件を求めよ。
どの変数についてとけって?
つか、その式適当だろう。
自然数a〜zが>>109の14個の方程式全てを満たす、って事。
この時k+2はどんな数になるか。
素数か?
いえす。んでk+2には全ての素数が出てくる。
114132人目の素数さん:03/06/22 04:52
>>109
マスマティカに溶かせたら?
>>113
これってマジなん?
詳しく知りたい
116132人目の素数さん:03/06/22 15:19
2以外の全ての偶数は、二つの素数の和で表せることを示せ。
ex. 4=2+2 6=3+3 8=3+5 10=5+5 12=5+7…
>>115
http://mathworld.wolfram.com/PrimeDiophantineEquations.html
# 詳しくはないけど

他にもこういう式はあるらしいね。
(ex. 他変数多項式で、p = f(x_1, …, x_n) > 0 なら p は素数のやつとか)
まとめたページがどっかあったと思ったけど見つからんかった。
118109:03/06/22 20:22
さて、109の問題に比べたら「私は>>117のリンク先からネタをぱくってきたかどうか?」って問題は簡単であろう。
119132人目の素数さん:03/06/22 22:35
((((((((((100!)!)!)!)!)!)!)!)!)!)!
>>119
顔文字かと思った
121あぼーん:あぼーん
あぼーん
>>116
それすげぇ不思議だよな。
それって一つの偶数に対して素数二つの組み合わせは一つしかないのかな

日本語下手でスマソ。誰か理解してくれ。そして教えてくれ
>>122
いいたい事はわかった。
つまり10=3+7=5+5だ
124132人目の素数さん:03/06/23 00:19
αは有理数として
x=3+2√2+√3
y=1+α√2+5√3
z=6-2√3
a,b,cは有理数とする
ax+by+cz=0をみたすa,b,cはa=b=c=0のときにかぎられるとする
このときのαを求めよ
>>123
本当に、ほんっとうにありがとう。
なんだろう、この気持ち。
凄くうれしい、凄く>>123に感謝したい。

本当にありがとう。ありがとう。
今日も一日が始まりましたね。
良い一日を。
126132人目の素数さん:03/06/23 02:43
sinX=4/5であるとき、Xが無理数であることを証明しなさい。
>>123
もとの問題を
 「異なる」素数二つの和
と、書き換えても反例はあるのかな?
>>127
5+13=7+11=18
29+61=31+59=90
とか、双子素数を使えばできるよ
>>124
普通の線形代数の問題に見えるんだけど・・・
130魔人Arithmetica:03/06/23 17:59
>>128
N(m)を2m=p+q(p, qは正の素数)の解の個数とする。
n以下の素数の個数は漸近的にn/logn
よって2n以下の偶数を二つの素数の和で表す方法は少なくともc_1 n^2/logn(c_1は正の定数)通りある。
よってある偶数2m(m<n)が存在して、N(m)は少なくともc_2 n/log^2n(c_2は正の定数)

nはいくらでも大きく取れるから、N(m)もいくらでも大きくなり得る、

具体的にはlimsup N(m)log^2m/m>0となる。

ここで問題。

S={m|N(m)>0}∪{1}とおくとき、inf#(S∩[1, n])/n>0であることを示せ。
131そこで問題です。:03/06/28 15:54
池袋のサンシャイン60は難解?
1000までかけて、円周率を1億桁くらい表示せよってのはふかのうか。
金田教授はやったけどなw
133132人目の素数さん:03/06/28 16:38
1/2-1/3+1/5-1/7+1/11-1/13+1/17-1/19+1/23-1/29+1/31-1/37・・・

の値はいくらになるか。
>>133
225469か
135132人目の素数さん:03/06/29 03:03
任意のnに対して
1/1+1/2+1/3+…+1/nは自然数とならないことを示せ。
136132人目の素数さん:03/06/29 03:51
反例
n=1
それ以上の時は1/p*2^kのkが一番大きくなってる部分を考えればよろし。
138supermathmania ◆ViEu89Okng :03/06/29 07:31
Re::>134 ?

問題(マルチ)
写像f:{整数}→{整数}で、任意の整数nに対してn=f(f(n+1)+f(n-1))となるようなものは存在するか?
139 ◆BhMath2chk :03/06/29 08:10
存在する。
140supermathmania ◆ViEu89Okng :03/06/29 09:01
Re::>139
それを証明してくれ。
141 ◆BhMath2chk :03/06/29 09:10
f(n)=an+bとするとa^2=1/2,b=0となるから
二回で1/2になるものを探すと
nが偶数のとき±n/2,nが奇数のとき±nというのが見つかる。
初めまして。現在大学受験を控えている高校三年生の男子です。
僕は数学が凄く苦手で、今の実力では大学合格は難しいと先生にも
言われてます。そこで、数学が得意な皆様に質問させて頂きます。
皆さんは数学をどのように勉強しましたか?勉強のコツなど、教えて下さい。
143supermathmania ◆ViEu89Okng :03/06/29 10:49
Re::>141
1行目と2行目がよく分からないのだが、
意外に簡単な例が見つかった。どうもありがとう。
>>142
苦手な人に聞いたほうがいいよ
これ本当
なんで?
146(´(´(´(´Д`ii):03/06/29 11:03

148132人目の素数さん:03/06/29 11:21
超難問!!!

f(x)が凸関数、つまり
 ∀a, b∈R:0<∀t<1:f(ta+(1-t)b)≦tf(a)+(1-t)f(b)
のとき、fは連続だそうですが、直感的には判るものの証明が思いつきません。
どう証明するのか、教えてください。
お願いします。
3 132人目の素数さん [sage] 03/04/07 03:27
●未解決問題は禁止
●宿題は自分で解け(自分の解ける問題だけ)
150407:03/06/29 11:29
>>148は、私がhttp://science.2ch.net/test/read.cgi/math/1056800354/407
で聞いている問題のコピペです。マルチではありません。
誰か◆ わからない問題はここに書いてね 104 ◆の煽りが貼っただけです。
でも、もし判る人がいたら教えて下さい。
お願いします。
151132人目の素数さん:03/06/29 11:30
お気に入り集 ☆
http://pleasant.free-city.net/
このスレにいたあるうざいコテを透明あぼ〜んに設定したらだいぶ快適になった
153山崎 渉:03/07/15 13:04

 __∧_∧_
 |(  ^^ )| <寝るぽ(^^)
 |\⌒⌒⌒\
 \ |⌒⌒⌒~|         山崎渉
   ~ ̄ ̄ ̄ ̄
154132人目の素数さん:03/07/17 02:01
超無限とは何か
155132人目の素数さん:03/07/17 02:01

8次元とは何か

156132人目の素数さん:03/07/17 02:04
巡回セールスマン問題を完全に解け



157132人目の素数さん:03/07/17 02:24
x^n+y^n=z^n (n≧3自然数)の非自明な整数解x、y、zは存在しないことを示せ
ab=bc=ca=1
ba=cb=ac=-1
aa=bb=cc=0
abc=bca=cab=0
な代数は可能か?
159_:03/07/17 05:15
age
161ドキュン:03/07/17 21:00
確率変数Xの確率密度関数をfX(x)とするとき、確率変数Y=|X^2-4|の分布関数および、密度関数を求めよ

絶対値の処理がわかりません。誰か教えてくれませんか?
またマルチか
163132人目の素数さん:03/07/17 21:23
>>142
おれの場合は生まれつきできたね。
だから、学校の授業を聴いているだけで高校まではできた。
だから、あなたと生まれが違う
164132人目の素数さん:03/07/17 21:33
幾何の問題です。
簡単のため真球の地球を考えます。
地表点Aが存在します。点Aから中心方向に正反対の地表を点Bとします。
簡単に言うと点Aの地球の真裏をBとします。
点Aと点Bの気温が完全一致する点Aが少なくとも一つ存在することを示せ。
また湿度気温が完全一致する点が少なくとも一つ存在することを示せ。
165132人目の素数さん:03/07/17 21:52
難しい話だね、通りすがりのサーファーに難解すぎる
166158:03/07/18 05:45
放置かよ!
167132人目の素数さん:03/07/18 05:59
>>158
じゃあ考えてやるか。

えー、普通に1は単位元、0は零元と解釈し、
表記から加法と乗法が入っていて、結合則
分配則その他が成り立っていると仮定すると、

abc=0 と bc=1 より a=0、同様に b=c=0
さらに ab=1、ba=-1 より、結局 a=b=c=0=1=-1

∴1点のみからなる代数で、かつその時に限り可能

これじゃダメ?
>>167
a=b=C=0=1=-1って何?

ヒント:
ある操作、関数としてもいいがそれをFとすると、
F(a)=0, F(b)=2, F(c)=5
F(ab)=F(ba)=F(a)+F(b)=2, F(bc)=F(cb)=F(b)+F(c)=7,
F(ca)=F(ac)=F(c)+F(a)=5,
F(abc)=F(bca)=F(cab)=F(a)+F(b)+F(c)=7
という示数が出て来るのです。
>>168
a=b=c=0だから
0=0*0=ab=1
 =ba=-1
ってことじゃないの?
1〜9,999,999,999,999,999,999,999,999,999,999,999,999,999,999,999,999,999までの数を全て足す

スーパーコンピュータでも使わない限りほとんど無理だな
>>170
???
99,999,999,999,999,999,999,999,999,999,999,999,999,999,999,999,999,990,
000,000,000,000,000,000,000,000,000,000,000,000,000,000,000,000,000/2
だから筆算でもできるだろ
173132人目の素数さん:03/07/18 10:05
単位立方体がある。
対角線を軸にして出来る回転体の体積を計算してくれ。
3次元の話だ。
174132人目の素数さん:03/07/18 10:22
>157を解ける香具師はいないのか?
>>174>>157
ワイルズの原論文見たことあるけど、百ページ以上もあって漏れには理解不能。
解ける香具師はワイルズ以下ほんの数名じゃないか?
176132人目の素数さん:03/07/18 20:59
173むずいよ・・・
173て円錐二個と円柱一個になるんじゃないのかな?
あんまり考えてないけど。
178132人目の素数さん:03/07/18 21:39
>>174
私はこの命題に驚くべき証明をもっているが、
余白が狭すぎるのでここに記すことはできない。
実数のべき集合 2^R の代表系を一つ選んでくるべし。
180132人目の素数さん:03/07/18 21:50
173
円柱2個の間はちょっと凹んでるような気がする。
181173:03/07/18 21:57
よく考えたら凹むねぇ。
立方体の対角線に垂直な面で立方体をシパーンて切断して
断面がどんななっているかに注目したら解けるかな。
>>169
0*0=1=-1 ??? *はどういう演算
183180:03/07/18 22:27
>>173
ベクトル使いました。
(5+4√3)π/27
自信ないけど、解き方はあってたはず。
>>183
おれの答えとちがう・・・どうやったの?
185180:03/07/19 01:24
(√3)π/3
でした。
おれもそうなった。
187132人目の素数さん:03/07/19 01:42
とてつもなく難しいのはゴールドバッハ、リーマンζ、コラッツだな
188内田栄治 ◆0KFWZfjnEk :03/07/19 01:45
>>187
あなたにとって数学=数論なのですか?
>>169
>>168も満たしてないし
コラッ!っ
191132人目の素数さん:03/07/19 07:05
実数のべき集合 2^R の代表系を一つ選んでくるべし。
これは
連続体仮説に関わってくる
数学基礎論の難問じゃないですか??
192169:03/07/19 08:48
>>182>>189
0*0は0と0の積のつもり。
0=aと0=bから0*0=abと0*0=baが出るが
仮定からab=1, ba=-1だから0=1=-1
ということなのだけど何か間違ってる?
>>158=>>168がやっているのはじゃんけんであって代数ではない。
>>193
でへへ
195169:03/07/19 10:36
代数じゃなくて代数系?
だったら存在するのは自明だけど。
196169:03/07/19 10:42
ン、そうでもないのかな。

とりあえず158のいう「代数」とは何かがよく分からない
可能か?と言っているので、可能でない、で。
>>193
正解
199132人目の素数さん:03/07/19 11:02
アダルト激安DVDショップ!1枚500円〜!



http://www3.to/900yendvd

200169:03/07/19 11:02
結局>>167のどこに不満があったのかよく分からないのだが
201132人目の素数さん:03/07/19 13:18
すいません、半年考えてるのですが答えが判りません


方眼紙の上にコンパスと30cm定規だけを使って正五角形を書きなさい。


ちなみに小学生の問題だそうです…
●未解決問題は禁止
●宿題は自分で解け(自分の解ける問題だけ)
>>201
せめて鉛筆とコンパスの芯もないと、書くのは困難だな。
>>201
正五角形の作図方法のことだったら、オレは学校で習ったが。
忘れたとしても、検索かければすぐ見つかるだろ。
例えば
 ttp://www.mitene.or.jp/~tomo-s/pentagon/pentagon10.html
 ttp://www.geocities.co.jp/Playtown-Dice/5061/sansu/pentadraw.htm
 ttp://www.nn.iij4u.or.jp/~hsat/misc/math/drawing/pentagon.html
等々
205132人目の素数さん:03/07/19 13:33
方眼紙を細長く折って結んだらいいんでないの。
206204:03/07/19 13:37
>>205
上手いね。参った。
207132人目の素数さん:03/07/19 13:45
>>201
小学生の問題はありえないだろ…。まー、計算問題じゃないから、
ナゾナゾ感覚でトライすればいつかはかけるだろうけど。
それに、204さんが言うように、ぐぐればいくらでも見つけられるんだから、
半年悩むってのも愚かよの(笑)
わかんなかったらとりあえずぐぐってみるクセつけることを勧める
なんでもいいから作れなら次のようにしてできる。
 
(基本作業)長さR<Sの2がとれたとき半径Rの円とその円周上の直径の端点となる
2点を中心とする半径Sの円をかきその交点ともとの円の中心の距離をとる。
それは√(S^2-R^2)となる。
 
(Step1)なんでもいいので2点とる。面倒なのでこのながさを1とする。
(Step2)半径1の円に内接する正六角形をかいて長さ2をとる。
(Step3)半径2の円に内接する正六角形をかいて長さ4をとる。
(Step4)基本作業をくりかえし長さ√6、√2をとる。
(Step5)半径1の円の直径を端点とする半径√5の円の交点の内のひとつと
半径√2の円の交点の内のひとつは距離が1+√5になる。
(Step6)辺の長さが2、1+√5、1+√5の二等辺三角形をかいてそれぞれの頂点から
半径2の円をかいてその交点をとると正5角形がつくれる。
 
もっとうまい方法はありそうだけど。
a, b, c をabc=1 を満たす正の実数とする.次の不等式を示せ.
( a - 1 + 1/b) ( b - 1 + 1/c) (c - 1 + 1/a) ≦1
IMOの過去問ですが。知識の多さではなく頭の良さが試せます。
210132人目の素数さん:03/07/28 16:07
>173
5/6π  ダ
>>157

解(x,y,z)=(A,B,C)が存在すると仮定する。

A^n+B^n=C^n⇔y^2=x^3+(A^n-B^n)x^2-(AB)^n

この楕円方程式y^2=x^3+ax^2+bx+cはモジュラーでありえない。
従って谷山-志村予想が成り立たない。

谷山-志村予想は証明済みであるので、矛盾である。
従って成り立つ。




適当に脳内補完汁。
全ての素数の和を求めよ。
難しいかどうかはわかんないけど、
学校で出た問題でとき方がわかんなかったので・・・
1)f(x+y)=f(x)+f(y)⇒f(x)=ax a∈R
2)[0,1]〜[0,1)を示せ(区間[0,1]から区間[0,1)への全単射を探せ)
>>213
2)
1/n -> 1/(n+1) (n=1, 2, 3, ...)
それ以外のところではidentity
でいいかな
1)はQ上ならできるけどRにすると連続性が必要では?
連続性を仮定しない場合の反例が昔どっかのスレにあったような
選択公理、Zornの補題
http://science.2ch.net/math/kako/1011/10113/1011351649.html
|RをQ上ベクトル空間とみなし基底煤+Aλ:λ∈Λ}とする。
|各λに対して実数Bλを対応させ
|X=X1A1+X2A2+・・・・+XnAnに対して、
|f(X)=X1B1+X2B2+・・・・+XnBnを対応させると
|fは条件を満たす。
|ここで例えばA1に対して1、それ以外には0を対応させると、不連続になる。
だす。
選択公理と 1) の反例の存在は同値ですか?
     ∧_∧  ∧_∧
ピュ.ー (  ・3・) (  ^^ ) <これからも僕たちを応援して下さいね(^^)。
  =〔~∪ ̄ ̄ ̄∪ ̄ ̄〕
  = ◎――――――◎                      山崎渉&ぼるじょあ
220山崎 渉:03/08/15 19:36
    (⌒V⌒)
   │ ^ ^ │<これからも僕を応援して下さいね(^^)。
  ⊂|    |つ
   (_)(_)                      山崎パン
221132人目の素人さん:03/09/23 03:34
>>209
(IMO 2000)
abc・・・=1 とくれば、とりあえず循環座標をとる。
a=x/y, b=y/z, c=z/x, x>0, y>0, z>0
これを与式の左辺に入れて整理すると、求める式は
(x-y+z)(y-z+x)(z-x+y)≦xyz
左辺の分母の3つの因数のうちの2つ以上が≦0だったら その平均値≦0 になるが、これは仮定に反する。
@)非正因子が1つのとき 左辺≦0,右辺>0 で成立.
A)3つとも正因子のとき
(x-y+z)(y-z+x) = x^2−(y-z)^2 ≦ x^2
(y-z+x)(z-x+y) ≦ y^2
(z-x+y)(x-y+z) ≦ z^2
辺辺かけて平方根をとると(各辺正により)求める式を得る。
Quantum ElectroDynamics.
>>221
ワラタ
223132人目の素数さん:03/09/25 22:36
0≦a,b,c≦1のとき、次の不等式を示せ。
a/(b+c+1) + b/(c+a+1) + c/(a+b+1) + (1-a)(1-b)(1-c) ≦ 1

〜さくらスレで質問された問題。どこかで解答済だったらスマソ。〜
224132人目の素数さん:03/09/25 23:04
>>223
スレ133, レス14 (03/08/23 21:49)
225132人目の素数さん:03/09/26 02:45
>>223
ちょっと上等な高校1年生で楽勝にとける。もう2度とこの板に書き込まないほう
がいいよ。
226132人目の素数さん:03/09/26 18:01
A3. p is an odd prime. The integers a, b, c, d are not multiples of
and for any integer n not a multiple of p,
we have {na/p} + {nb/p} + {nc/p} + {nd/p} = 2, where { } denotes the fractional part. Show that we can find at least two pairs from a, b, c, d whose sum is divisible by p.
227aaad ◆ozOtJW9BFA :03/09/26 18:18
因数分解の無ずいの下さい。
x^105-1 の因数分解。
229132人目の素数さん:03/09/26 18:38
素因数分解の無ずいの下さい。
230132人目の素数さん:03/09/26 19:23
2^16384+1
231132人目の素数さん:03/09/26 19:29
分解の無ずいの下さい。
(・∀・)

これを分解せよ
233132人目の素数さん:03/09/26 20:02
解の無ずいの下さい。
カイイノー

アア テガトドカナイ
ムズイヨ

ダレカ
オレノ
セナカカイテ
無ずいの下さい。
236132人目の素数さん:03/09/26 20:39
下さい。
237aaad ◆ozOtJW9BFA :03/09/26 20:41
何かわかんねぇ。。。
238132人目の素数さん:03/09/26 20:45
わかったよおまえら、
生まれ変わってそこらの草にでもなれば
になれば髄(ずい)がなくなるからいっぺん逝っとけ

あと、クレクレ君はお断り

a[1] = 1
a[n] = ( 1 / (2n-1) ) * Σ(k = 1 to n-1) ( a[k] * a[n-k] )

lim(n→∞) a[n] / a[n+1] = ?

・・・形から解法分かりそうだな
241132人目の素数さん:03/09/27 01:15
>>223
f(x)=1/(x+1)-1+x/2 とおくと、与式=a・f(b+c)+b・f(c+a)+c・f(a+b)+1-abc だから、求める式は、f(b+c)≦bc/3, etc.
0<x<1のとき、f(x)=-x(1-x)/2(x+1)<0≦bc/3.
1<x<2のとき、f(x)-(x-1)/3=(x-1)(x-2)/2(x+1)<0 より f(b+c)<(b+c-1)/3≦bc/3. q.e.d.
〜恐ろしく難かない問題でした。〜

そこで問題です。 与式≧7/8 ?

〜IMO掘りする? 天高く馬肥ゆる..〜
242132人目の素数さん:03/09/27 02:55
問題 4.

手品師が1から100までの数字が書かれた100枚のカードを持っている.手品師はその100枚のカード全てを3つの箱(赤い箱,白い箱,青い箱)に入れて,それぞれの箱に少なくとも1枚以上のカードが入っているようにする.

 

ある観客が,異なる2つの箱からカードを1枚ずつ取り出して,カードに書かれている数字の合計を手品師に教える.教えられた値をもとに,手品師はカードが引かれなかった箱の色を当てる.

 

このようなトリックが成立するようなカードの入れ方は何通りあるか.

 

(少なくとも1枚のカードが別の箱に入っているとき,異なる入れ方だと数える.)

 

問題 5.

次の条件を満たす正整数nは存在するか.

 

nを割り切る相異なる素数はちょうど2000 個ある.

2n + 1 はnで割り切れる.

IMOの過去門
奇数の完全数はない
244132人目の素数さん:03/09/27 04:43
>>242
A−B−√C≒Abc
    あべし=あべし         アベあああああああああああああああ 


ファSfdファfだfだa     
afdaf
 
Fだf fdkfdあfdあ

だfdだ
だf
gwrbhthhtげh
                             A. のりおちゃんぽーん
245132人目の素数さん:03/09/27 06:06
nは自然数とする。
2^n+1がnで割り切れるための必要十分条件を求めよ。

P(p):=
(pは素数)
&
(2^(p-1)≡1 mod p)
&
{(2^r≡1 mod p)&(r<=p)⇒(r=p)or(r=0)}
X:={p∈N|P(p)}
#X=∞を示せ。#Xは集合Xの要素数。(無限集合に限り∞と書く。)
一辺がLの正方形ABCDがある。
Aを中心に半径Lの円を描く。
BCの中点を中心に半径L/2の円を描く。
二つの円が重なる所の面積を求める。

説明が下手ですいません。
>>246
はい、やり直し
249132人目の素数さん:03/09/27 22:23
>>247=http://science.2ch.net/test/read.cgi/math/1064407210/428
マルチ・スレッドは駄目だよ
>>249
正直書いた覚えないんですけど・・・(汗。
弟の宿題手伝ってたら、とある問題に興味を引かれ、、
それをちょっと改題したやつなんですが・・・。
251132人目の素人さん:03/09/28 00:45
>>245
n=3^k (k=0,1,2,・・・)?
十分性:
[(2^n)+1]/3n=f(n) とおくと、f(1)=1, f(3n)=f(n)・[3(nf(n))^2 -3nf(n) +1]
P(p):= (pは素数) &
(2^r≡1 mod p)&(0<r<=p)⇔r+1=p
X:={p∈N|pはP(p)を満たす}
#X=∞を示せ。#Xは集合Xの要素数。(無限集合に限り∞と書く。)
253132人目の素人さん:03/09/28 04:02
245 って 242 問題5 の補題?
254132人目の素人さん:03/09/28 04:20
次の不等式をみたす有界無限実数列:x_0,x_1,x_2,・・・の例を与えよ。
すべての異なる非負整数i,jに対して
|x_i-x_j|・|i-j|≧1

(注)実数列x_0,x_1,x_2,・・・が有界であるとは、
ある定数Cが存在して、すべての非負整数i≧0に対して、
|x_i|≦C
が成り立つことである。(IMO 1991,第6問より)
255132人目の素数さん:03/09/28 04:21
>>253 むしろ拡張じゃない?
256132人目の素人さん:03/09/28 19:11
>247 〜さくらスレの問題〜
半径r1の円1と半径r2の円2があるとし、中心間の距離をdとする。
これらの円は互いに交わるとする: |r1-r2|≦d≦r1+r2
このとき共通部分の面積S及びこれを中心軸廻りに回転した立体の体積Vは、
S(r1,r2,d)=(r1^2)・[arccos(c1)-c1・sqrt(1-c1^2)]+(R2^2)・[arccos(c2)-c2・sqrt(1-c2^2)]
V(r1,r2,d)=(π/3)・(r1^3)・[(1-c1)^2]・(2+c1)+(π/3)・(r2^3)・[(1-c2)^2]・(2+c2)
ここに、c1=(d^2+r1^2-r2^2)/(2・d・r1), c2=(d^2-r1^2+r2^2)/(2・d・r2) ・・・第二余弦定理

円1の中心から弦を見た角の1/2とをθ1とすると、c1=cos(θ1)。 c2も同様。

とくにθ1+θ2=π/2のとき、r1^2+r2^2=d^2,c1=r1/d,c2=r2/d,
S(r1,r2,d)=(r1^2)・arccos(r1/d)+(r2^2)・arccos(r2/d)−r1・r2

------------------------------------------------------
本題では、r1=L,r2=L/2,d=L・sqrt(5)/2 より、c1=2/sqrt(5),c2=1/sqrt(5) だから
S = [arctan(1/2)+arctan(2)/4-1/2]×L^2 ≒ 0.240434788・・・ L^2

arctan(1/2)≒ 0.463647609・・・,arctan(2) ≒ 1.107148718・・・

解答が下手でスマソ。なお、本件はさくらスレ134にて解答済。
257132人目の素数さん:03/09/29 22:40
分からない問題スレより

2 以上の自然数 n に対し、n 以下の素数全ての積を p(n) とおく。
(p(2)=2 p(10)=2*3*5*7 など)
このとき lim[n → ∞]Σ[k=2 〜 n]p(k)/k! を求めよ。
>>257
それはむずいわ
259132人目の素数さん:03/09/30 13:03
age
>>257
かなり大甘な評価で0になりそうなんだけど。ちがうかな?
0?
>>261
ああ、ゴメン。狽ゥ。逝ってきます。これ答えチャンと出る問題なのかな?
だったら気合いれてかんがえてみるんだけど出題者不明だからな。どうなんだろ?
>>262
>>257は名人スレでコテが思いつくままつくった出題者自身答えもってない問題。
考えるだけ時間のムダ。
 
http://science.2ch.net/test/read.cgi/math/1049207247/118
3 132人目の素数さん [sage] 03/04/07 03:27
●未解決問題は禁止
●宿題は自分で解け(自分の解ける問題だけ)
265132人目の素数さん:03/09/30 16:43
ってかlim[x→a]杷(a)の説き方だけでもわかるヤシに教えてほしいよ…
狽項数n(a=1、2、3…)としての話ね…
266132人目の素数さん:03/09/30 18:54
F_1=1, F_2=2, F_3=3, F_4=5, ...をFibonacci数列とする。

Σ[k=0, 1, 2, ...]1/(F_{2^k})を求めよ。
すくなくともp(x)〜exp(x)みたいだから納k=1,n]p(k)/n!なら0なんだけどな。
分母k!だったか。これは解ける気がまったくしない・・・
268132人目の素数さん:03/09/30 23:06
>>266
Fibonacci数列というと普通F_1=F_2=1となるものをいうみたいだけど
>>266のはそれじゃないんだけどどっちなの?これであってるの?
これはちゃんと答えだせる問題?
269132人目の素数さん:03/10/01 01:11
(2^n)+1が n^2 で割り切れる → n=3. (IMO 1980)
>>267
>納k=1,n]p(k)/n!なら0なんだけどな。
お前バカだろ
>>270
lim[n→∞]納k=1,n]p(k)/n!=0でまちがいないと思うけど。
272132人目の素人さん:03/10/01 02:23
lim[n→∞] Σ[k=2,n]p(k)/k! なら 2.58924612663799・・・・・なんだけどな。
まあ、どうでもいいが....
>>272
その数字はどっからきたの?計算機で順にたしていって近似値だしただけ?
それともlim[n→∞] Σ[k=2,n]p(k)/k!の値をなんかπとかeとかの多項式で
あらわすことに成功したの?
lim[n→∞] Σ[k=1,n](p(k)/k!)
lim[n→∞] (Σ[k=1,n]p(k))/n!
>>274
で?
277132人目の素数さん:03/10/01 08:11
e^sinxのマクローリン展開教えてください
278132人目の素人さん:03/10/02 00:14
>254 (IMO 1991)
i≠j,mは自然数とし、m≠平方数とする。
xを越えない最大の整数を[x]、xの小数部分を{x}とする。
sqrt(m)≡μ とおくと、[i・μ] = i・μ - {i・μ}
[i・μ]-[j・μ] = (i-j)・μ - {i・μ} + {j・μ}
N = (i-j)・μ - {iμ} + {jμ}
両辺を2乗して移項すると、
N^2 - m・(i-j)^2 = ({iμ}-{jμ})・(-2(i-j)・μ+{iμ}-{jμ})
左辺は0でない整数だから、
1 ≦ |{iμ}-{jμ}|・|-2(i-j)・μ+{iμ}-{jμ}| ≦ |{iμ}-{jμ}|・(2μ+1)・|i-j|
よって x_i = (2μ+1)・({iμ}-1/2) とおけば求める式を得る。
μ≡sqrt(m),C=μ+1/2, q.e.d.
テニスの王子様の越前って全然すごくない!!!だって本人の力じゃなくて運でしょ???
何が運かって、越前南次郎の子供として生まれたから、テニスが強いだけでしょ????
本人の力何も関係ないじゃん!!!!!今これを読んでるキミだって運良く南次郎の子供に生まれたらテニスの王子様だよ!
たまたま運がよかったから、テニスが強いだけじゃん!!!
氷帝の髪長い奴のほうがまだマシ!彼は自分で死ぬほど練習して強くなってるから!!
もともとはテニスでは勝てない運命だった奴が、死ぬほど努力して運命を変えるのが本当の凄い奴だ!!!!!!!!!!
越前は運で勝ってるだけ!!!!越前の力じゃない!!!!!!!!!!!!!!!!!!!!!!!!
死ねや越前!!!!!!中学生じゃないだろお前!!!!!!????
やってられんわ!くそが!!!だまされたわ!!!!
みんな早く気付け!!!!!!越前の糞さに!!!!!!!!
あああ!!!!!!!!!
280132人目の素人さん:03/10/04 10:44
>>273
3.14 とか e とかの多項式であらわすことに失敗しました。

S 〜 e - (1/31)*4 -3.14/(31^4)
281132人目の素数さん:03/10/05 09:50
ワードやエクセルでインテグラルやn乗、ベクトルなんかの記号を
入力て出来るんですか?
282132人目の素数さん:03/10/05 10:35
小ネタ
a,b,c>=0 のとき
4(a^2+b^2+c^2-bc-ca-ab)[(bc+ca+ab)^2-3abc(a+b+c)]-[(a+b+c)(bc+ca+ab)-9abc] >= 0
等号成立は D=(a-b)(b-c)(c-a)=0 のとき.

基本対称式: s1=a+b+c, s2=bc+ca+ab, s3=abc
283132人目の素数さん:03/10/05 10:40
↑次数が合わぬ.
4(a^2+b^2+c^2-bc-ca-ab)[(bc+ca+ab)^2-3abc(a+b+c)]-[(a+b+c)(bc+ca+ab)-9abc]^2 >= 0
等号成立は D=(a-b)(b-c)(c-a)=0 のとき.
スマソ.
284132人目の素数さん:03/10/05 11:09
3年前から時々思い出しては考えてるんだけど
可換体Kの可算無限直積のK-ベクトル空間としての基底って、
具体的に構成できる?
非可算次元になることは分かったんだけど。
285ななし:03/10/05 12:33
>>284
そのベクトル空間の元を基底で表示するとき、「有限個の基底で表示」か
「無限個の基底で表示」かで、違ってくると思う。
286284:03/10/05 19:18
>>285
「ベクトル空間の基底」といえば有限個と思ってたんだけど。
代数的基底と言った方がよかったかな。
>>284
>可換体Kの可算無限直積
これって普通に読むと K*K*K… っていうことだよね?
それの
>K-ベクトル空間としての基底
なら E_n:=(0,…0,1,0,…0…) ←n番目だけ1で他の要素は0
でいいんじゃないの?
でも多分こういうこと言ってるんじゃないよねえ・・・
(1,1,1,1,・・・)がもれてしまいます。
>>288
Σ[n=1 to ∞]E_nはどうなりますか
てゆうか

>>286
「ベクトル空間の基底」といえば有限個と思ってたんだけど。

この思い込みはいったいどこから・・・
291284:03/10/05 21:14
>>290
書き方が悪かった。訂正。
こういうことを言いたかった。
ベクトル空間Vの部分集合Bが基底とは、
∀v∈V, ∃c_b∈K s.t.
 v=納b∈B] (c_b)b & 有限個のbを除いてc_b=0
292284:03/10/05 21:16
条件が足りなかった。極小な集合でないといけないね。
>>289は学部生
>>284

その手の事は多分無理。
>>291
もう忘れたけど基底の条件ってそんなのだったっけ
例えば体K上の形式的ベキ級数環K[[X]]の部分集合B={1 , X , X^2 , X^3 ・・・}って基底じゃないんか
>>295
(vi)がベクトル空間Vの基底である
⇔(i)(vi)は1次独立である。
  (ii)任意のv∈Vに対したかだか有限個が0でない(ai)が存在しv=蚤ivi
    が成立する。
>例えば体K上の形式的ベキ級数環K[[X]]の部分集合B={1 , X , X^2 , X^3 ・・・}って基底じゃないんか
基底ではない。
297ななし:03/10/06 00:23
>>287
それだと、
x = (x1, x2, …, xn , …)
に対して、形式的には

x = Σ[n=1, ∞] xnE(n)

となるけど、↑の Σの意味(無限和)の意味が問題になってきます。
今可換体 K 上のベクトル空間だから、位相がどうなっているかわからないからです。
>>295
1+x+x^2+x^3+....
x+x^2+x^3+....
x^2+x^3+x^4+.....
........
はK上一次独立
かつ
K[[x]]を生成
じゃないの?
299284:03/10/06 10:59
>>298
1+x^2+x^4+x^6+...
が外れてる。
K^NとK[[x]]はKベクトル空間としては同型だから、
可算な基底はもたないはず。
300283:03/10/06 23:24
3次元ベクトルを u=(b-c,c-a,a-b), v=(a(b-c),b(c-a),c(a-b))とおく。
2(a^2+b^2+c^2-bc-ca-ab) = (b-c)^2+(c-a)^2+(a-b)^2 = |u|^2.
2[(bc+ca+ab)^2-3abc・(a+b+c)] = [a・(b-c)]^2+[b・(c-a)]^2+[c・(a-b)]^2 = |v|^2.
(a+b+c)・(bc+ca+ab)-9abc = a・(b-c)^2+b・(c-a)^2+c・(a-b)^2 = (u・v)
また、v×u = (D,D,D)より、
与式 = |u|^2・|v|^2-(u・v)^2 = |v×u|^2 = 3・D^2 >= 0. 等号成立はD=0のとき. q.e.d.
301132人目の素数さん:03/10/06 23:39
きぼんぬ証明。
Nを2以上の自然数とするとき、Nと2Nの間に少なくとも
一つの素数が存在することを示せ


302132人目の素数さん:03/10/06 23:42
方程式
1+1/2^s+1/3^s+1/4^s+1/5^s+1/6^s+1/7^s+1/8^s+・・・=0
の解sは、全てs=1/2+ti(tは実数)の形で表されることを示せ。
303132人目の素数さん:03/10/06 23:48
>>302
それ本当にそんな難しいの?
304132人目の素数さん:03/10/06 23:48
>>303
それほどでもないですよ。頑張って解いてみて下さい。
305132人目の素数さん:03/10/06 23:53
>>302
オマエ、アフォか?収束せんだろう。
306132人目の素数さん:03/10/07 00:09
>>305
ここに、馬鹿発見w
リーマンも泣いてるよ、きっと
308132人目の素数さん:03/10/07 00:11
>>305
リーマン予想を知らんのか
309132人目の素数さん:03/10/07 00:12
>>307
302に言ってる?
310132人目の素数さん:03/10/07 00:19
>>309
うん。定義が間違ってるもん。
>>306
ここに、馬鹿発見w
312132人目の素数さん:03/10/07 00:25
>>302
sが分母に付いている、だとか、解析接続云々を大目に見たとしても、完全な誤り。
オマエ等全員、出直して来い。
313132人目の素数さん:03/10/07 00:31
>>312
は??本当に馬鹿なの!?
クソスレ化進行中〜
まあ>>312が正しいわけだが。
316132人目の素数さん:03/10/07 00:40
>>312
かわいそうなうんこ君に、説明してやるよw
ゼータ関数とは

ζ(s) = 1/1^s + 1/2^s + 1/3^s + 1/4^s + 1/5^s + 1/6^s + …

   = Σ1/x^s
という形で表される関数のことだ。
ζ(s) =0を満たすsは、全て実部が1/2になっている。これが有名な未解決問題
リーマン予想だ。わかったか?こんなん常識だぞ。そしたらなんで>>302が完全
な誤りなんだ?あ?コラ。なんとか言えタコあるいは氏ね
(・∀・)イイヨイイヨー
318132人目の素数さん:03/10/07 00:48
誤りじゃないですよ
319132人目の素数さん:03/10/07 00:49
>>317
どっち派?312と316。
>>319
なるべく長くもめてほしい派
321132人目の素数さん:03/10/07 00:53
312みたいな人って、自分の誤りを絶対素直に認めないんだよね。
まぁ、「完全な誤り。オマエ等全員、出直して来い。俺が正しいんだ
天才なんだすごいんだてめぇらひれ伏せ」とまで言い切っちゃってるからねw
322132人目の素数さん:03/10/07 00:54
>>319
いや普通に316だろ。
323132人目の素数さん:03/10/07 00:55
>>321
と言うより、書き込んでこなくなりましたねwぐぐってみて、
やっと式の意味知ったのかな?
324132人目の素数さん:03/10/07 00:57
>>315も腐ってるね。早く吊ってほしいよこうゆうゴミどもは。
325132人目の素数さん:03/10/07 00:58
312と315は死あるのみ
結局、収束しないが正しいの?
もういいすか?
>>326
収束はしない。ただ解析接続はできる。
329132人目の素数さん:03/10/07 01:03
>>312
>>315
二度とくんな二度と数学すんな氏ねカス
ひっぱるな〜。もう見てないのかな?
331132人目の素数さん:03/10/07 01:05
>>328
で、316は正しいんでしょ?
332132人目の素数さん:03/10/07 01:08
302の出した問題にも、なんら誤りはない。
自明ななんちゃらを除く、には
誰も言わないのレスね…
334312:03/10/07 01:13
生きてくのいやになった
335312:03/10/07 01:15
>>330
いや、普通の方程式としてみたら負の偶数ってどう考えても自明じゃないと思うんだが・・・
336312:03/10/07 01:15
しまった。名前312のままだったw
>>333
そうです。誰が最初に我慢できなくなるかの我慢くらべでした。あなたの負け。
>>316 にはとても自明でないと
339132人目の素数さん:03/10/07 01:17
負の偶数は自明じゃにゃい
340132人目の素数さん:03/10/07 01:19
だから>>333の言う条件はいらないだろ。負の偶数は繰り込みとか言う
まったく訳わからん操作から求めてるんだからさ
341132人目の素数さん:03/10/07 01:19
>>337
あ、馬鹿発見w
342333:03/10/07 01:20
漏れ、負け組だか…
343132人目の素数さん:03/10/07 01:24
はい、おまつりしゅうりょ

平行六面体の体積Vと表面積Sについて、
S^3≧216V^2
が成り立つことを示せ。
344132人目の素数さん:03/10/07 01:25
で、結局何なの
345132人目の素数さん:03/10/07 01:26
333=312=315ですか?
346132人目の素数さん:03/10/07 01:28
>>345
そうだろ
347132人目の素数さん:03/10/07 01:29
>>343
「解いて下さい」とか「教えて下さい」くらい、言えば?
348132人目の素数さん:03/10/07 01:31
で、結局何なの。

方程式
1+1/2^s+1/3^s+1/4^s+1/5^s+1/6^s+1/7^s+1/8^s+・・・=0
の解sは、全てs=1/2+ti(tは実数)の形で表されることを示せ。

これは問題として間違ってるの、あってるの?誰か教えれ!!!

収束しないから間違い。
>>348 「負の偶数」「自明」でぐぐれ。
次世代のワイルズスレにでも逝ってこいよ。
さんざんガイシュツ。
352132人目の素数さん:03/10/07 01:38
>>349も・・・馬鹿か?
じゃあ学会で発表したら?リーマン予想の否定を証明できたってことで
一躍有名人だよ。フィールズ賞ももらえるかもねw
t の絶対値が sqrt(3)/2 より大きかったら収束するでしょ。
354132人目の素数さん:03/10/07 01:40
>>351
誰に言ってんの?
>>353
まじ?任意のσで?
>>354
勘違いしている馬鹿全員
357132人目の素数さん:03/10/07 01:43
>>354
おめぇだタコ
358132人目の素数さん:03/10/07 01:44
どっちが勘違いなんですか?また、どうしてですか?
359132人目の素数さん:03/10/07 01:44
>>356
どっちが勘違いなんですか?また、どうしてですか?

>>352
1+1/2^s+1/3^s+1/4^s+1/5^s+1/6^s+1/7^s+1/8^s+・・・
がRe s≦1で発散することもわからない馬鹿。
中途半端な知識で得意になるのはやめて、
少しはてめーの頭で考えろ、大馬鹿者。
361132人目の素数さん:03/10/07 01:47
>>355
任意でどうする。実部は1/2
虚部tが絶対値が sqrt(3)/2より大きい数
だろ。それがこの方程式の解の全てだ、ってのが予想なんだけどね
>>361
σ=1/2、|t|≧√3/2でディリクレ級数収束するの?しらんかった。何にのってるの?
363132人目の素数さん:03/10/07 01:53
>>360
おまえばっかじゃねぇの!!!?
s=1/2+14.1347....i とか
s=1/2+21.0220....i とか
s=1/2+25.0108....i とか
の値で0に収束するっつーのタコ
この大馬鹿者が
Re s>1の領域で、
1+1/2^s+1/3^s+1/4^s+1/5^s+1/6^s+1/7^s+1/8^s+・・・
と一致するような関数の零点の実部が1/2ということで、
1+1/2^s+1/3^s+1/4^s+1/5^s+1/6^s+1/7^s+1/8^s+・・・
の零点の実部が1/2というわけではない。
365132人目の素数さん:03/10/07 01:54
>>362
ゼータ関数の本。ディリクレ収束だかどうかは知らんが
366132人目の素数さん:03/10/07 01:56
1+1/2^s+1/3^s+1/4^s+1/5^s+1/6^s+1/7^s+1/8^s+・・・
と一致するような関数 ,とは!!?
>>365
もしかして解析接続できてる領域で収束してるって勘違いしてるんじゃないの?
納n=0,∞]z^nはσ<1に解析接続されるけどこの級数は|z|>1では収束しない。
つまり一般には収束する領域⊂≠解析接続される領域だよ?
>>363
おまえみたいな馬鹿をこれ以上相手すんのは嫌だから、
文献だけ提示しておく。
河田敬義「数論」岩波の209ページから熟読しとけ大馬鹿者。
369132人目の素数さん:03/10/07 02:00
ジェータ関数っていいな
370132人目の素数さん:03/10/07 02:03
>>368
お前も全然気づいてないな。それで数学科か?アフォめ・・・
君もその本もう一度熟読したら、わかるだろ
371132人目の素数さん:03/10/07 02:05
>>367
ちゃいまんがな。とにかくリーマン予想関係調べてみな
>>371
ほんとにちがうのか?あの級数収束する領域がσ≦1でも(解析接続可能じゃなくて)
級数自体が収束するってかいてある教科書なり論文なりがホントにあるの?ソースは?
373132人目の素数さん:03/10/07 02:10
ダメだ。終わってる
ガセか・・・寝よ。
375132人目の素数さん:03/10/07 02:12
こんな馬鹿を相手にする俺も馬鹿かもしれんw

>>368のP211
Σ_[n=1〜∞]a_n/n^sは、収束に関して次のいずれか。
(i) すべてのsに対して収束しない。
(ii) すべてのsに対して収束する
(iii) あるσ_0∈Rが存在しRe s>σ_0では収束し、Re s<σ_0では
  収束しない。Re s=σ_0の場合は収束する場合もしない場合もある。

直線Re s=σ_0を収束線という。リーマンゼータの場合の収束線は
Re s=1
377132人目の素数さん:03/10/07 02:15
378132人目の素数さん:03/10/07 02:17
>>376
いや、てかお前が馬鹿。普通の高校生だろ?お前。数学かぶれの。
379132人目の素数さん:03/10/07 02:19
(。”。)ん〜
自分の間違いに気づいた粘着が約1名絡んでいる模様・・・
>>376
おお、やっぱりそうか。σ<1では任意のtで収束しないんだな。やっぱりガセじゃん。
382132人目の素数さん:03/10/07 02:36
ここの連中には恐ろしく難解な問題だったわけだなw
383132人目の素数さん:03/10/07 02:45
問題を理解したらば、収束も糞もないことくらい。だって問題には
あの文章しかないんだからね同値変形できるだろ
あぁ、粘着もやだけど、理解できないしったかもやだ・・・
やっぱりリーマンは泣いてるねw
>>383から電波を感じます。
恐ろしく不愉快なちゃんねらをだせ!
http://science.2ch.net/test/read.cgi/math/1049652059/
というか、
1+1/2^s+1/3^s+1/4^s+1/5^s+1/6^s+1/7^s+1/8^s+・・・
について話してるのかζ(s)について話してるのかはっきりしろ。

もしもとの問題がζ(s)のことを言ってるなら、自明な零点が存在するから間違い。
自明な零点を除外するつもりなら、有名な未解決問題(Riemann予想)。
よって
・未解決問題は禁止(自分で解ける問題だけ)
のルールに反する。スレ違い。

もしもとの問題がΣn^(-s)のことを言ってるなら、この級数の収束領域は
{Res>1}∪{Res=1, Ims≠0}。よって問題自体が無意味。以上。

なお、この級数の収束領域は
Σ{a<n≦b}n^(-s)=(b^{1-s}-a^{1-s})/(1-s)-s\int{a to b}(x-[x]-1/2)/x^{s+1}dx+(b^{-s}-a^{-s})/2
をつかって求められる。
(Titchmarsh, the theory of the Riemann Zeta-function, p.14)
また、ここでa=1, b→∞とすることにより
ζ(s)=-s\int{1 to ∞}(x-[x]-1/2)/x^{s+1}dx+1/(s-1)+1/2(Res>1)
となる。

そして右辺がRes>0で収束し、正則関数を定義することから、
ζ(s)(Res>0)の表示を得ることができる。
388132人目の素数さん:03/10/08 01:02
>>301
きぼんぬ参照
スレ「n!(n+1)!=m!」のレス #54, #58, #62, #70-73
389132人目の素数さん:03/10/09 00:51
283をn次元に拡張できるか?
ただし外積はより一般的な交代積で置き換える。
|U(X)V|^2 = Σ[1≦i<j≦n] (U_i・V_j−U_j・V_i)^2
>>389
>>283?って?
391132人目の素数さん:03/10/09 01:22
>>343
隣り合う3面をS_a,S_b,S_cとする。
平行六面体の稜は4本づつ平行である。その1つの方向からこの六面体を眺めると、
S_aの射影である平行四辺形 S'_a が見える。
面S_a と対面の面間隔(垂線の高さ)を h_a 等とおく。
このとき、S_a ≧ S'_a = h_b * h_c / sin(α) ≧ h_b * h_c ・・・・・・(1)
ここに、α は S_b と S_c のなす二面角。
一方、S_b = V/h_b ・・・・・・(2)
S_c = V/h_c ・・・・・・(3)
(1)×(2)×(3)より、S_a・S_b・S_c ≧ V^2
これを S = 2・(S_a+S_b+S_c) ≧ 6・(S_a・S_b・S_c)^(1/3) に代入する。
等号成立は立方体。
q.e.d.
392282:03/10/12 00:10
問題:
P_k = (k次の基本対称式)/(項数nCk) とおく。
つまり、P_0=1, P_1=s_1/3, P_2=s_2/3, P_3=s_3.
このとき、 P_1 ≦ (P_2)^(1/2) ≦ ・・・・・ ≦ (P_k)^(1/k) ≦・・・・・≦ (P_n)^(1/n)を示せ。

ヒント:
P_(k-1)・P_(k+1) ≦ (P_k)^2 をk乗してその積をとる。

参考文献:
1. G.H.Hardy,J.E.Littlewood,G.Polya:"Inequalities",Cambridge (1934) §2.22 公式51-55
2. E.F.Beckenbach,R.Bellman:"Inequalities",Ergebnisse叢書,Springer(1961) p.11
393392:03/10/12 00:16
↑不等号の向きが反対。
このとき、P_1 ≧ (P_2)^(1/2) ≧ ・・・・・ ≧ (P_k)^(1/k) ≧ ・・・・・ ≧ (P_n)^(1/n) を示せ。
スマソ.
394388:03/10/12 01:03
B[0,1]を閉区間[0,1]から複素数上への有界関数の為す線型空間、
C[0,1]を閉区間[0,1]から複素数上への連続関数の為す線型空間とする。
それぞれノルムは絶対値の上限によって与えられるとする。

T:B[0,1]→C[0,1]を∀x∈C[0,1]に対してT(x)=xを満たす線型作用素とする時、
Tは有界でない事を証明せよ。
396392:03/10/12 14:37
ヒントの補足説明
h(t) = p_(k-1) + 2・p_k・t + p_(k+1)・t^2 が実根をもてば成立する。

F(t) = Π[i=1,n](t+a_i) = Σ[j=0,n](nCj)・p_(n-j)・(t^j),
f(t) = {(d/dt)^(n-k-1)}・F(t),
G(t) = (t^(k+1))・f(1/t),
g(t) = {(d/dt)^(k-1)}・G(t) とおけば
g(t) = (n!/2)・h(t)
等号成立は gが重根をもつ → Fがn重根をもつ → a_iが等しいとき.
397392:03/10/12 15:27
系(Sierpinski)
相加平均A=P_1, 相乗平均G=(P_n)^(1/n), 調和平均H=P_n/P_(n-1)とおくと、
A^(n-1)・H ≧ G^n ≧ A・H^(n-1)
398ななし:03/10/12 15:54
>>395
連続関数列で、その極限が連続関数にならない例を作れば良いだけ?
399132人目の素数さん:03/10/12 17:39
1+1=?
>>398
でも位相は一様ノルムで定まってるんだよ。
401ななし:03/10/12 18:40
>>398
あそっか。すまそです。
高校生に等周問題を説明せよ。
実数の部分集合全体から、自然数の有限部分集合への写像Tで
A⊂BならばT(A)⊂T(B)となるものは存在するか?するならば例を挙げよ
しないならばこれを示せ
>>403
∀A⊂R, T(A)={}
405404:03/10/13 16:41
なんか変だと思ったら
⊂が真部分集合を表すのかな。
そうだったらスマソ
>>403
T(A) = {1} など。任意の恒等写像が条件を満たす。
"⊂" が真部分集合の意味だったら『存在しない』。
以下、"⊂" は真部分集合の意味で使う。

整列定理により、R に整列順序を入れる。以下、この整列順序を ≦w で表す。
<w もこの順序の意味で使う。すなわち、a <w b ⇔ a ≦w b かつ a ≠ b
集合族 {A_i}[i∈R] を、次のように定める: A_i = { x | x ≦w i }
明らかに、任意の i, j ∈ R に対し i <w j ⇒ A_i ⊂ A_j
集合族 {A_i} の濃度は、自然数の有限部分集合全体 S のそれよりも真に大きい。
(S から自然数の有限列全体への単写があるが有限列全体はたかだか加算無限個)
従って任意の写像 T: {A_i} → S は単射とはならない。
換言すれば、ある i, j が存在し、A_i ⊂ A_j かつ T(A_i) = T(A_j) となる。
任意の T: P(R) → S を {A_i} に制限して考えれば上記の理由より条件を満たさない。

選択公理を使わずにやるには、少し考えないと。というか可能なのかな?
>>406
整列順序をどこで使ったのかよく分からない。。。
普通の順序でも同じ証明が通用しない?
P(R)の全順序かつ非可算な部分集合をつくればいいわけだから。
408簡単な証明:03/10/13 17:16
「⊂」が真部分集合の意味ならば…

実数の部分集合全体から、自然数の有限部分集合への写像Tで
>>403の条件を満たすものが存在すると仮定する。
このとき、T([0,1])も有限集合であるから、n=#T([0,1])とおく。

I_j=[0,2^{-j}]とおくと仮定より明らかに#T(I_j)<#T(I_{j-1})。よって、#T(I_j)≦#T(I_0)-j=n-j。
したがって#T(I_{n+1})<0となるが、これは矛盾である。

よって>>403の条件を満たすTは存在しない。
409406:03/10/13 17:52
>>407 確かに整列させる必要なかったですね(汗)。
>>408 smart!

(問) では T: P(R) → P(N) の場合はどうか?
410132人目の素数さん:03/10/14 00:26
任意の自然数Nに対して、N個の素数
a(1),a(2),……,a(N)
が存在し、これらが等差数列をなすようにできる事を示せ。

証明できたら神
>>410
未解決問題?
これ、確か未解決だったはず。
N = 11 くらいまでは条件を満たす素数が確認されてたはず。
>>412

Newsgroups: sci.math
From: [email protected] (Paul Pritchard)
Subject: 22 primes in arithmetic progression
Message-ID: <[email protected]>
Organization: Griffith University.
Date: Thu, 18 Mar 1993 02:42:42 GMT
Lines: 139

The first known arithmetic progression of 22 primes
has just been discovered:

11410337850553 + 4609098694200 i, 0 <= i < 22.

414132人目の素数さん:03/10/14 13:09
ミニロト(31個の数字の中から5個の数字を選ぶ)において
3個の当たり番号を含む券を必ず買うには最低何通りの組み合
わせを買えばよいか。ちなみに当たり番号も5個。
例えば、10個の数字から5個選び、3個の当たり番号を含む
券を必ず買うには、最低
12345、678910 のように2枚買えばよい。

教えてください。
415神光臨:03/10/14 21:23
>>410
a(1),a(2),……,a(N)
が全て等しい素数であるとき、公差0の等差数列になる。
このとき、任意のNについて、a(1),a(2),……,a(N)を作る事が可能。

Q.E.D.


さぁ、神とあがめろ!
416Which不一致 ◆v.V7zKGUME :03/10/14 21:27
晒しあげ
>>410>>415どっちの皿仕上げかはっきりしる!
実は>>417
419132人目の素数さん:03/10/19 03:41
n個の実数を a_1,a_2,・・・・・・,a_n、それらの差積を D とするとき、
|D|^2 ≦ n・Π(k=1,n-1) {Σ(i=1,n) |a_i|^(2k)}

n=3のとき、
|D|^2 ≦ 3(S1^2-2・S2){S1^4-4・S1^2・S2+4・S1・S3+2・S2^2}
ここに S1=a+b+c, S2=bc+ca+ab, S3=abc は基本対称式.
420Which不一致 ◆v.V7zKGUME :03/10/19 22:07
南海
421419:03/10/21 01:39
-補足-

一般式 |V|^2 ≦ Π(k=1,n){Σ(i=1,n)|v_(i,k)|^2} に
ファンデルモンド行列 v_(i,k)=(a_i)^(k-1) を代入する.

差積はファンデルモンド行列式だから.

ダイエー マンセー
422123人目の素人さん:03/10/23 02:35
さくらスレ3 の #99, 398, 444 & 531 も南海。
不等式ネタはもういい
不等式ヲタにはうれしいのだが・・・
425422:03/10/25 04:00
#531は等式でつだ。
ユークリッド3次元空間内を任意の形状により任意の領域区間に分けるとき
隣接する異なる領域区間を異なる色で彩色するようにすると、最低限必要な
ある特定の色の種類の個数は定まらず、上限個数が無いことを証明せよ。
>>426
紐みたいなので互いに繋げばいいだけじゃん。
小数点以下自然数を並べた数

0.12345678910111213....

は超越数であることを示せ。
Pを素数全体の集合とする。
Σ[n=1,∞] Σ[p∈P] 1/(p^n) < 1

を示せ。
430132人目の素人さん:03/10/29 02:28
n=1だけ見ても、Σ[p∈P](1/p) > 1/2+1/3+1/5 = 1+1/30.
431132人目の素数さん:03/10/29 02:36
>>429
間違えた・・・
Σ[n=2,∞] Σ[p∈P] 1/(p^n) < 1


逝ってくるわ。 他のスレでも出題間違ってるし・・・
a+b=1,b+d=2のとき、a+cは無理数であることを示せ
つーか、めっちゃ簡単だし・・・駄目だわ。今日は調子が悪い。

集合Sを1と素数からなる集合とする。Sの要素を小さい順から並べ
a(1)=S(2) -S(1)
a(2)=S(3) -S(2)
  ・・・
a(n)=S(n+1) -S(n)
とすれば、a(n)≧1 が成立する。 等号はn=1,2の時にのみ成立。

 Σ[n=2,∞] Σ[p∈P] 1/(p^n)
=Σ[p∈P] Σ[n=2,∞] 1/(p^n)
=Σ[p∈P] (1/p^2) * Σ[n=0,∞] 1/(p^n)
=Σ[p∈P] (1/p) * (1/( p - 1 ))
=Σ[p∈S,p≠1] (1/p) * (1/( p - 1 ))
<Σ[n=2,∞] a(n-1) * (1/S(n))*(1/(S(n) - a(n-1)))
=Σ[n=1,∞] a(n) * (1/S(n+1))*(1/(S(n))
=Σ[n=1,∞] 1/(S(n) - 1/(S(n+1)
=1/S(1)
=1


簡単すぎる・・・
434 :03/10/29 17:09
 ナンバーズ&ロトの解析と予想ほど難解な問題は無い。
435クリプト:03/10/29 20:53
1〜N(整数)までの数字をランダムにM個選び、そのうちのM-1つの数字を使い
四則演算だけで残り1つの数字を導き出すことのできるNとMを数列として求めよ。

ここにも書かせてもらいますね。


>>435
NとMを数列として求めよ。
この意味がわからん。
437132人目の素数さん:03/10/29 22:42
>>428
これって本当に成り立つの?
超越数ってなんだっけ・・・?
>>437
あんな数を根に持つ恥ずかしい多項式なんて、無いんじゃないの?
>>437
塩川先生の本で正しいとかいてあった記憶が・・・
441132人目の素数さん:03/10/29 23:15
書名は?
無理数と超越数、塩川宇賢、森北出版
いま読んでみたら正しいらしい。
0.12345678910111213....
はチャンパノウン数とかいうらしい。超越数だというのをK. Mahlerという人が証明したそうだ。
証明ものってる。(p122-125)
http://www.google.co.jp/search?q=0.12345678910111213&ie=UTF-8&oe=UTF-8&hl=ja&lr=
0.12345678910111213
でぐぐってみた。

割とたくさん載ってたな。

>>442
んで、証明は?
本見るのが面倒なんで書いてくれるとうれしい。

とか言ってみるテスト。
444クリプト:03/10/30 02:51
>>436
可能なNとMの組をすべてもってこいってことです。
445132人目の素数さん:03/10/30 03:33
N=5、M=3とすると、5C3=10だから、全部で10通りの
場合がある訳だが? 数列という意味が、今いちわからんな。
446445:03/10/30 03:39
そして、残りのひとつがそれぞれ3通りある訳だから、
全部で3×10=30通りの場合がある訳だがな・・・。
447445:03/10/30 03:47
具体的に申すと、
(1,2,3)(1,3,4)(1,4,5)(2,3,5)
の4つが「クリプト的」。残りの6つが「非クリプト的」と
言えそうだが?
448445:03/10/30 03:49
N,Mが小さい場合には、このようにすぐわかるね・・。
449445:03/10/30 04:16
待てよ。N>=Mとは限らんのかな? (簡単すぎるもんな)
そうすると、M>Nの場合は、重複を許すわけだ。
450445:03/10/30 04:18
ふ〜む
451クリプト:03/10/30 04:53
たとえばN,M=(3,2)、とか勿論一つのNに対してMは何個あってもいい。
これをN=f(n),M=g(n、f(n))の形で求めたら凄くない?
気に入ってもらえてありがとう。
452クリプト:03/10/30 06:06
M=g_i(n、f(n))だな。
それと、
1.重複を許さないでN>=M
2.重複を許してN>=M
3.N<M
に場合わけされるね。
わーい
難解かわからないが、これ解けるでしょうか?

http://science.2ch.net/test/read.cgi/math/1064879216/666
455132人目の素数さん:03/10/30 14:59
>>434
 ただの統計と確立なら話は簡単だが、+でカオスや複素係数まで
踏み込むと、手動計算では既にお手上げですよね!
456132人目の素数さん:03/10/30 15:53
確立じゃないぞ、ボケ!
457455:03/10/30 17:12
>>456
 スマン「確率」でした。
458132人目の素数さん:03/10/31 19:47
以下の証明を読んで次の問いに答えよ

「4以上の全ての偶数は2つの素数の和であらわせる」
言数μの儀環δ(μ)によって外数μ'/偶数は定位を持つ。(自明)
線形乖離により轍環はδによる写像σの約値を持つ。
轍環は無限順列を持たない為、輪位は定位と双対ではない。(μ'までも乖離される。)
律価をοとすると言群をMとし、単置換をπとすると、約値が相似単置換π'に相当し
∀{∀(∀σ , ∃π) ,∃π' s.t δμ=φ},∃ ο∈NM s.t δπο∽σπ'μ が言える

これを展開すれば、言数定理によって、乖離され、
δπμ'=φ となる為、補遊値は0になる。
自然数においてδの域数 ω(δ)=2,
πの弄数 Å(π)=2 であり、 ω(δ)Å(π)=4
補遊値=0だから4+0=4。

∴4以上の全ての偶数は2つの素数の和で表せる

[問い]
この証明の問題点を少なくとも2つ説明せよ。
なお、πの輪位性と定位性の両立については考慮しないものとする。
459ワケノさくら:03/11/01 03:10
>413
0: 11410337850553,
1: 16019436544753,
2: 20628535238953,
3: 25237633933153,
4: 29846732627353,
5: 34455831321553,
6: 39064930015753,
7: 43674028709953,
8: 48283127404153,
9: 52892226098353,
10: 57501324792553,
11: 62110423486753,
12: 66719522180953,
13: 71328620875153,
14: 75937719569353,
15: 80546818263553,
16: 85155916957753,
17: 89765015651953,
18: 94374114346153,
19: 98983213040353,
20: 103592311734553,
21: 108201410428753,
22: 112810509122953 = 61*1849352608573.
>>459
-1: 6801239156353, = 53*193*664897757.
461ワケノさくら:03/11/07 23:43
>460
Sankus.
22: 112810509122953 = 61*107*17283669239.
462132人目の素数さん:03/11/08 03:32
(sinθ)^(2/n)と(cosθ)^(2/n)がともに有理数となるような
θ、nの組があればそれを示し、なければ証明しなさい
>>462
qを適当な有理数とし、θ=π/4、n=1/log_[2](q)とおけば、(sinθ)^(2/n)=(cosθ)^(2/n)=q
>>463
>>462はnが整数という条件での出題だろう。
x=(sinθ)^(2/n), y=(cosθ)^(2/n)とおくとx^n+y^n=1になるから、Fermatの定理に帰着されるね。
だとしたら自明解ぬいとかんと。突っ込みどこ満載杉。
だせ。
467132人目の素数さん:03/12/03 01:45
【問題】
a,b,cは正の有理数, a+b+c=1 ⇒ (x^a)(y^b)(z^c)/(x+y+z)≦(a^a)(b^b)(c^c).
等号条件は x:y:z = a:b:c

おながいします.
x,y,zは任意の正の実数?
469132人目の素数さん:03/12/03 01:49
>467
x,y,zは正の実数。
未定定数法ですぐ終わるのでわ?
471132人目の素数さん:03/12/03 02:00
【問題】
a,b,c,A,B,C>0 のとき、
1/{(a^3)・(b+c)}+1/{(b^3)・(c+a)}+1/{(c^3)・(a+b)} ≧ (3/2)・(1/abc)^(4/3).
(A^2)/(B+C)+(B^2)/(C+A)+(C^2)/(A+B) ≧ (3/2)・(ABC)^(1/3).
472132人目の素数さん:03/12/03 03:32
俺以外にも不等式ヲタがいたか…
473132人目の素数さん:03/12/03 19:39
おたずねいたします。4次の多項式で吸引周期、
放物的周期、ジーゲル円板を同時にもつものを作成せよ。
できれば簡単な証明もあると助かります。よろしくおねがいします。


474132人目の素数さん:03/12/03 20:15
>>473
http://science2.2ch.net/test/read.cgi/math/1069642994/129


3 132人目の素数さん 03/04/07 03:27
●未解決問題は禁止
●宿題は自分で解け(自分の解ける問題だけ)
475132人目の素数さん:03/12/03 21:59
以下の系における運動体Kの奇跡を数式化せよ
x y 平面状に
V:X^2+y^2=(,7233x1,4959965x10^8)^2
E:x^2+y^2=(1,4959965x10^8)^2
J:x^2+y^2=(5,2026x1,4959965x10^8)2
S:x^2+y^2=(9,5549x1,4959965x10^8)2
とV E J S と4つの円があり、それぞれの円周上を球体 v e j s
が反時計回りに回転運動している 加えて原点にも静止球体SSがあるものとする
SS v e j s の速度 半径 並びに質量は以下の通り定める
SS:半径6,960x10^5 質量322946
v:0,615/s 半径6052 質量0,815
e:29,78/s 半径6378 質量1
j:13,06/s 半径71492 質量317,83
s:9,65/s 半径60268 質量95,16
今kはホーマン遷移軌道によりeを出発し 球体Vに近接軌道を2回行い
それによる増速および針路変更を経た後 jに向かう
再びjの影響における増速 針路変更を1回経てsを通過する
このような運動をkが行う場合のkの軌道方程式を求めよ
477476:03/12/04 01:11
訂正。
V:X^2+y^2=(,7233x1,4959965x10^8)^2 →V:X^2+y^2=(0,7233x1,4959965x10^8)^2
かっしーに
ありゃりゃ
480132人目の素数さん:03/12/04 19:44
本当に難しい問題は短文なのだよ。
短文よこしたまえ。
>>480
なら貴方、>>476を解けるのですか??
有理係数多項式f,gに対してf(Q)=g(Q)の時、
あるa,b∈Qがあってf(ax+b)≡g(x)となる事を示せ。
483132人目の素数さん:03/12/06 10:05
>467
x/a=X, y/b=Y, z/c=Z とおけば (X^a)(Y^b)(Z^c)/(aX+bY+cZ)≦1.
等号条件は X=Y=Z.

>471
A=1/a, B=1/b, C=1/c とおけば同じ式でつ.
484132人目の素数さん:03/12/12 21:51
あげ
【参考HP】
集団ストーカー被害(集団ストーカー行為とサイバーストーカー)
http://abff268r.infoseek.livedoor.com/stalker.htm

【関連ストーカースレ】
集団ストーカー被害者スレ
http://tmp.2ch.net/test/read.cgi/company/1055490461/
集団ストーカー被害者スレ2
http://tmp.2ch.net/test/read.cgi/company/1058775125/
集団ストーカー被害者スレ3
http://tmp.2ch.net/test/read.cgi/company/1060990002/
集団ストーカー被害者スレ4
http://tmp.2ch.net/test/read.cgi/company/1064380274/
集団ストーカー被害者スレ5
http://tmp2.2ch.net/test/read.cgi/company/1065826192/

集団ストーカー被害者スレ 生活版
http://life2.2ch.net/test/read.cgi/kankon/1057836761/
集団ストーカー被害者スレ 生活版 2
http://life2.2ch.net/test/read.cgi/kankon/1061590830/
集団ストーカーの実態スレ 生活板3
http://life3.2ch.net/test/read.cgi/kankon/1067586197/

探偵を使ったリストラ工作−6−
http://tmp2.2ch.net/test/read.cgi/company/1070536699/l50

ストーカー被害
http://society.2ch.net/test/read.cgi/soc/1056771332/
486132人目の素数さん:03/12/13 16:00
さくらスレ4にありますた。苦戦の様子。

206 :132人目の素数さん :03/12/09 22:04
a,b,cは三角形の3辺、rは内接円の半径、Rは外接円の半径のとき
 r/(2R) ≦ abc / sqrt{2(a^2+b^2)・(b^2+c^2)・(c^2+a^2)}
(出典 MOCP 2000.37)
http://www.cms.math.ca/Competitions/MOCP/2000/sol_oct.pdf
模範解答の2行目が意味不明です。(中略)
問題文に鋭角三角形とは書いてないから、正しくは
 = (b-c)^2・cos(A).
であって、0以上とは限らないんじゃないのですか?

254 :132人目の素数さん :03/12/10 00:18
206です。
とりあえず左辺を、次式を利用して a,b,cだけの式にしたら
6次式になって止まりました。
 S = abc/(4R) = r(a+b+c)/2.

こんな式か?
(a^2+b^2)・(b^2+c^2)・(c^2+a^2)・(a^2-|b-c|^2)・(b^2-|c-a|^2)・(c^2-|a-b|^2)
= (a^2+b^2)・(b^2+c^2)・(c^2+a^2)・(a+b-c)^2・(b+c-a)^2・(c+a-b)^2 ≦ 8・(abc)^4.

(注) 鋭角三角形のときは簡単でつ。(模範解答)
(b^2+c^2)・(a^2-|b-c|^2) = (b^2+c^2)・2bc(1-cos(A)) ≦ 2・(a^2)bc.
487132人目の素数さん:03/12/13 22:40
>471
"上": A=1/a, B=1/b, C=1/c とおけば"下"と同じ式でつ。
IMO-1995(Canada) Q2

"下": Let S=(A^2)/(B+C)+(B^2)/(C+A)+(C^2)/(A+B).
Using Cauchy inequalty,
2(A+B+C)・S = {(B+C)+(C+A)+(A+B)}・S ≧ (A+B+C)^2.
∴ S ≧ (A+B+C)/2 ≧ (3/2)・(ABC)^(1/3).
便乗質問。
正の数a,b,cに対して、次の不等式を示せ。
3{a/(b+2c) + b/(c+2a) + c/(a+2b)}
 ≧ (a+b+c){1/(b+2c) + 1/(c+2a) + 1/(a+2b)}
神降臨の予感、煽り降臨の悪寒。
490132人目の素数さん:03/12/14 01:48
>>487
IMO-1995(Canada) Q2は abc=1という条件不等式なのだが・・・
(A^2)/(B+C)+(B^2)/(C+A)+(C^2)/(A+B)≧(3/2)・(ABC)^(1/3)
は同次だからABC=1という束縛条件は導入しても一般性をうしなわないのでは?
492132人目の素数さん:03/12/14 02:07
もうちょっと分かりやすくお願いします。
493132人目の素数さん:03/12/14 02:30
>>476
これGTOで麗美が家庭教師に出した問題じゃないか?
>>493
カッシーニの起動計算屋根
495132人目の素数さん:03/12/14 02:42
分かるかな?

s>0とする。
正項級数納n=2,∞]1/{n(logn)^s}の収束、発散について議論せよ。
>>495
sは複素数?
>>496
みたいだね。
>>495
Res=1上がむずい・・・
なんとなくRes=1上ではs=1で発散、s≠1で収束しそう。
発散しつつ500ゲト
501132人目の素数さん:03/12/14 20:24
>>495
答えキボンage
ん?>>495

Re(s)>0とする。
級数納n=2,∞]1/{n(logn)^s}の収束、発散について議論せよ。

って問題なんだね。確かに難しそうだ。
503132人目の素数さん:03/12/14 22:15
http://www5e.biglobe.ne.jp/~yb_net/clip/img/3553.jpg

タイ━━━━||Φ|(|゚|∀|゚|)|Φ||━━━━ホ!! されますた!!!!!
うんこー
505495:03/12/15 01:22
書き忘れます田…sは複素数ではありません…
506132人目の素数さん:03/12/15 16:11
  三|三
  イ `<             ,..-──- 、         _|_
   ̄             /. : : : : : : : : : \        |_ ヽ >>503-505
   ∧           /.: : : : : : : : : : : : : : ヽ      (j  )
   /  \        ,!::: : : :,-…-…-ミ: : : : :',
              {:: : : : :i '⌒'  '⌒' i: : : : :}     _ヽ_∠
  └┼┘          {:: : : : | ェェ  ェェ |: : : : :}       lニl l |
.   |_|_|  , 、      { : : : :|   ,.、   |:: : : :;!      l─| l 亅
   __   ヽ ヽ.  _ .ヾ: :: :i r‐-ニ-┐ | : : :ノ        _
    /     }  >'´.-!、 ゞイ! ヽ 二゙ノ イゞ‐′      l  `ヽ
   ´⌒)    |    −!   \` ー一'´丿 \       l/⌒ヽ
    -'    ノ    ,二!\   \___/   /`丶、      _ノ
        /\  /    \   /~ト、   /    l \
       / 、 `ソ!      \/l::::|ハ/     l-7 _ヽ
      /\  ,へi    ⊂ニ''ー-ゝ_`ヽ、    |_厂 _゙:、
      ∧   ̄ ,ト|    >‐- ̄`    \.  | .r'´  ヽ、
507132人目の素人さん:03/12/15 21:40
>486
さくらスレ4にありますた。解決の様子。

369 :132人目の素人さん :03/12/15 21:34
鋭角三角形の場合は模範解答が有効なので、Aが鈍角の場合を考えまつ。

= (bc/2)sin(A) ≦ bc/2 ・・・・・・ (1).
A≧π/2 より, (b^2+c^2) ≦ a^2 ・・・・・・ (2).
となるので、残りの (a^2+b^2)(a^2+c^2) が 2(as)^2 より小なることを示しまつ。
F = 2(as)^2 - (a^2+b^2)(a^2+c^2)
= (1/2)(a^2+2bc)(a^2-b^2-c^2)+(b+c-a)a^3 +bc(b^2+bc+c^2)
> (1/2)(a^2+2bc)(a^2-b^2-c^2).
ここで A≧π/2 なので、F>0.
∴ (a^2+b^2)(a^2+c^2) < 2(as)^2 ・・・・・・ (3).

上記 (1)^4 ×(2)×(3) より,
(a^2+b^2)(b^2+c^2)(c^2+a^2)・(竸4) < (1/8)(abc)^4・(s^2).
∴ r/(2R) = 2(竸2)/(abcs) < abc/sqrt{2(a^2+b^2)(b^2+c^2)(c^2+a^2)}.
508132人目の素数さん:03/12/15 22:08
f(x), g(x)を互いに素な多項式とし、方程式f(x)g(x){f(x)+g(x)}=0の
相異なる解の個数をnとする。このとき

max{deg f, deg g}≦n-1

であることを示せ。
509507:03/12/15 22:12
=(ABCの面積)
s=(a+b+c)/2 が抜けてますた......すまそ。
510132人目の素数さん:03/12/15 22:35
>495
s∈R なら積分判定法を使えば?
I(s) = ∫[e,∞) 1/[x{ln(x)^s}] dx = ∫[1,∞) {1/(u^s)}du.
For s≠1, I(s) = [1/{(s-1)u^(s-1)}](u:1→∞)
In case s<1, I(s) →∞ (diverge)
In case s>1, I(s) = 1/(s-1). (converge)
In case s=1, I(1) = [ln(u)](u:1→∞) = ∞ (diverge)

>505
sが複素数でないなら、 s∈R でもないでつが......
>>508
学コンの宿題に告示してる
512132人目の素人さん:03/12/15 22:52
>488
便乗レス。
b+2c=x, c+2a=y, a+2b=z とおくと、
3・(a/x+b/y+c/z)-(a+b+c)・(1/x+1/y+1/z) = (2a-b-c)/x+(2b-c-a)/y+(2c-a-b)/z
= (y-x)/x+(z-y)/y+(x-z)/z = (y/x + z/y + x/z)−3 ≧ 3・1^(1/3)−3 = 0.
最後のは「相加平均≧相乗平均」でつ。等号成立はx=y=z, a=b=c.
>>507
ありがとう。
さくらスレに解答されていたとは気づきませんでした
>>510
Re(s)が収束横座標に等しいとき一般にはむずかしくなるのでは?
この問題ならRe(s)<1で発散、Re(s)>1で収束はいえてもRe(s)=1ではs=1で発散するからと
いってRe(s)=1で発散するとはいえないんじゃないの?
>>512
ありがとうございます。
似たようなやつですが、こやつもお願いします。
正の数a,b,cに対して、次の不等式を示せ。
a/(b+2c) + b/(c+2a) + c/(a+2b)
 ≧ (a+b+c)^2/{(b+2c)(c+2a)(a+2b)}
a/(b+2c) + b/(c+2a) + c/(a+2b)
 ≧ (a+b+c)^3/{(b+2c)(c+2a)(a+2b)}
のような伊予柑
>>516
>>515は知り合いから出された手書きの問題なので
どうだったかぁゃιぃです。
518132人目の素人さん:03/12/16 21:04
>515
分母を払って
a(c+2a)(a+2b)+b(a+2b)(b+2c)+c(b+2c)(c+2a)−(a+b+c)^3
= a(a-c)^2+b(b-a)^2+c(c-b)^2 ≧ 0.
∴ a/(b+2c)+b/(c+2a)+c/(a+2b) ≧ (a+b+c)^3/{(b+2c)(c+2a)(a+2b)} ≧ 1.
最後のは「相加平均≧相乗平均」でつ。 等号条件は a=b=c.
>>515
さすがスヌーピー! そこに痺れる憧れるぅ!
ありがと〜う! (AA見つからず)
>>507
F = 2(as)^2 - (a^2+b^2)(a^2+c^2)
= (1/2)(a^2+2bc)(a^2-b^2-c^2)+(b+c-a)a^3 +bc(b^2+bc+c^2)
のところですが、第3項は bc(b^2-bc+c^2) ですね。
         /  / _,.-、___,へ/ ヽ、ー--、-、  \´/∠彡!_            -+-
 _,.-‐--、 / /⌒/ / ヽ/  ヽ   \  \-、   \二/7ヘ           (_レ'ヽ
r'     )/ /   /       ト、 ヽ、 、ヽ、\\!  }三_ノ彡}
[`ー---‐‐! /   /  /  ,イ ト l \ l  l ヽ ヽ、 \\〈二ニ´ 〈           レ |
L_=ニ三三/  /  /  / l l ヽ、  ー! L__! Lヽ l } }二 ̄ ̄]            ノ
 〈 ̄ ̄ l  /   ハ  レト、!\ヽ  | ,!,;=l=;;、「_ト、| | ,イ_=ニ三j´         -+-、ヾ
 !二ニ=!  l     レi !,,,,,_ ヽヽ ヾ! レ"i!ー、:.ヾヾ!ハ ! | >--]           l ノ
 >-‐‐! |ヽ、| l   L_!〃‐ヾ' `ヽ!    l!-クO:l!i/r‐く/ヽ `ー-'
 ヾー--'レヽトl ヽ  ト」!|ト‐ク:}i,       ヾcッハj!(ヽヽ∨ / ̄`ヽ          ヽ-‐
  ヽ二_/ ヽ-ヽ、 !〈lハo();;ッi!,_   、    ´  ̄`|(ヽ !'! /     \        (_____
   [三二j  i i⌒| トi 〉ゞ=''"`   _,. -┐ lヽ / 〉 !| ヽ   ヽ   \
    ラ_ノ____ゞ、_l l |/ ) | (ヽ  「    l  ! l/ / lト   |  / __   \      (
      /    /ハ l ! ヽ j !  ヽ.  ノ  ,!  /   | |  l   / ー-、 /       )
     /     l l ヽ \   ヽ、    ̄ / l {  / ! _ /  ,.-'" ̄ヽ      (
    /     ヽ ヽヽ \  ヽ  ̄ ´   \   `\ヘ \-‐'"    _,.-ヽ    !7 !7
   /   /  \   }__\ `   }ー-、__,,.-'",/\    /   `ー-、-'"´    \   o o
522132人目の素数さん:03/12/16 22:41
ふと疑問に思ったこと。>>488>>515の2つの不等式

a/(b+2c) + b/(c+2a) + c/(a+2b)
 ≧ {(a+b+c)/3}{1/(b+2c) + 1/(c+2a) + 1/(a+2b)} ≧ 1
a/(b+2c) + b/(c+2a) + c/(a+2b)
 ≧ (a+b+c)^3/{(b+2c)(c+2a)(a+2b)} ≧ 1

において、真ん中の項の大小は定まるのだろうか?
と思い、苦戦中。
523132人目の素人さん:03/12/17 20:52
>522
b+2c=x, c+2a=y, a+2b=z, (x+y+z)/3=A, (xyz)^(1/3)=G, 3/(1/x+1/y+1/z)=H とおくと、
(上) = (A/3)・(3/H) = A/H, (A/G)^3 = (下)

(G^3)/H = (1/3)(xy+yz+xy) ≦ (1/3)(xy+yz+zx)+(1/9){(x-y)^2+(y-z)^2+(z-x)^2}
= (1/9)(x+y+z)^2 = A^2.
∴ (上) = (A/H) ≦ (A/G)^3 = (下).

nコのときは....
【Lemma】(Sierpinski)
For additive mean(A), geometric mean(G), harmonic mean(H)
of n positive numbers(n≧2),
A・H^(n-1) ≦ G^n ≦ A^(n-1)・H.
Equality: all n members are identical, or n=2.(説明略)
【Corollary】
H≦G≦A (∵H≦A)
5241/2:03/12/17 22:20
>>495
Re s>1, Re s<1のときは既に議論されてるので、Re s=1のときだけ。

a, bを整数とし、f(x)を[a, b]上のC1級関数、G(x)=Σ{n≦x}g(n)とおくと、
Σf(x)g(x)=Σ{a<x≦b}f(x)(G(x)-G(x-1))=Σ{a≦x<b}(f(x)-f(x+1))G(x)+f(b)G(b)-f(a)G(a)
a<x≦b
=f(b)G(b)-f(a)G(a)-∫[a,b]f'(t)G(t)dt

f(x)=1/{x(logx)^s}, g(n)=1とおくと、

G(x)=[x]=x-{x}({x}はxの小数部分)
f'(x)G(x)={s(logx)^(s-1)+(logx)^s}(x-{x})/{x^2 (logx)^(2s)}
=s/{x(logx)^(s+1)} + 1/{x(logx)^s}-O(x^(-2))

よって1≦a≦bならば

Σ{a<x≦b}f(n)=(logb)^(-s)-(loga)^(-s)-∫[a,b]s/{x(logx)^(s+1)}+1/{x(logx)^s}dx+O(a^(-1))

∫[a,b]dx/{x(logx)^2}=O(1/aloga)よりRe s=1ならば

Σ{a<x≦b}f(n)=∫[a,b]dx/{x(logx)^s}+O(1/loga)

となる。
(続く)
5252/2:03/12/17 22:21
logx=uとおくと、∫[a,b]dx/{x(logx)^s}=∫[loga, logb]u^(-s)du.

s=1のとき、∫u^(-s)du=log uよりこの積分はbとともに∞に発散。
s≠1ならば∫u^(-s)du=-u^(-ti)/ti(s=1+ti)よりこの積分は振動する。

したがって、Re s=1のとき、Σf(n)は収束しない。Q.E.D.

なお、Re s>1, Im s≠0のとき、∫[a,b]dx/{x(logx)^s}=∫[loga, logb]u^(-s)du
=a^(1-s)/(1-s)-b^(1-s)/(1-s)→a^(1-s)/(1-s)(as b→∞)となることを
利用すると、Σf(n)をRe s>0, s≠1まで解析接続できる。
(ζ関数の値を研究するときにはこの方法が出発点となる)
計算あってる?
527132人目の素数さん:03/12/18 02:39
2元2次形式の判別式Dが正である不定値形式のreduced formを求めよ
528132人目の素数さん:03/12/18 02:41
>>523
ありがとうございます。
もしかして additive mean → arithmetric mean でしょうか。
すみません。
>>523
Sherpinskiの不等式を探したら、
「不等式への招待」P.79 で見つけました。
まだまだ知らんことばかり…。
いつもありがと〜(AA略)
>>522-523より、正の数 a,b,c に対して
a/(b+2c) + b/(c+2a) + c/(a+2b)
 ≧ (a+b+c)^3/{(b+2c)(c+2a)(a+2b)}
 ≧ {(a+b+c)/3}{1/(b+2c) + 1/(c+2a) + 1/(a+2b)}
 ≧ 1
が成り立つことが分かった。ここで問題。a,b,c,p,qを正の数として
 a/(pb+qc) + b/(pc+qa) + c/(pa+qb)
に対しても同様の不等式を作れ。
>>530
a/(pb+qc) + b/(pc+qa) + c/(pa+qb) ≧ 3/(p+q)

532132人目の素数さん:03/12/19 00:26
527難しすぎた?
>>532
>>527は意味がわからん。いちいち意味しらべて解く香具師なんかいねー
>>533
残念だな。
君を数ヲタと思っていたのだが…。
535132人目の素数さん:03/12/19 19:37
意味分からん。電波か。
536523:03/12/20 13:24
>530,531
a/(pb+qc) + b/(pc+qa) + c/(pa+qb)
≧ ((a+b+c)/3){1/(pb+qc) + 1/(pc+qa) + 1/(pa+qb)} ≧ 3/(p+q).
最後のは「相加平均≧調和平均」でつ。等号条件は a=b=c.

pb+qc=x, pc+qa=y, pa+qb=z, A=(x+y+z)/3=(a+b+c)(p+q)/3,
H=3/(1/x+1/y+1/z) とおくと、A/H≧1.
537536:03/12/20 19:29
左側は 1/2≦p/q≦2 がないと無理かも....
538132人目の素数さん:03/12/20 20:08
>467,483
x/a=X, y/b=Y, z/c=Z とおけば、
F = (x^a)(y^b)(z^c) / (a^a)(b^b)(c^c) = (X^a)(Y^b)(Z^c).
G = x+y+z = aX+bY+cZ.
ここで a,b,c∈Qだから通分して a=A/n,b=B/n,c=C/n とする(A,B,C,n∈N)。
題意より A+B+C=n.
F = {(X^A)(Y^B)(Z^C)}^(1/n) = 相乗平均.
G = (AX+BY+CZ)/n = 相加平均.
∴ F≦G.
539523:03/12/20 20:48
>528
そうでつね。
>529
A(H^(n-1))/(G^n) ≦ 1 ≦ (A^(n-1))H/(G^n)
がnについて単調であることを用いて、代数で出せまつ。
また、[397] は基本対称式を使ってまつ.
540132人目の素数さん:03/12/20 23:01
時間がないのですどなたか助けてください

あるホルモンの血中濃度の継時的変化を個体別に調べ
各時間ごとの平均値を取りました。
それをグラフにすると、ある時間にピーク値が集まっていました。
このことから「〜時に血中濃度が高くなる傾向がある」と言いたいとき
どのような統計解析を行えばいいのでしょうか?
541132人目の素数さん:03/12/20 23:04
542132人目の素数さん:03/12/21 01:41
すまん。

http://seraph.mistyhill.org/under/3rdstr.htm

_| ̄|○タノム
543132人目の素数さん:03/12/21 03:29
>>542
いい子だから死にたまえ!

○ミ
       ○  
       \) ̄
  _| ̄|ミ <
544132人目の素数さん:03/12/21 03:48
すいません。。。スロ板から来たおつむの出来が少々悪過ぎる20歳ですが
質問して良いでしょうか?
確率についてなのですが、合成比率の出し方と説明をしていただきたいです。
(公式と何故そうなるのか)
とりあえず1/100と1/200の合成比率だと3/200≒1/66.6で
まず分母の最小公倍数(?)を出してその数値になるまで分母を掛けた後
その掛けた数を分子に入れて足し、
足した分子/最小公倍数=合成比率と言うことでよろしいですか?
>>544
超絶にスレ違いな感が否めない・・・がマジレスしてみる。

合成比率という単語はそもそもなんぞや?
ややこしい単語を使って分数の足し算をやっているようにしか見えない。

どういう状況下でそういう計算に迫られたのかを書くべし。

1/100の確率で事象Aが発生し、1/200の確率で事象Bが発生し、AとBが同時に起こりえないとき、
AまたはBのいずれか一方でも発生する確率の計算をしているのであればあっているのだが。
546132人目の素数さん:03/12/21 04:51
ネタにマジレス救われない。
547132人目の素数さん:03/12/21 05:12
>>545
すんません、気付かない間にスロット用語使ってたようですw
まずこれはスロットの大当たり確率の話でして
スロットにはビッグボーナスとレギュラーボーナスがあり
これらは大抵別々で抽選されています。はたまたATと言われるボーナスもあり
これらを総合的に引き当てる確率を計算したかったのです。(合成比率)
ちなみに最後の2行見るからに自分の計算合ってるようで安心しましたw

>>546
俺の中じゃ恐ろしく難解だったんです・・・
548544:03/12/21 05:16
と言うか・・・これは分数の足し算だったのかΣ( ̄◇ ̄;)!!
脳みその衰えって怖いですねw
・・・世の中に分数の足し算が出来ない人間が実在することを改めて実感する瞬間。
550132人目の素数さん:03/12/21 05:57
塾で高校生に教えているけど、
分数の足し算ができない進学校の生徒は沢山いるよ。
最近よく見かけるのが、2a=3bをaについて解けない人たち。
551544:03/12/21 06:19
>>550
それなら分かる!w
3b/2でそ?
552132人目の素数さん:03/12/21 06:35
マジレスしてる時点で、このスレのレベルの低さが(ry
553544:03/12/21 07:09
>>552
てめぇそれは俺のことを言ってるんですか?
ごめんなさい。レベルが低いのは俺一人なんです。
554523:03/12/21 11:42
(説明補足)
【Theorem】(Sierpinski)
For additive mean(A), geometric mean(G), harmonic mean(H)
of n positive numbers(n≧2),
A・H^(n-1) ≦ G^n ≦ A^(n-1)・H.
Equality: all n members are identical, or n=2.

n=2 のとき AH=G^2 なので、下の補題から出まつ。
Arithmetic mean(A), Geometric mean(G), Harmonic mean(H) for n positive numbers a_1,a_2,・・・・・・, a_n とおく。また、
A', G', H' for n+1 positive numbers a_1, a_2,・・・・・, a_n & a' とおく。

【Lemma】 D.S.Mitrinovic and P.M.Vasic (1976)
Let F=(A^(n-1))H/(G^n) and F'=(A'^n)H'/(G'^(n+1)), then F'≧F.

証明はむずかしくない。
A'=(nA+a')/(n+1), G'^(n+1)=(G^n)a', H'=H・{a'(n+1)/(H+na')}.
とおけるから、
F/F' = {(A^(n-1)/(A'^n)}{(H+na')/(n+1)} = {(A^(n-1)/(A'^n)}{nA'-(n-1)A-(A-H)/(n+1)} ≦{(A^(n-1)/(A'^n)}{nA'-(n-1)A} = {nx-(n-1)}/(x^n)
where x=A'/A.
x^n-nx+(n-1) = ((x-1)^2)Σ(k=0,n-2)(n-1-k)(x^k)≧0 (for x≧0)
したがって
F'≧F (n≧2)
を得る。(Fは単調増加)
もう一方の不等式も全く同様にして示される。q.e.d.

途中で A-H≧0 を使いますた。これは
A/H = (1/n^2){Σ(i=1,n)a_i}{Σ(j=1,n)1/a_j} =1+(1/n^2)Σ(i<j){(a_i/a_j)+(a_j/a_i)-2} =1+(1/n^2)Σ(i<j){(a_i-a_j)^2}/(a_i・a_j) ≧1.

【Reference】『不等式への招待』第2版
おぉ〜、523様。(先生と呼ばせてもらいます)
いつも不等式の問題を解いてもらってありがとうございます。
556132人目の素数さん:03/12/21 14:55
スロットの問題は笑った(^^)
数学を難しく考えすぎる人多いよな〜なんでやろ??
557392:03/12/23 15:10
[392]ヒントの補足説明(Supplement)
【Theorem】For n positive numbers a_1, a_2, ・・・・,a_n,
we denote P_k = (k次の基本対称式)/(項数nCk), then
Q_k = (P_k)^2 - P_(k-1)・P_(k+1) ≧ 0.

【説明】nに関する帰納法(induction)による。
For n=2, Q_1 = (P_1)^2 - P_2 = (1/n^2)・{Σ(i=1,n) a_i}^2 - (2/n(n-1))・{Σ(i<j) a_i・a_j} = (1/n^2(n-1))Σ(i<j) (a_i-a_j)^2 ≧ 0. で成立。

nのとき成立するとする。
For n+1 positive numbers, a_1, a_2,・・・・・,a_n,a', we denote P'_k (0≦k≦n+1). Then,
P'_k = {(n+1-k)・P_k+k・P_(k-1)a'}/(n+1).

(n+1)^2・Q'_k = (n+1)^2・{(P'_k)^2-P'_(k-1)・P'_(k+1)}
= (n-k)(n-k+2)・Q_k+(n-k)(k-1)・{P_(k-1)・P_k-P_(k-2)・P_(k+1)}a'+(k^2-1)Q_(k-1)a'^2
+ {P_k-P_(k-1)a'}^2
≧ (n-k)(k-1)・{P_(k-1)・P_k-P_(k-2)・P_(k+1)}a'.
= (n-k)(k-1)・P_(k-1)・P_(k+1){P_k/P_(k+1)-P_(k-2)/P_(k-1)}a' ≧ 0.
where we used P_k/P_(k+1) ≧ P_(k-1)/P_k ≧ P_(k-2)/P_(k-1).
∴ Q'_k ≧ 0. (q.e.d.)

【Note】 P_0=1, P_1=A, P_n=G^n, P_n/P_(n-1)=H.
[396]では微分法を使いますたが、今回は代数だけでつ。
558132人目の素数さん:03/12/23 19:37
pを6m+1(m: 整数)の形の素数とする。このとき

x^4+y^4+z^4=2*p^n

は無限に多くの整数解(x, y, z, n)を持つことを示せ。
>>558
まじすか??どうやるの??なににのってるの???
>>538が謎ですが、詳しい説明をお願いします。
538のように置き換えて n=3 とすると、示すべき不等式は
 (AX+BY+CZ)/3 ≧ (X^A Y^B Z^C)^(1/3)
ここから、どのように相加相乗を使うのですか?
561132人目の素数さん:03/12/23 21:51
√5はいくつかおしえてください
>>561
電卓さんか
Googleさんに訊いてみましょう。
http://www.google.com/search?num=50&lr=lang_ja&q=5^(1/2)
>>560
(A,B,C)=(0,1,2)のとき(Y+Z+Z)/3≧(YZZ)^(1/3)。
(A,B,C)=(1,1,1)のとき(X+Y+Z)/3≧(XYZ)^(1/3)。
564132人目の素数さん:03/12/24 16:13
>>563
アフォかヴァカかと小一時間…
565538:03/12/24 21:29
>560
XがA個、YがB個、ZがC個 の合計n個の平均を考えまつ。(A+B+C=n)
n=3 のときは [563] のようになると思われ。
566557:03/12/24 21:50
-Amendment-
For n≧2 and 1≦k≦n-1, Q_k = (P_k)^2 - P_(k-1)・P_(k+1) ≧0.

For n=2 or k=1,
Q_1 = (P_1)^2 - P_2 = (1/n^2)・{Σ(i=1,n) a_i}^2 - (2/n(n-1))・{Σ(i<j) a_i・a_j}
= (1/n^2(n-1))Σ(i<j) (a_i-a_j)^2 ≧ 0. で成立。
567ティム:03/12/25 01:17
http://science2.2ch.net/test/read.cgi/math/1072161439/l50
ある意味恐ろしく難解な問題だな
>>563-565
なるほど。
569392:04/01/02 21:34
392です。
あけますて おめでとう ございまつ。
昨年のまとめをすれば、
[396],[557] ⇒ [554](Sierpinski) ⇒ H≦G≦A.
でしょうか。 では本年も宜しく。

[557]では便宜上 P_(n+1)=0, P'_(n+2)=0 とおくのがいいようです。
とき方を教えてください。
△ABCの面積が10√3、cosB=1/7、cosC=11/14のとき
a、b、c、Aを求めよ。
という問題なのですが・・
0≦Θ<2πのとき次の不等式を満たすΘの値の範囲を求めよ。
sinΘ≦tanΘ
572132人目の素数さん:04/01/02 23:17
>>571
他の人もあなたと同じ問題で悩んでるようですよ
二人で考えてみたらいかがですか?
573132人目の素数さん:04/01/03 00:07
>>570
辺bについて預言低利を 用いるんだ、あくまでもかっこよくな。
さらに、面積からbとcの関係が求まるだろ?この時涙流しちゃいけないぜ
上の二式を連立させて因数分解してみろよ。まわりの人間をうっとりさせるんだ!。
あと、cos(B+C)の値も求めな子猫ちゃん
>>570
 sinB=√{1−(1/7)^2}=4√3/7、 sinC=√{1−(11/14)^2}=5√3/14
 sinA=sin(π−B−C)=sin(B+C)=sinB・cosC+sinC・cosB=4√3/7・11/14+5√3/14・1/7=√3/2
 cosA=cos(π−B−C)=−cos(B+C)=−cosB・cosC+sinC・sinB=−1/7・11/14+5√3/14・4√3/7=1/2
よって、∠A=60°。次に、三角形の面積の公式より、
 10√3=1/2・bc・sinA=bc√3/4=1/2・ca・sinB=ca2√3/7=1/2・ab・sinC=ab5√3/28
⇔ bc=40,ca=35,ab=56
第二式を第一式で割って a=7b/8 で、これを第三式に代入し b^2=8^2 つまり b=8。他の式に代入し、a=7,c=5。

>>571
sinθ≦tanθ=sinθ/cosθ … @ だから、
(1) 0≦θ<π/2 または 3π/2<θ<2π つまり cosθ>0 のとき
  @ ⇔ sinθ(1−cosθ)≧0 ⇔ sinθ≧0 ⇔ 0≦θ≦π
 前提条件と合わせて、 0≦θ<π/2
(2) π/2<θ<3π/2 つまり cosθ<0 のとき
  @ ⇔ sinθ(1−cosθ)≧0 ⇔ sinθ≦0 ⇔ π≦θ<2π
 前提条件と合わせて、 π≦θ<3π/2
(1),(2)合わせて、0≦θ<π/2 または π≦θ<3π/2
>>570>>571 「恐ろしく難解な」宿題ばかりだなw 次回から、宿題は

くだらねぇ問題はここへ書け ver.3.14(25桁略)2795
http://science2.2ch.net/test/read.cgi/math/1069034436/
分からない問題はここに書いてね146
http://science2.2ch.net/test/read.cgi/math/1073028647/
厨房攻防、宿題テスト支援スレ2(答え教えたる)
http://science2.2ch.net/test/read.cgi/math/1049714540/

で質問してください

>>570 加法定理を習っていれば>>573>>574でいいけど
多分、高1で加法定理を習っていないだろうから=加法定理を使ってはいけないなら
正弦定理
a/sinA=b/sinB=c/sinC ⇔ sinA=(a/b)sinB=(a/c)sinC
より sinA をa、b、cで表わす。
[解法>>573] cosA をa、b、cで表わす
 cosA、cosB、cosC それぞれについて余弦定理の式をつくり、a、b、c、cosAを求める
[解法>>574]三角形の面積の公式と正弦定理からa、b、c、sinAを求める

>>571 式が与えられたら、分母≠0 を真先にチェックすること
tanθ=sinθ/cosθだから、cosθ≠0 ⇔ θ≠π/2、3π/2 ですね
対数の式が与えられた場合も、0<底<1 または 底>1、真数>0 を真先にチェックしましょう

576132人目の素数さん:04/01/05 00:24
>>944
漏れもスロ板住人だYO!・・・スロ板住人を数学板で初めて見た気がする。


某スレでちらっと話題に出てたことを問題にしてみました。
未熟者の漏れが作った問題なのでマズい点がありそう・・・指摘して下さると嬉しいです。

問題
黄金比「(√5-1)/2:1」、白銀比「√2:1」、に続く「赤銅比」なるものを作りたい。

条件1:黄金比と白銀比の関係を元に赤銅比を作らなければならない
条件2:その関係の中で、黄金比より優れる比が存在してはならない
条件3:優れている順に「黄金比>白銀比>赤銅比」となるようにする
条件4:「視覚的な美しさ」は優劣の基準にはなり得ない

以上の条件より
・赤銅比を作ることは可能か?
・可能ならばどのような比になるか?
以下の各視点ごとに論ぜよ

(1)正n角形の一辺:対角線
(2)連分数展開
(3)プラトン立体同士の相貫体の頂点を結んで出来る菱形n面体の表面対角線比


(2)はいくらでも考え方がありそう。
(3)の視点から論じたら面白いんだろうな、とは思うけどさっぱり分かりません・・・

他の視点でも構いません。高校卒業程度の知識しかない漏れにはおそらく理解不能ですが・・・
577:04/01/05 04:46
 mathmaniaに問題を捧げる。
【超難問:組み合わせ】
円周上に相異なるn個の赤い点とn個の青い点を取る。この2n角形のなす対角線
はどの3本も同一点で交わることはないとする(つまり一般の位置にとる)。赤
い点同士を結ぶ線分を赤い線と呼ぶ(当然{}_nC_2=n(n+1)/2本存在)。同様に
青い点同士を結ぶ線分を青い線と呼ぶ。赤い線と青い線の交わるところを真交
点と呼ぶ。
真交点の個数の最大値を求めよ。

予想は付くだろうが証明を書ければなかなか凄いと思う。俺はかなり苦労した
が・・・。ちなみに予想も付かないようだと、多分お呼びではない。難易度は東
大入試は目じゃない。俺は出たことないから知らんが数学オリンピックより難
しいのでは。
578132人目の素数さん:04/01/05 06:37
>>577
取り敢えず赤と青が1つずつ交互に並ぶときが最大と予想は出来る罠。
579132人目の素数さん:04/01/05 06:58
【千葉大学飛び入学試験予想問題 問題1】
(1)1,2,3,4,5…,nを並べ替えた時にすべてのi(i=1,2,3‥,n)についてi番目にiが
来ない確率をp(n)とおきます。p(n)とp(n-1),p(n-2)の間に漸化式を立てなさい。
(2)p(n)を求めなさい。
(3)nを無限大にしたときのp(n)の極限値を求め、あわせてこの数値の物理的意味を
考察しなさい。
         ,.-'''"-─ `ー,--─'''''''''''i-、,,
      ,.-,/        /::::::::::::::::::::::!,,  \
     (  ,'          i:::::::::::::::::::::;ノ ヽ-、,,/''ー'''"7
      `''|          |:::::::::::::::::::::}     ``ー''"
        !       '、:::::::::::::::::::i
        '、 `-=''''フ'ー''ヽ、::::::::::/ヽ、-─-、,,-'''ヽ                         
         \_/     ヽ--く   _,,,..--┴-、 ヽ
                     ``"      \>
581132人目の素数さん:04/01/05 11:46
>>576
連分数展開だけを考えると、比率は「a:1」a>1に統一して
黄金比「(√5+1)/2:1」、白銀比「√2:1」、赤銅比「R:1」のとき
(√5 +1)/2=1/(1-1/(1+1/(1-1/(1+…)…)=φ =1.61803…
(√8 +0)/2=2/(2-2/(2+2/(2-2/(2+…)…)=√2=1.41421…
(√13-1)/2=3/(3-3/(3+3/(3-3/(3+…)…)=R =1.30277…
全て4の数字を使用したものは黄金比に関連する数値になる。
>>581
正解です。(2)はやっぱり瞬殺だったか・・・
でも、別解もまだあるかも。
583979:04/01/05 14:38
>>577 神の問題難しい。2ちゃんねらー風情には無理ぽ。
584583:04/01/05 14:55
>>577
尤も予想は出来る(578と同じ)。最大性の証明が難しすぎる。
>>577
できた。
青い点と赤い点が交互にならぶときの真交点数をC(n)とおく。
C(n)が漸化式
C(2)=1、C(3)=6、
C(k+1)=C(k)+(1/2)(k^2+2k) (kが偶数のとき)
C(k+1)=C(k)+(1/2)(k+2k+3) (kが奇数のとき)
をみたすのは容易。
任意に赤い点をn個、青い点をn個一般の位置にとるとき真交点数がC(n)よりちいさい・・・(※)を
nに関する帰納法でしめす。
(I)n=2,3のとき 容易
(II)n≦k(k≧3)のとき成立すると仮定してn=k+1と仮定する。
いま任意に赤い点をn個、青い点をn個一般の位置にとる。真交点数をxとおく。
(i)kが偶数のとき
赤い線と青い線を交叉しないようにひとつずつえらぶ。
この赤い線で青い点がb1,b2個に分割されていてこの青い線で赤い点がr1,r2個に分割されているとする。
いまこの2本の線を除去すると真交点はr1r2+b1b2個減る。
帰納法の仮定よりx-(r1r2+b1b2)≦C(k)。いっぽうでb1+b2=r1+r2=k+1よりr1r2+b1b2≦(1/2)(k^2+2k)
∴x≦C(k)+(1/2)(k^2+2k)≦C(k+1)
(ii)kが奇数のとき
赤い線と青い線を交叉するようにひとつずつえらぶ。
この赤い線で青い点がb1,b2個に分割されていてこの青い線で赤い点がr1,r2個に分割されているとする。
いまこの2本の線を除去すると真交点はr1r2+b1b2+1個減る。
帰納法の仮定よりx-(r1r2+b1b2)-1≦C(k)。いっぽうでb1+b2=r1+r2=k+1よりr1r2+b1b2≦(1/2)(k+2k+1)
∴x≦C(k)+(1/2)(k+2k+3)≦C(k+1)
(i)(ii)よりいづれにせよ(※)はn=k+1のときにも成立。
∴(I)(II)より(※)はすべての自然数nで成立。
586585:04/01/05 20:08
しまった。間違ってる。吊ってきます。
>>577
n=587のとき
真交点の数は1107444766個であっとるけ?
正解なら解けたと思う
>>577
もしいいなら、誘導を入れさせてもらいたい
それに気付くかで、高校の定期試験レベルになるか
数学オリンピック超えになるかどうか決まると思う
589577:04/01/05 21:20
意外と解かれてしまっているようで、微妙に感動してます。
>>588
しかし流石にどんな誘導を入れても高校の定期試験にはならないと自負してい
るんだけど、どのような誘導を入れたら高校定期試験レベルになるとお考えな
んですか?
個人的には赤と青が交互になるときの真交点の数を求めよというので(1)、その
最大性で(2)とは出来ると思いますが、この場合、(2)は高校定期試験にはなら
ないと思う((1)は定期試験でもOK)。
>>587
nを使って表すとどうなりましたか?
590132人目の素数さん:04/01/05 22:03
>>590
{n^2・(n^2-1)}/12
あと、赤青が交互になるとき真交点の数が最高ってのは、
証明なしで使ってもいいよね?
591132人目の素数さん:04/01/05 22:33
自分にレスしとる
グスッ(´・ω・。`)ショボーン
592132人目の素数さん:04/01/05 23:08
〜〜〜誘導〜〜〜
円周上の任意の青い点を基点とする。
基点から時計まわりに青い点を1個2個3個・・・と数えていき、
m番目の青い点をBmとする。(基点は数えないでね)
このとき、基点とBmを結ぶ青い線上にある真交点の数をm・nを用いて表せ。
>>592
俺は基点をB1として時計回りにB2,…,Bnとした。
BkとB(k+1)の間にある違う色の点の数をx(k)とすると、

x1+x2+…x(n)=nが成り立ち、
真交点の個数は、Σ(l-k)(n-(l-k))x(k)x(l) (Σの範囲は1<=k<l<=n)
で表せる。

制約条件付きの最大値問題で、ラグランジュの未定乗数法でも使えばそれほど難しくない。
高校範囲だと少しむずいか。
>>593
解答おながいしまつ
ほぼこれで解答なんだけどね。

例えばn=4の場合だと、
B1とB2で分けられる円周上の2領域に、それぞれx1,(x2+x3+x4)の違う色の点がある。
よって、B1とB2を結ぶ点上にできる真交点は、x1*(x2+x3+x4)
同様に、
B1とB3…(x1+x2)*(x3*x4)
B1とB4…(x1+x2+x3)*x4

って感じで真交点の数が出る。これをすべて足したのがΣの式。今のn=4だと、

L=3*x1x2+4*x1x3+3*x1x4+3*x2x3+4*x2x4+3*x3x4

ってなる。ちょっと見にくいかな…。

ラグランジュの未定乗数法は、L-λ(x1+…x4)を作って、これをx1,…x4,λで偏微分してゼロとなる式をつくりゃいい。
x1=x2=x3=x4が出てくる。まぁ式の対称性からほぼ明らかなんだが…。

一般のnでも同様。式が煩雑だから書かないけど。

Σの計算は普通にx(k)=1とおいて和を計算すれば、>>590氏の解答が出る。
>>595
未定乗数法、L-λ(x1+…+x4-1)だな、失敬。
>>596 まだ間違えてる。L-λ(x1+…+x4-4)です…。ごめんなさい。
当然nなら、L-L-λ(x1+…+xn-n)です。
まだ必死
599132人目の素数さん:04/01/06 00:11
adjoint semisimple代数群$G$に対して境界が正規交差になる代数多様体
としてのコンパクト化$X$が(一般には無限に)存在する。この時$X$上の
両側$G _{sc}$同変なベクトル束の成す圏と表現と適当な条件を満たすその
フィルターの族の組の成す圏との圏同値を証明せよ。
なお、この結果は$G$がトーラスの場合のKlyachkoの定理の類似であるが、
群が非可換で左右の作用が同一視出来ないために新しい拘束条件が生じる。
定理の応用として$X$がwonderfulと呼ばれているときにはランクの低い
同変バンドルは直線束の直和になっていること等が従う。また、上の命題は
「対称空間上のベクトル束を与えたとき、その対称空間のコンパクト化上への
拡張を記述せよ」というKostantの問題の特別な場合の解決になっている。

600132人目の素数さん:04/01/06 00:17
あのーラグランジュだの未定乗数法だの
そんな難しいこと言わなくても解けると思うんだよ。
だってさ、漏れリアル工房だよ?
図を書いて基点とBmを結ぶ青い線上の
真交点の数をmn使って表してみなよ
高校生の知識の範囲で解けるからさ。
未定定数法でとけるんだからそれでいいじゃん。
コンパスと定規だけで正7角形を作図せよ
>>595
>>600
最大であることの証明がないから駄目。
>>603
じゃ追加しといて。
605577:04/01/06 00:58
 「神」とかいう恥ずかしい名前で投稿したのに、解かれていてびっくりです。
 脱帽です。誰が解いているのだろう…。知っている人だったりして…。
>>590 いや、違うんですよ。「赤青交互のときの個数を求める」が(1)、「赤青
交互のときが最大であることを示す」が(2)と言った訳なんです。なので(1)だ
けでも進歩ではあるんですが、それだけでは不十分です。難易度は(1)<<(2)で
す。(1)は600さんのおっしゃるとおり、高校初年度級で解けます。定期試験と
してはかなり良問と思います。(2)の最大性はかなり難しいです。見た目自明
なのですが、交互のときが最大になることは帰納法を使ってもまだ難しい。
>>593,595 私の解答は最大性の証明でB4二枚になってしまったので、これほ
どコンパクトな解答があり、びっくりです。しかしラグランジュが出てくると
は…。
>>599 これは知りませんでした。2ちゃんねらーの底力に脱帽です。でも難
しい問題ですよね(これを難しくないとか言われたら辛いナァ…)。

どうでもいいですが、579にレス。これはただの攪乱順列の問題ですね。自明。
606132人目の素数さん:04/01/06 01:19
とりあえず漏れの答えを書いとくね
基点とBmを結ぶ青い線上にある真交点の数はm(n-m)と表される。
このときmは1からn-1である。
∴基点とBmを結ぶ青い線上にある真交点の数の和をSとおくと、
S=Σ[m=1,n-1]m・(n-m)となる。
また、基点になりうる点の数はn個であり、このとき全ての真交点を二回ずつ数えているので、
Sをn/2倍してやれば、題の求める真交点の数である。
∴S・(n/2)
=(n/2)・Σ[m=1,n-1]m・(n-m)
={n^2・(n^2-1)}/12
だめかえ?
607577:04/01/06 01:24
取り敢えず(1)「赤青交互のときの個数を求める」は590さんのであってます。こ
れは複数の方(584,587,600)がおっしゃってる通りで、さほど難しくはありま
せん。
590さんの解が最大である証明(つまり(2)の部分)は、まだ与えられてないよう
ですね(595さんの主張が正しければこの方針で解決しているのですが)。やはり
難しいのかな。挑戦キボン。
何度も言うとおり(1)<<(2)なんで(2)の解答(しかも簡潔なら尚良し。私は労働
の結晶というしかないような冗長な解答しか持ってないので)を期待してます。
まぁ599さんはどうやら(2)も解いたも同然なんでしょうが。
私としては595さんの言う方針で今よりも簡潔な解答が得られるかどうかを確
認してみます。煩雑な多項式がでてくるのでちょっと大変そうですが。
608577:04/01/06 01:27
>>606
(1)「赤青交互のときの個数を求める」の部分の解答としては、恐らく最
も簡潔な解答と思います。
ただ何度も言っている通り(1)の時が真交点が最大になるということを
証明しないといけないので、そこが抜けています(赤と青が交互に来る
ときが真交点が最大になるということは全然自明でないですよね)。
609577:04/01/06 01:30
というわけで605撤回(よく見るとまだ最大性ははっきりとは示されてい
ない。595さんは方針は示されてますが)。(1)は解かれてしまっている
ので、(2)の部分『真交点の最大値が{n^2・(n^2-1)}/12である』ことの
証明を希望します(簡潔歓迎)。
610132人目の素数さん:04/01/06 01:55
そうか…、よく考えると最大性の証明は難しいヨカーン。
(1)は簡単ね。厨房でも出来るか。
611132人目の素数さん:04/01/06 02:12
基点とBmを結ぶ青い線をBLmとする。
赤青交互に並べるときBL1上にはn-1個の真交点がある。
しかし、B1を反時計回りにR1よりも基点へ近付けるとき
n-1個の真交点が消えてしまう。
∴赤青交互に並ぶとき真交点は最大となる。
漏れは勘違い君ですか?
612132人目の素数さん:04/01/06 02:13
クロロニトロベンゼンのo,m,pのもっとも安定な構造をとっている時その原子間距離を全て示せ
613577:04/01/06 02:24
>>611 勘違いです。
・まず、B1を移動させるのならば、B1とBm(M=2,3,…,n)を結ぶ青い線
全てについて、それらと交わる真交点を考えなければなりません。そ
れらの真交点の合計値の増減を見る必要がありますよね?
・またこの方法が仮に正しいとしても、赤青交互から一つだけずらし
たときのことしか考慮していないので、赤青交互から激しくずれたと
きに最大値を取る可能性を否定できてません。
614610:04/01/06 02:28
ラグランジュの乗数法の解答を誰か完成させてくれることキボン。
>>609
できたかも。
Aij=|i-j|(n-|i-j|)でさだめられる行列をとってきて問題は束縛条件肺i=n、xi≧0、(この集合をΔとする。)
のもとで2次形式Q=尿ijxixjの最大値がxi=1のときであることをしめせばよい。以下ベクトルはすべて
列ベクトルで考える。
Bi=i(n-i)とおく。Aは巡回行列であり、かつ対称行列である行列。よってAの固有値は1のべき根ζをとって
B0ζ^0+B1ζ^2+・・・+B(n-1)ζ^(n-1)の形の元全体。よって1以外の固有値はすべて負。
固有値を(e1=1,e2,e3,・・・,en)とする。Aは直交行列UをとってC=UAU^(-1)=diag(e1,e2,・・・,en)と
対角化される。Uによって(1,0,・・・,0)にうつされるベクトルは(1/(√n),1/(√n),・・・,1/(√n))であり
よってUによってΔは(√n,*,*,・・・,*)の形のベクトルにうつされる。(y1,・・・yn)が(√n,*,*,・・・,*)なる形のベクトル
全体を走るときe2,・・・,en<0により(y1,・・・yn)=(√n,0,0,・・・,0)のときが最大である。そしてこのベクトルは
(x1,・・・,xn)=(1,1,・・・,1)の像でありΔの元ゆえこのときがたしかに最大である。
616577:04/01/06 02:37
>>615
凄い……。今日はこれから実験があるので、実験が終わったら精読させ
ていただきます。何か一読したところ良さそうな予感がしたんですが。

しかし行列論に持ち込むなんて発想は全く私にはありませんでした…。
617615:04/01/06 02:40
修正でつ
B0ζ^0+B1ζ^2+・・・+B(n-1)ζ^(n-1)の形の元全体。よってB0+・・・+B(n-1)以外の固有値はすべて負。
でつ。
618577:04/01/06 04:20
>>615,617 多分OKと思います。高校生にはやや難しいでしょうが、証明とし
ての純度は高いですね。有難うございました。

 一応>>1さんのご希望には沿えるくらいの難易度ではあったと自負してます。
これ(勿論(2)を含んで)が簡単だと言う香具師は多分尋常じゃなく優秀です。
619132人目の素数さん:04/01/06 06:24
神の問題難しかった…。俺には(1)が関の山。
620研究する数学者:04/01/06 09:15
なんか神とかいう香具師の問題、難しいな。
漏れも問題を出しておきます。残念ながら神とかいう香具師のよりはずっと簡
単(特に(1)は簡単)。ただ、客観的に見れば簡単な問題ではないが。

(1)[0,1]区間の一様な乱数がある。この乱数をn個発生させたときの最小値を
X_nとおく。E(X_n)を求めよ。
(2)f(x)=Σ_[k=0→n](a_k)x^kとする。nを固定したとき
∀x∈[-1,1];|f(x)|≦1⇒Σ_[k=0→n]|a_k|≦s_n
を常に満たす最大のs_nを求めよ。
621研究する数学者:04/01/06 09:17
上の(2)の問題文間違えた。
(2)f(x)=Σ_[k=0→n](a_k)x^kとする。nを固定したとき
Σ_[k=0→n]|a_k|≦s_n⇒∀x∈[-1,1];|f(x)|≦1
を常に満たす最大のs_nを求めよ。
>>620-621
↓これで桶?
(1) 1/(n+1)
(2) s_n=1
623研究する数学者:04/01/07 20:09
>>622 阿寒。
(1)は正解。結構面白い問題と思ったけどどう?確率密度関数がきちんと分か
らないと解けないと思うよ。
(2)は阿寒。s_n=1としてしまうとn≧2のときにはa_0=a_1=…=a_n=1のとき
|f(1)|=n>1で矛盾。駄目っすよ。

でもまぁ(1)が解けているのでいいかな。ありがとうございます。
624在野の数学者:04/01/07 20:16
こんな問題銅?(1)から難しいで。
(1)R^n上に点A_1,…,A_n,B_1,…,B_mを取る。nが自然数のときは
Σ_{1≦i<j≦n}d(A_i,A_j)+Σ_{1≦i<j≦m}d(B_i,B_j)≦Σ_{i,j}d(A_i,B_j)
を示せ。ただしd(X,Y)でX,Yの距離とする。
(2)上の性質を満たす距離空間はどのようなものか?(今のところR^nと
任意のグラフについては成り立つことを確認済み)
>>623
>(2)は阿寒。s_n=1としてしまうとn≧2のときにはa_0=a_1=…=a_n=1のとき
>|f(1)|=n>1で矛盾。駄目っすよ。
 
なにこれ?a_0=a_1=…=a_n=1ってどれを0にしてどれを1にするの?
626研究する数学者:04/01/07 20:28
>>625
う、済まないっす。(2)は>>625さんので合ってますね。というか出題ミ
スで全然難問じゃなくなってしまいました。大変失礼しました。
>>624
(2)は問題になってないのでは?この手の「必要十分条件を求めよ」系の出題は
受験数学のように求められてる条件の形に暗黙の了解、たとえば「〜をみたす
ためのxの条件を求めよ」なら「1≦x or -3<x<2」のような形でこたえないといけない
みたいな暗黙の了解がないと問題として成立しないと思う。こういう掲示板で出題するなら
「〜であることが必要十分であることをしめせ。」みたいに条件を特定してしまわないと
問題にならない。
628研究する数学者:04/01/07 20:45
>>627 はい、よく考えると問題になってなかったっす。勘違いしてま
した。反省。
629132人目の素数さん:04/01/07 20:45
>>624 n=1ならば直線上だし、何とかなりそうな予感。銅?
>>624
これの(1)って成立するの?
n=1でA1=0、Bi=i (1≦i≦m)とかにすると
左辺≧播(Bi,Bj)≧d(B1,B2)+・・・+d(B1,B(m-1))+d(B1,Bm)+・・・+d(B(m-1)1,Bm)=(m-1)(m-2)
右辺=播(AiBj)=d(0,1)+d(0,2)+・・・+d(0.m)=(1/2)m(m+1)
で左辺≦右辺なんて一般には成立しないんでは?
631在野の数学者:04/01/07 22:51
>>630 すまない。問題の訂正をします。

(1)R^k上に点A_1,…,A_n,B_1,…,B_n,B_{n+1}を取る。kが自然数のとき
は Σ_{1≦i<j≦n}d(A_i,A_j)+Σ_{1≦i<j≦n+1}d(B_i,B_j)≦Σ_{i,j}d(A_i,B_j)
となることを示せ。ただしd(X,Y)でX,Yの距離とする。
(2)上の性質を満たす距離空間はどのようなものか?(今のところR^kと
任意のグラフ(点と線です)については成り立つことを確認済み)
632132人目の素数さん:04/01/07 22:53
↑k=2,n=2とすると有名な五角形の性質になりますね。
633132人目の素数さん:04/01/08 02:50
>>631
n=2のときは結局三角不等式に帰着できる感じ。まだ証明は完成させてないけど。
634633:04/01/08 02:54
n=2じゃなくてk=2のときだった。このときは平面だからね。
635132人目の素数さん:04/01/08 12:03
>>631 k=3のときは想像もつかない。劇難問かも。
 この性質は一般の距離空間では成立しないの?反例でもあるのかな?
a+b=3
b+c=5
d=?
637132人目の素数さん:04/01/08 16:32
???
638132人目の素数さん:04/01/08 23:46
>>631
ヒントおながいします。age
とあるサイトにあった問題です

2をx乗するとこのような数字になります
123454321...........................................
つまり最初の9ケタの数字の並びが「123454321」である
その後にはどんな数字が来てもいい、何桁の数字になってもいい
この条件を満たす最小のxを求めよ
640在野の数学者:04/01/09 01:13
>>638
(1)一次元のときを証明せよ。そして射影。
(2)確か反例となる距離空間があったような…(忘却の彼方)
641132人目の素数さん:04/01/09 01:34
平面上にn個(n>2)の点がある。
この中から任意に選んだ2点を通る直線上には、必ずその2点以外の点があるとする。

このときこのn個の点は一直線上にあることを示せ。
>>641
難解、ってよりか面白い問題だね、どっちかっつうと。
643132人目の素数さん:04/01/09 01:45
>>639
最小のxは存在しない
644研究する数学者:04/01/09 01:50
>>641 敢えて釣られて証明してみよう。
 直線ABと点Cとの距離をd(AB,C)と書く。
 いま、n個の点が一直線上に存在しないのならば必ず2点だけしか通らない直
線が存在することを示せばよい。

 まずn個の点が一直線上に存在しないと仮定すればd(AB,C)を正の値にする点
A,B,Cの組み合わせが存在する。それらの中でd(AB,C)を最小とするような点の
組み合わせをA',B',C'とおく。
 この時、直線A'B'と点C'の距離は正の値であり、これより小さい正の距離を
持つような組み合わせは存在しない。
 もし直線A'B'上に別の点D'も存在したらどうなるだろう?このときは自明に
d(A'B',C')を下回る距離が存在してしまう。すると矛盾。よってこのような直
線A'B'上には他の点が存在しない。

 これで示したことになりますよね。あってますか?>>641
645132人目の素数さん:04/01/09 01:50
>>639
何故
>>641
当たり前の事だけど、どう説明してよいやら・・・
>>643
あ、xは整数ね
648132人目の素数さん:04/01/09 01:56
>>644
それはネタ?それとも素?
649132人目の素数さん:04/01/09 01:56
>>646
非ユークリッドでは¬自明
あってる。
>>640
なるほど。(1)できた。
k=1のとき
P={A1・・・,An,B1・・・B(n+1)}とおく。各PuにたいしてPu=AiとなるAiの数をau、Pu=BjとなるBjの数をbuとおく。
このとき与式の左辺-右辺は
納u>v](Pu-Pv)(aubv+avbu-auav-bubv)
となる。(ただしP={P1<P2<・・・}とおいた。)これが正になることをしめせばよい。
つまりR^1の部分集合P={P1<P2<・・・}とそのうえの非負整数値関数au,buが
蚤u=n、巴u=n+1をみたすときS=納u>v](Pu-Pv)(aubv+avbu-auav-bubv)が0以上であることをしめせばよい。
そうでないものが存在したと仮定する。そのような(P,au,bu)のうちPの元数が最小のものをとる。
もし3元以上あるとするとSはP2について線形であるためP2の位置をP1かP3にずらしても値が増えないようにできる。
つまり一個Pの元数をへらせる。これはPの元数の最小性に反する。よってPの元数は2以下である。
Pの元数が1のときはS=0ゆえ矛盾。Pの元数が2とするとき
S=(a1b1+a2b2-a1a2-b1b2)=(a1+b1)^2-(2n+1)(a1+b1)+n(n+1)
ここでa1+b1は整数なのでSはa1+b1=n、またはn+1のときに最小であるがそのときS=0であるゆえ矛盾。
∴k=1のときは成立。
一般のとき
各θ∈P^(k-1)の元にたいし向きがθである直線L(θ)をとって各C∈R^kのそれへAi,Bkの正射影をP(θ)とする。
k=1の場合はすでにしめしているので
Σ_{1≦i<j≦n}d(A_i(θ),A_j(θ))+Σ_{1≦i<j≦n+1}d(B_i(θ),B_j(θ))≦Σ_{i,j}d(A_i(θ),B_j(θ))・・・(※)
が成立する。P^(k-1)上の関数としてd(P(θ),Q(θ))は連続関数であり
∫[θ∈P^(k-1)]d(P(θ),Q(θ))=V(P^(k-1))d(P,Q) (V(P^(k-1))はP^(k-1)の体積)
であるから(※)をP^(k-1)上積分して与式を得る。
652132人目の素数さん:04/01/09 02:09
>>644
>もし直線A'B'上に別の点D'も存在したらどうなるだろう?このときは自明に
>d(A'B',C')を下回る距離が存在してしまう。

なんで?
>>641
はシルベスターガライの定理の香具師だな。数オリ系の本にのってた。
垂線を下して直線を分
けて二点含む方で遠い
点と直線を結ぶと近い
点との距離は小さい。
655在野の数学者:04/01/09 03:13
>>651
凄い。そう、その通りです。まぁグラフ上で(1)が成り立つのは多分あなたには
すぐ分かることです。
しかしこれが成り立つという距離空間はどういうものなんだろう?
656132人目の素数さん:04/01/09 06:43
>>639
2^70279608 = 1.234543218... × 10^21156270
657132人目の素数さん:04/01/09 06:49
凄い…。3問ほぼ同時に解決か?
>>639のxの最小性と
>>631の(2)
がまだ未解決ですね。656さんも(自分では証明したのかもしれないが)
xの最小性については言及していない。
658656:04/01/09 11:07
>>639 >>657
70279608 が題意を満たす最小の正整数であることの証明

t を実数として、f(t) を「t に最も近い整数と t との差の絶対値」とする。
a_n を log2(=log_10(2))の連分数展開を n項(?)で打ち切ってできる規約分数の分母とする。
(a_1=3, a_2=10, a_3=93,..., a_16=51132157, a_17=146964308)
題意を満たす x について、f(x log2 - log123454321.5) < 2×10^(-9) が成り立つ。
ゆえに、題意を満たす整数 x, y (0 < x < y) があったとすると、f((y - x) log2) < 4×10^(-9)
連分数の性質から、a_17 は f(a_17 log2) < f(a_16 log2) となる最小の正整数であることが言える。
f(a_16 log2) > 4×10^(-9) なので、f((y - x) log2) < 4×10^(-9) となるためには、y - x ≧ a_17 でなければならない。
つまり、題意を満たす正整数で、x < a_17 = 146964308 となる x は高々1個しか存在しない。
x = 70279608 がその1個。
AB=AC=AD=AE=BC=BD=BE=1。
CD=CE=DE=2。
AC+AD+AE+BC+BD+BE<AB+CD+CE+DE。
660657:04/01/09 12:36
>>659凄い。距離の公理を満たしているから距離空間だが、しっかりと反例に
なっている。
やはり>>631 >>640が言及しているように(2)には反例があったのか。
あと>>658さんがおっしゃるようにxの最小性もOKみたいですね。
3問とも解決!
661132人目の素数さん:04/01/09 14:00
>>660



        だ        れ        き        み        ?
662132人目の素数さん:04/01/09 14:13
さwかwげwのwうwたwがwwwwww
きwこwえwてwくwるwよwwwwww

wwっうぇwwっうぇwwっうぇwwっうぇww

うはwうはwおkwおkw
wwっうぇwwっうぇwwっうぇ
663132人目の素数さん:04/01/09 14:17
>>661
 あなたも誰?
 ところで放物線y=x^2の(0,0)から(a,a^2)迄の弧の長さって意外と
求めるのが難しい。積分計算、高校時代に結構頑張って見つけまし
た。なのでこのスレで出題します。
664132人目の素数さん:04/01/09 14:28
>>663
ただの線積分じゃん(ちょっと計算めんどいけど,)。
ちゃんと授業出てたか?
665663:04/01/09 15:11
>>664 まぁただの線積分だけど、それ求めるのちょっと大変よ。
というか∫√(1+x^2)dxが分かればいいんだけど、この積分、自
明じゃないでしょ?
666自明:04/01/09 15:17
自明

とりあえず此のスレに出題すんなー帰れ
667665:04/01/09 16:12
>>665 そうか、このレベルの問題では駄目か。すまない。
(1)多項式f(x)は常にf(x)f(x+1)=f(x^2+x+1)が成り立つ。このような多項式
をすべて求めよ。
(2)2つ以上の都市A_1,A_2,…,A_nがある。それぞれの都市はお互いにリンク
しているかしていないかのいずれかである。これらを2つの国に次の条件を
満たすように分割したい。
(条件)どの都市も自分の属している国の中でリンクしている都市が丁度偶数
個ある。
このような2国への分割が可能であることを示せ。ただし一方の国が空であっ
ても良い。
これでも簡単すぎるというのなら帰ります。
あるスポーツドリンクはあきびんを5本もっていくと、新しい1本と交換できる
365本飲むには?本買えばよい (灘中)
>>667
ほう。(2)はやや手応えあるか。

>>668
オヤスミマン☆
一筆書きの問題ってよく出るよな
x^3 - y^2 = 2

これを満たす整数x,yは(x,y)=(3,5)と(3,-5)の2種類のみである
この事を証明せよ。もしくは反例を提出せよ。
672132人目の素数さん:04/01/09 18:33
某大学院生試験問題 (解ける人はといてみてください)

F(x,t)=(1/(√2πt))exp((-x^2)/2t)とし、g(x)をR上であるような
1変数連続函数とする。このとき、以下に答えよ。

1.t>0に対して、積分:∫[-∞,∞]g(y)F(x-y,t)dtは収束する。
  そこで、f(x,t)=∫[-∞,∞]g(y)F(x-y,t)dtとする。
  この時、f(x,t)は偏微分方程式∂f/∂t=((∂^2)x)/(2∂(x^2))
を満たすことを示せ。但し、微分と積分の順序交換は可能である。

2.∫[-∞,∞]F(x,t)=1を示せ。

3.<難問>
  各点xについて、lim_[x→∞]f(x,t)=g(x)を示せ
いくらなんでも誤植が多すぎるだろ
674132人目の素数さん:04/01/09 21:05
1から9までの数字を全部使って数を表すことを
「小町算」といいます。

(例) 1×2×34×5×6-7×8×9=1975
987+6×54×3+21=1980

このように1から9までの数を「増加の順」か「減少の順」に
並べて適当に記号(今回は+、−、×、÷の4種類)を入れ、
年数を表すのです。

では問題です。
今年、2004年を小町算で表してみてください。

>>674
その前に単発スレの削除依頼を。
>>671
楕円曲線系はいらね
自然数全体の集合N={1, 2, 3, ...}の分割A, B(つまりN=A∪B, A∩B=φとなるもの)を

すべてのn≧3に対して#{(x, y)|x+y=n, x≦y, x, y∈A}=#{(x, y)|x+y=n, x≦y, x, y∈B}

という性質を満たすように取れることを示せ。
スレ違いっぽいんですけど、
何処に書けばいいのかどうも分からないのでとりあえずここに書きます。

ちょっと質問なんですが、数の呼び名っていろいろありますよね?
一、十、百、千、……、無量大数までは調べて分かってんですけど、
その上って何なんですか?
調べても分かりませんでした。

誰か分かる方、教えてくださいませ。
無量大数→モナー→ギコ→イイ→・・ だよ。
さよなら
680132人目の素数さん:04/01/10 09:10
 671ではない者だが、ここの住人は>>671も解けないのかと思うと溜息が
出た。所詮2ちゃんねらーやな。
 677はそれよりはちと難しいかな。667は(1)は簡単だが(2)は難しい。
 今のところ677以外は解いた。
681132人目の素数さん:04/01/10 09:19
>678
10^21 zetta
10^24 yotta 酔った,寄った
10^27 xetta
10^30 watta 割った
10^33 vatta 飛蝗
10^36 utta 打った
10^39 tatta 立った
10^42 satta 去った
10^45 ratta
10^48 quitta 来た
10^51 patta
10^54 otta 追った,居った
10^57 notta 乗った, 乗った
10^60 motta 持った
10^63 lotta
10^66 kotta 凝った
10^69 jotta
10^72 itta 居た,鋳た,射た
10^75 hotta 掘った
10^78 gotta ごった返す
10^81 fotta
10^84 etta 得た
10^87 detta 出た
10^90 catta 買った
10^93 batta 飛蝗
10^96 atta 有った

ぢゃないか?
682132人目の素数さん:04/01/10 17:57
解くのに半年かかった問題がある
面倒だからここには書かないけど
>>680
楕円曲線の整点もとめる問題はいらん。この問題あちこちのスレにはりまくるのやめてくれ。
>>680
>>21の答えかいてくれ。ちなみにこれはちゃんと解ける問題。
>>680
>>671の答えおながいします。
>>683
俺(671)はここにしか書いてないよ
マルチはしてない
この問題、難問スレにはふさわしくないかなとか思ってしまってはいるけど
>>680
>>677は大学入試レベルの問題かと
>>687
答えおながいします
>>688
20くらいまでの整数を A,B に振り分けていってみたら、パターンがわかるよ
>>689
もう自分で答えだすの諦めもーした。答えかいてくさい。
y^2=x^3-2のほうの証明

α=(-2)^(1/2)とおく。

x^3=y^2+2=(y+α)(y-α)

Q(α)の類数は1なので

y+α=β^3, y+α=αβ^3, y+α=α^2 β^3=-2 β^3(β∈Z[α])

のいずれかが成り立つ。β=a+bαとおく。

第一の場合、y+α=(a^3-6ab^2)+(3a^2 b-2b^3)α, 3a^2 b-2b^3=b(3a^2-2b^2)=1より
b=1, a=±1。a^3-6ab^2=±5よりx=3, y=±5。

第二の場合、y-α=-α(a-bα)^3より、x^3=α^2(a^2+2b^2)^3=-2(a^2+2b^2)^3となり、
(-2)^(1/3)が有理数であることになるが、これは矛盾。

第三の場合も同様に4^(1/3)が有理数であることになり、矛盾。

よって、y^2=x^3-2の整数解は(3, ±5)のみ。
>>690
A = {x | x∈N, x ≡ 0,1,3 (mod 6)}、B = N - A。
n≦8 で、A,B が問題の条件を満たしているのはすぐわかる。
A,B が n で問題の条件を満たしていれば、n+6 でも満たしていることを証明しる。
>>691-692
すばらすぃ・・・
>>680
>>672の問題を解答してほしいです。
695692:04/01/10 21:34
>>690
n≦11 まで調べてから帰納法使ったほうがいいや。スレ汚しスマソ。
>>692
14=1+13=7+7
14=4+10

ってか、それで構成したら一般に12m+2のとき1個ずれない?
697692:04/01/10 22:07
>>696
逝ってきます…
698132人目の素数さん:04/01/10 22:16
age
687 名前:132人目の素数さん 投稿日:04/01/10 19:41
>>680
>>677は大学入試レベルの問題かと



697 名前:692 投稿日:04/01/10 22:07
>>696
逝ってきます…
700692:04/01/11 03:19
定理)男が k人、女が k人の 2k人から、k組のペアを作ると、男同士のペアの数と女同士のペアの数は等しい。

>>677の証明

A = {x| x = 1 または、x>1 で x-1 の2進表記に 0 が奇数個ある}、
B = N - A。

n=2k+1 のとき、(1,2k), (2,2k-1), (3,2k-2),..., (k,k+1) というk組のペアの中の、A の元同志のペアの数と、B の元同志のペアの数が等しければ、問題の条件が成り立っている。
A,B の構成から、{1,2,..,2k} の中の A の元の数と B の元の数は等しいので、あとは上の定理を使えばよい。

n=2k のときは、(1,2k-1), (2,2k-2),..., (k,k) という k個のペアを作ることになる。
つまり、{1,...,k}と{k,...,2k-1}の2k個の数の中から k個のペアを作る(k は2度数える)。
{1,2,...,2k-2} の中の A の元の数と B の元の数は等しい。
k と 2k-1 は、片方が A の元なら、もう片方は B の元。これも A,B の構成から明らか。
結局 {1,...,k}と{k,...,2k-1} の 2k個の数の中の A の元の数と B の元の数は等しい。
>>700
おお、正解くさいけど
 
>k と 2k-1 は、片方が A の元なら、もう片方は B の元。
 
これはホント?
>>700
そのままだと16、31∈Bとなってkと2k-1がことなるグループにはいらないからダメだけど
A={k|k-1の2進展開の0の数が奇数}
B={k|k-1の2進展開の0の数が偶数}
ただし1∈Aとしとけばそのままの証明でよさそうだ。解決だな。
>>702
30=11110(2)
31∈A
704702:04/01/11 05:01
>>703
ちがうよ。31∈Bといってるのは>>700の定義の場合。
>>700の定義だと16=10000(2)なので16∈B、31=11111(2)なので31∈Bとなって
求める条件をみたさない。
オレの定義だと16-1=15=1111(2)なので16∈B、31-1=11110(2)なので31∈Aとなって
たしかにkと2k-1はちがうグループにはいる。
よくかんがえると>>700の定義のままだと
 
>A,B の構成から、{1,2,..,2k} の中の A の元の数と B の元の数は等しいので、あとは上の定理を使えばよい。
 
これもいえてないけどオレの定義だと桶。
↑まあ冷たい水で顔洗ってこいよ
>>705
???あらってきた。なんか間違ってる?
>>705
すまん。わかった。吊ってくる。
708677:04/01/11 12:54
>>702
すげえ。すぐ解けるとは思わなかった。カンニングじゃなきゃあんたプロになれるわ。

問題の元ネタ→ http://journals.impan.gov.pl/aa/Inf/110-3-7.html
(こっちの証明はやはり初等的だが少し難しい)
>>708
皮肉ですか。
710132人目の素数さん:04/01/11 13:50
>>667を誰か解いて。
>>709
俺もオモタw
712132人目の素数さん:04/01/11 15:48
>>667
ヒントおながいします
713667:04/01/11 16:18
>>712
(1)多項式f(x)は常にf(x)f(x+1)=f(x^2+x+1)が成り立つ。このような多項式
をすべて求めよ。
(2)2つ以上の都市A_1,A_2,…,A_nがある。それぞれの都市はお互いにリンク
しているかしていないかのいずれかである。これらを2つの国に次の条件を
満たすように分割したい。
(条件)どの都市も自分の属している国の中でリンクしている都市が丁度偶数
個ある。
このような2国への分割が可能であることを示せ。ただし一方の国が空であっ
ても良い。
(1)hint:xにx-1を代入せよ。
(2)hint:帰納法
>>713
(2)のヒントをもすこしおながいします。
715132人目の素数さん:04/01/12 02:35
>667
f(x)=(x^2+1)^n (n≧0), または f(x)≡0.
716667:04/01/12 02:36
>>715 (1)はOKです。
 (2)のhintは眠いんで寝た後に書き込みます。
717132人目の素数さん:04/01/12 02:54
f(x)=x^2+∫[e,1](f(t)/2t)dtを満たすf(x)は?
718132人目の素数さん:04/01/12 03:43
難しすぎてわからないか
719132人目の素数さん:04/01/12 05:02
>>717
学校の宿題を出してんじゃねーよ
実数集合A={a_i|1≦i≦n}において
Σ[1≦i≦n]a_i=p、Σ[1≦i≦n](a_i)2=q(p,q定数)が成り立っている。
Σ[1≦i≦n](ai)3のとり得る値の範囲を求めよ。
また、最小値、最大値をとるときの集合A(a_i≦a_(i+1),1≦i≦n-1)を求めよ。
ただし、iは自然数、nは3以上の自然数とする。

恐ろしくではないが、難問と思われ
>>667
できた。(1)は答えだけはでてるけど案外やってみるとむずかった。
オレはf(x)=0の解をa1,・・・,anとするときf(x)f(x+1)=f(x^2+x+1)の対数微分とって
(A)nは偶数でありaiをならべかえてa(2k-1)=-a(2k) (k=1,・・・,n/2)とできる。とくに
(ai)^k=0 (k:奇数)・・・(※)
(B)各aiにたいし方程式x^2+x+1=aiの解をbi、ciとするとき
bi,ciはf(x)=0、f(x+1)=0の解をならべかえたもの全体となる。とくに
(((ai)^2+ai+1)^k+((ai-1)^2+(ai-1)+1)^k)=2(ai)^k (k:非負整数)・・・(※※)
(※)、(※※)から>>715がでた。
(2)グラフG=(V,E)についてV=(v1,v2,・・・,vn)とならべる。各点viを端点とする辺の数をmiとする。
行列Rij=Rij(G)∈M(n,Z/2Z)を
Rij=mi (i=j)、0 (i≠j,{vi,vj}は辺でない)、1 (i≠j,{vi,vj}は辺)
でさだめる。方程式
納j]RijXj≡mi (mod 2) (∀i)
が整数解をもつことをしめせばよい。それには(ci)をZ/2Z係数のベクトルとするとき
納i]ciRij≡0 (mod 2) (∀j)⇒納i]cimi≡0 (mod 2)
をしめせばよいがそれは次の補題によりしめされる。
−補題−
グラフG=(V,E)、V={v1・・・vn}に対してG'=(V',E')を以下でさだめる。
V'={v1,・・・,v(n-2),w}、
(vi,vj)がG'の辺⇔(vi,vj)がGの辺、
(vi,w)がG'の辺⇔((vi,v(n-1))がGの辺&(vi,vn)がGの辺)or((vi,v(n-1))がGの辺&(vi,vn)がGの辺)
このときR(G')=ER(G)Fである。ただしE、Fは次でさだめられる行列
Eij=1 if (i=j or i=n,j=n-1) 0 otherwise
Fij=1 if (i=j or i=n-1,j=n) 0 otherwise
これから(ci)の0でない項の数に関する「帰納法」で示せる。
722667:04/01/12 21:22
>>721 すごい。特に(2)(まあ(1)はできるでしょうけど)。
 結構難しかったと思います。楽しんでもらえましたらうれしい。
723132人目の素数さん:04/01/12 21:35
(恐ろしく難解、とまではいきませんが)
総当たり戦の問題。
2nチームが総当たりリーグ戦を行う。
各チーム(2n−1)回の試合を行うことになるのだが、
(2n−1)回の組み合わせにおいて、
 @どのチームも毎回どこかと対戦
 A重複なき対戦
を満たすような組み合わせが存在することを示せ。

(例)n=3の場合
チームをA,B,C,D,E,Fとすると、
1回目A-B、C-D、E-F
2回目A-C、B-E、D-F
3回目A-D、B-F、C-E
4回目A-E、B-D、C-F
5回目A-F、B-C、D-E
となる。
724132人目の素数さん:04/01/12 21:41
重複なき対戦




・・・なんかかっこいい
725132人目の素数さん:04/01/13 16:36
整数係数多項式f(x)はn∈N⇒f(n)>nを満たす。a_1=1,a_i=f(a_{i-1})で定義
される集合S={a_i|1≦i}を考える。
いま、∀k∈N ∃T⊂S s.t.#T=∞ and ∀m∈T;m/k∈Nが満たされている。
このときf(x)=x+1しかありえないことを示せ。
726132人目の素数さん:04/01/13 17:46
>>725 なんか、∀k∈N ∃T⊂S s.t.#T=∞ and ∀m∈T;m/k∈Nとか
敢えて難しく書いてるとしか思えないけど、それを差し引いても難
しいね。
727132人目の素数さん:04/01/13 21:22
>>725 やばぽ・・・さっぱりわからない。ま、f(x)=x+1しかなさそうな気
はするんだが。2ちゃんねらー風情には無理でないの?
>>723
正2n-1角形P1P2・・・P(2n-1)をかいて重心をOとする。
第k日目は直線OPkに関して対称な2チームとk-nを対戦させればよい。
729728:04/01/13 22:01
訂正。直線OPkに関して対称な2チームとk-2nを対戦させればよいだ。
>>726-727
直感的には明らかっつーか
∀k∈N ∃T⊂S s.t.(T⊂kN , #T=∞)ってことだから
十分大きな範囲ではだいたいS=Nで、そうするとa_iはiの一次式。
そこでa_i=i+c (c≧0)とかするとfも一次式なんでf(x)=x+d⇒d=1ワーイメチャクチャいい加減だーい

って・・
731132人目の素数さん:04/01/13 22:36
>>729
確かに。そのとおりですね。
なんか長い間悩んで損した気分。
732132人目の素数さん:04/01/14 00:11
>>730
 いや、流石にこりゃ駄目だろう。
 「十分大きな範囲ではだいたいS=Nで」が激しく違うし。
>>732
いやS=Nが到達点だろ。
だってf(x)=x+1ならa(i)=a(i-1)+1∴a(i)=i∴S=Nなんだから。

各kに対してTはkNから高々有限個の元を除いたもの(∵T⊂kNかつ#T=∞)
ということまでは当然成立する。そのTとSがそもそもT⊂Sという関係だから
「だいたいS=N」という感じ。実際にはS=NでT=kNとなってるわけだ。
>>725
できた。まず任意のs∈Sに対し(f(s)-s)|s。なぜならそうでないs∈Sがとれたとする。
s=a_iなるiをとってf(s)-s=Mとおく。j≧iにたいしてa_j≡a_i (mod M)であるが仮定から
a_i=s≠0 (mod M)。よってi項目以降にMの倍数は存在しない。これは仮定に反する。
よって特にf(x)は1次式であり最高次の係数は1。f(x)=x+cであるが仮定よりc>0。
c≠1であればa_i≡1 (mod c)により仮定に反する。ゆえにf(x)=x+1。
735721:04/01/14 01:02
>>721>>667の(2)の解答だけどよくよくかんがえたらあんなわけわからん補題を
使う必要なかった。Z/2Zの線形方程式納j]Rij・xj=Riiが解をもつことを示す問題に
還元するまではいいとしてRijは対称行列であることをもちいて任意の
納i]ci・Rij=0 (∀j)であるベクトル(ci)にたいし
0=納i,j]ci・Rij・cj=納i]Rii・ci・ci+2納i<j]Rij・ci・cj=納i]Rii・ci
でよかった。
736734:04/01/14 01:41
>>734
>よって特にf(x)は1次式であり最高次の係数は1。
ここはギャップあるかな?ちょっと丁寧にやればf(x)-x≦xなるxが無数に
存在せねばならないのでf(x)は1次で最高次の係数は1か2。
2だとしてf(x)=2x+cとすると(x+c)|xなるxが無数に存在せねばならないが
(x+c)|x⇔(x+c)|cであるがこれをみたす整数は有限個しかない。矛盾。
737:04/01/18 20:26
a_1=1
a_{n+1}={1+4a_n+√(1+24a_n)}/16

a_nの一般項を求めよ。
さぁ数学に自信のあるみんな、悩んでくれ。
残念ながらそれほど難しくない。漏れも堕ちたものだ・・・。
>>737
(8+6/2^(n-2)+1/2^(2n-4))/24
739132人目の素人さん:04/01/18 22:27
>738
ということは, b_n = √(1+24a_n)-3 とおくのが簡単でつね。
b_1=2, b_(n+1)=(b_n)/2.
aを自然数の定数、xを正の実数とするとき、次の関数を定義する。
y=x^x^・・・^x (右辺に登場するxの数はa回)
xをyとaで示せ。
どうやら>>740は、x^(x^x)≠(x^x)^xを知らないようだな。
742:04/01/18 22:46
>>738 正解。このスレレベルにしては簡単すぎた。申し訳ない。
743132人目の素数さん:04/01/18 22:55
二次関数の基本的な問題ですが、お願いします。

【問題1】
二次関数y=2x^2のグラフを平行移動したもので
点(1,3)を通り、
頂点のx座標が正で、かつ、その頂点が直線y=2x−3 上にある式を求めよ。

【問題2】
x≧−2、y≧−2、2x+y=8 のとき
xyの最大値をM、最小値をmとする。
この時最大値Mと最小値mとの差はいくつか。

【問題3】
二次不等式ax^2+bx+1≧0 の解が、−2≦x≦3 であるとき
a、bの値は次のうちどれか。

【問題4】
x^2+(K+1)x−2K=0 が、異符号の2つの解を持つとき
Kの値の範囲を求めよ。

【問題5】
放物線 y=x^2+2x+4m+4 とx軸が共有点を持たないとき
mの値の範囲を求めよ

自分で答えをだしてみたんですが、
完璧にあっているのか自信がありません。
>>743

3 132人目の素数さん 03/04/07 03:27
●未解決問題は禁止
●宿題は自分で解け(自分の解ける問題だけ)
>>743
丸痴氏ね
>>743
自分で答えを出したんなら、それを書け馬鹿
747132人目の素数さん:04/01/18 23:22
火星の軌跡と

地球の地軸との交点を

求めてみろよ

面白いから
748:04/01/20 16:30
平面状に楕円Cがある。平面上の頂点の集合Gは次の性質を満たす。
G:Gに含まれるどの要素Aも、楕円Cの外側に存在し、かつCへの2つの接線が直交する。
Gがどのような図形をなすかを求めよ。

結構難しい。とは言えごりごりの計算でも出来なくはないが…。
綺麗な解答が得られたら神。
円に決まってるだろう。
750132人目の素数さん:04/01/20 17:12
>>748
楕円Cの式を x^2/a^2+y^2/b^2=1 、Aの座標を(X,Y)とする。
Aを通る直線の方程式をtをパラメータとして
(x,y)=t(cosθ,sinθ)+(X,Y) ・・・(1)とする。これをCの式に代入する。
b^2(tcosθ+X)^2+a^2(tsinθ+Y)^2=a^2b^2
整理して
(b^2cos^2θ+a^2sin^2θ)t^2+2(b^2Xcosθ+a^2Ysinθ)t+b^2X^2+a^2Y^2-a^2b^2=0
tの2次方程式と見たとき重解を持つので
(b^2Xcosθ+a^2Ysinθ)^2-(b^2cos^2θ+a^2sin^2θ)(b^2X^2+a^2Y^2-a^2b^2)=0 ・・・(2)
一方、Aにおいて直線(1)と直行する直線は(1)の式で
θをθ±π/2に置き換えたものだから(2)と同様に
(b^2Xsinθ-a^2Ycosθ)^2-(b^2sin^2θ+a^2cos^2θ)(b^2X^2+a^2Y^2-a^2b^2)=0 ・・・(3)
が成り立つ。(2)と(3)とを加えると
b^4X^2+a^4Y^2-(a^2+b^2)(b^2X^2+a^2Y^2-a^2b^2)=0
整理すると X^2 + Y^2 = a^2 + b^2 。よって、Aの集合Gは円になる。
751:04/01/20 23:16
>>751
 割と綺麗な解答ですね!私は5変数の連立方程式をゴリゴリ解いたので・・・。
>>750
 まぁ円にはなるのですが、それを当たり前というためには何か変換を加えな
いと難しくないですか?うまい平面図形の変換ありますか?
>>751=紙
            _       |    |
 \      ─ ̄ /       |_ _ |─
            /     ─ ̄|    |           _─
 / ̄ヽ     / _       |     |   _ _ ─ ̄
/    |    ∠- ̄ ヽ      |   ヽ、|
     |          |     |
    /     /ヽ、 /      ヽ_─ー
   ノ      ヽ_/

                          _    |         | _ \\       |
       _ ─ 、       _ _─ ー ̄ /   |       ─ー| ̄           | _
  _─ ̄     ヽ            /     |          |             | ̄
            |            /      |          |  / ̄ヽ、    _ |_
           /  _─ー 、    |       |/ヽ        |      /  /   | \
          /         )    ヽ      || ヽ    /  |  ヽ_     |    |  ヽ、
       ─ ̄       -─'      \    |   \_/   |    ─ー   \_ノ
753132人目の素数さん:04/01/21 13:13
>>749=>>752
そう言わず>>751=髪に答えてヤレ
749≠752。
>>754
752さんですか?
756:04/01/21 15:08
ここもレベル落ちたな。あばよ。
757132人目の素数さん:04/01/21 16:28
廃れているので、簡単な問題でも書いておこう。

1.円周上に任意に取った3点が鋭角三角形をなす
  確率を求めてみれ。

2.半径aの円柱が3本互いに直交して交わるとき
  共通部分の体積を求めてみれ。
758132人目の素数さん:04/01/21 16:48
>>757
1. もう少し条件を絞ってくれないと、答えがいくらでも考えられる。

2. 円柱が交わるってどういう意味。円柱の中心が同一の点で垂直に交わってる事?
759お願いします:04/01/22 01:30
「sin1の近似値を求めよ」という問題です。
どうかお願いします。
760132人目の素数さん:04/01/22 07:02
>>759
1
761132人目の素数さん:04/01/22 07:02
というのは嘘
>>758
(2)はおそらく芝工の入試問題とおもわれ.
763:04/01/22 18:14
>>758 円周上に一様分布で3点取るということだろう。それなら解が出るし。
 あと>>751は俺だが>>756は俺ではない。
神様がそんな細かいことに拘るなんて
765:04/01/23 02:38
>>764
 済まん。ついつい口出してしまった。
 まぁ>>757は誰か解いてくれ。
 お詫びにこのスレに難問を出そうと思う。ただ超難問ではない。残念。

(1)代数学の基本定理というのがある。ご存知、f(x)=0はf(x)がn次多項式
ならばn個以下の複素数解を持つというやつだ(重解を考え、「以下」とした)。
これを証明してほしい。(ちなみに「代数学の基本定理」は常識なので、知ら
なかったら覚えておくように。因数定理を習ってたら当たり前に見えよう)
 ま、帰納法なのだが、f(x)=0が「一つ以上」解を持つというところを示すの
が難しい。俺は複素関数論での証明しか知らん。純代数的に解けないか?

(2)さて、(1)は知識問題だったが、これは本題。f,gが0でない多項式で、
常にf(x^2+x+1)=f(x)g(x)が成立する。このときf(x)は偶数次多項式であ
ることを示せ。
>>765

「純代数的に」の意味が判らないが、解が存在するのは、実数(複素数)
の連続性に拠っているから解析的要素が入るのはいたしかたないんじゃ
ないかな?
767:04/01/23 02:52
>>766 ありがとう。やっぱりそうなのかな…。解析的要素無しに証明
しようと思ったが失敗した。
 Liouvilleの定理から純解析的に証明することはできるのに。
>>767
その定理には沢山の証明がある.
代数学の教科書にも載ってる. とりあえずそれを嫁.
それで納得するかどうかは知らんが.
Qの代数閉体が存在し、Cであることを示せ。ってことだろう?
無理じゃね?
770132人目の素数さん:04/01/23 03:00
>>765 (2)はムズイネ
ここはあからさまに難しい問題は全面的に放置されてるな。
解ける範囲のことしかしてない。
一つの難問を全員で協力しながら解けよ
>>765 (2)
f(x) = x-i, g(x) = x+1+i のとき、f(x^2+x+1) = f(x)g(x)。
なんか勘違いしてたらスマソ
>>772
そんなことは猿でもわかる。
>>765
xが実数のときx<x^2+x+1。
>>765
n個以下なら簡単。
776132人目の素数さん:04/01/23 14:17
>>775
 確かに…。多分n個解があることを示せと書きたかったんだろう。そうす
れば確かに代数学の基本定理だ。
 しかしミスの多い神だな。
>>772
 当然実多項式なんだろう。
777132人目の素数さん:04/01/23 15:07
>>765
(2)
f(x^2+x+1)=f(x)g(x)
f(x)=f(s(x))/g(x)=fs/g
fs=fss/gs->f=fsfss/gs=(x^a2+...)(x^a4+...)/(x^b2+...)
=(x^2(3a-b)+...) とか?
779132人目の素数さん:04/01/23 15:34
>>777
f=fsfss/gsg
>>779
それも間違い。
781 ◆BhMath2chk :04/01/23 20:00
>>765
fが実係数なら実根を持たないので偶数次。
782132人目の素数さん:04/01/23 20:21
>>781 実根を持たない理由は?
>>774 >>781 こんなこと?
f(x) が奇数次なら、f(x)=0 は実根を持つ。最大の実根を α とする。
f(α^2+α+1) = f(α)g(α)。
f(α)=0 より、f(α^2+α+1)=0。つまり α^2+α+1 も f(x)=0 の実根。
α は実数なので α^2+α+1>α。
これは、α が最大の実根であることに反する。矛盾。
>>757
いちお穴を埋めとく
1.1/4
2.8(2-√2)a^3
  (y^2+z^2<a^2, z^2+x^2<a^2, x^2+y^2<a^2 の共通部分の体積)
785132人目の素数さん:04/01/23 22:03
>>758>>784を見比べると
レベルの違いがよくわかるね。
786132人目の素数さん:04/01/24 00:36
この問題を完璧に解く事が出来れば一流数学家

(95年度・某超難関大入学試験問題)

[1]

xy平面上に、関数x^2+(y-8^)2=2、x=√2(2≦y≦8)、x=-√2(2≦y≦8)、
y=8(x≦l√2l)、y=2(x≦l√2l)、y=-x^2+4(-3≦x≦-√2、√2≦x≦3)
がある。これらのグラフの概形を同一xy平面上に描き、グラフからいえることを
簡潔に述べよ。


この問題、見かけによらずマジで難しい。
おまいらも一度やってみるといいよ。






間違い多すぎ。
788132人目の素数さん:04/01/24 03:13
>>672
遅レスだが、簡単じゃんコレ。
偏微で良く出てくるheat kernelの問題だぜ。
解析系の院生なら誰でも解けると思うんだが。
ちなみにH13年東大院試専門Bの第12問と途中まで同じだ。

略解
(1)具体的に偏微分を計算するだけ
(2)x/2√t=yと変数変換して(これは定石)、
さらに極座標に持ち込み∫[-∞,∞]e^(-y)^2dy=√πを使うと出る
(3)問題文自体間違っている(!)のでお話しにならないのだが、
>>672が解いて欲しいと思った問題はおそらく
有界連続関数gのconvolutionの広義一様収束性だろう。
確かに少々面倒くさいが、「このやり方を知ってますか」という知識問題。
具体的には積分区間を極小と非有界の二つに分割してやって、
片方にgの一様連続性、もう片方に熱核の性質
(原点近く以外ではほとんど0を取る)を使ってやれば良い。
この(3)自体、PDEの本には大抵載っているが、例えば
熊ノ郷準『偏微分方程式』(共立出版)のp95にそのまま載ってる。

・・・俺には他の問題の方がよっぽど難しく見えるよ、お前らすげぇな。
789788:04/01/24 03:17
多分、皆高校数学範囲の問題を期待してると思われるので、
次のような論証問題を一つ。

問題
面積1の7個の多角形が面積4の正方形の中に描かれている。
このとき、共通部分の面積が1/7以上の2個の多角形が存在することを示せ。
問題文は1,2も間違ってるんだが。
791788:04/01/24 03:20
うん、だから解いて欲しいと思われる問題を脳内変換したw
>>672がどんな顔でカキコんだのか想像するとかなり恥ずかしいな。
今のとこ答えでてない問題

n≧2とする。n次元ユークリッド空間E^nからE^nへの写像f:E^n→E^nが
 
 d(P,Q)=1⇒d(f(P),f'Q))=1
 
をみたすときfは等長写像であることを示せ。ただしd(P,Q)はPQの距離をあらわすとする。
>>792
仮定がよくわからん
P,Q∈E^nでいいのか?
f'(Q)についてるプライムは誤植?
>>793
>P,Q∈E^nでいいのか?
いいでつ
 
>f'(Q)についてるプライムは誤植?
誤植でつ
それ、面白い問題スレで出てなかったか?
796132人目の素数さん:04/01/25 02:20
Q.789 の類題を作りますた。
(類題)
面積1のn個の閉曲線が面積Aの閉曲線の中に描かれている(n>A)。
このとき、共通部分の面積が (n-A)/{n(n-1)/2} 以上の2個の閉曲線が存在することを示せ。
797132人目の素数さん:04/01/25 17:02
>>796
一般化とはやるな〜。
ということは解けたんだろうな?
>>792
fがユニタリを言うのか。
自明っぽいけど難しそうだな。
>>796
次をしめせばよい。
Xi(1≦i≦k)がR^nの可測集合のときV=m(∪Xi)≧納i]m(Xi)-納i≠j]m(Xi∩Xj) (mは測度)
(証明)
fiをXiの定義関数とする。f(x)=納i]fi(x)とおく。
g(x)を∪[i≠j](Xi∩Xj)の定義関数とする。容易に
納i]m(Xi)
=∫f(x)dx
納i≠j]m(Xi∩Xj)
=納i≠j]∫fi(x)fj(x)dx
=∫納i≠j]fi(x)fj(x)dx
=∫[f≧2]C[f(x),2]dx
=∫C[f(x),2]g(x)dx
∴納i]m(Xi)-納i≠j]m(Xi∩Xj)=∫(f(x)-C[f(x),2]g(x))dx
ここでf(x)≧1のときに積分核は
f(x)=1⇒(f(x)-C[f(x),2]g(x))=1
f(x)≧2⇒(f(x)-C[f(x),2]g(x))≦1
いづれの場合も1より小さいので
∫(f(x)-C[2,f(x)]g(x))dx
≦∫[f≧1]dx
=m(∪Xi)
これより所与の評価をえる。□
桶?
だれか>>792挑戦してみてちょ。
>>792 ムズいね。
(1) n=2で証明できれば、nが一般の場合に拡張可能
(2) n=2の場合、dは、長さ3^(k/2)(k≧0)および長さ(√11−√3)/2を保存する
(3) n=2の場合、dが長さlを保存すれば、k≧2に対し、 d(P, Q)≦kl ⇒ d(f(P), f(Q))≦kl
等は比較的簡単に証明できる。
でも、これから先が判らない。
フィボナッチ数って一般化できるのですか?
漸化式で
a(n+2)=a(n)+a(n+1)         (n>1)
a(1)=1
a(2)=1
ってところまでしかわかりません。・
801132人目の素数さん:04/01/25 21:45
その漸化式でa(3)って求められるんだろうかw
>>801
ワラターヨ
>>800
一般化とは一般項を求めることか。そうならば、先ず、
 a(n+2)−αa(n+1)=β{a(n+1)−αa(n)} …@
とおく。@は a(n+2)=(α+β)a(n+1)−αβa(n) と同値だから、α+β=1,αβ=−1。
α,βは、二次方程式 x²−x−1=0 を解いて、α=(1±√5)/2、β=(1干√5)/2 (複号同順)。
@より、a(n+1)−(1±√5)a(n)/2 は、初項=1−α×1=(1干√5)/2=β、公比β=(1干√5)/2の等比数列で、
 a(n+1)−(1±√5)a(n)/2={(1干√5)/2}^n
±に関し辺々引き算すると、
 √5a(n)={a(n+1)−(1−√5)a(n)/2}−{a(n+1)−(1+√5)a(n)/2}
  ={(1+√5)/2}^2−{(1−√5)/2}^2
∴a(n)=[{(1+√5)/2}^n−{(1−√5)/2}^n]/√5
804132人目の素数さん:04/01/25 23:27
a(n+2)=a(n)+a(n+1)         (n>=1)
>>799
いい線いってまつ。(2)ができたならかなりいい感じでつ。ポイントは
ある1より小さい正の定数cが存在して
fがながさlを保つ→fはながさclも保つ
をしめすこと。こっからあとはほんの一工夫。
正三角形をつないだものを二回使って
aが保存ならa/3が保存を示す。
aが保存ならa/3+a/3+a/3=aから
a以下とaより大きいが区別できる事を示す。
長さを三進法で表して全ての長さを保存を示す。
>>807
勘違い。
>>807>>808
>>807=>>808なん?できたん?
>>792 (・3・)工エェー
長いので、二回に分けて書き込むYo!

補題1 n=2(二次元)の場合、dが長さLを保存すれば、長さ(√3)Lも保存する。
∵)d(P,Q)=√3Lのとき、A,Bを△ABP,△ABQが共に一辺の長さがLの正三角形になるように取れる。
仮定より、△f(A)f(B)f(P),△f(A)f(B)f(Q)も一辺の長さがLの正三角形になる。
よって、d(f(P),f(Q))=√3Lまたは0となる。
d(f(P),f(Q))=0、つまりf(P)=f(Q)と仮定する。
Cを、d(P,C)=√3L,d(Q,C)=Lとなるように取る。
仮定より、d(f(P),f(C))=d(f(Q),f(C))=Lだが、上記結果をPC間に当て嵌めると、
d(f(P),f(C))=√3Lまたは0だから矛盾。
よって、d(f(P),f(Q))=√3L ■

補題2 a:=(√11−√3)/2=0.792…とおく。n=2(二次元)の場合、dが長さLを保存すれば、長さaLも保存する。
∵)d(P,Q)=aLのとき、A,Bを△ABPが一辺の長さがLの正三角形に、
また、△ABQがd(A,Q)=d(B,Q)=√3Lの二等辺三角形になるように取れる。
仮定と補題1よりdは長さLと√3Lを保存するから、△f(A)f(B)f(P)≡△ABP、△f(A)f(B)f(Q)≡△ABQ。
よって、d(f(P),f(Q))=(√11±√3)L/2となる。
d(f(P),f(Q))=(√11+√3)L/2=L×2.524…と仮定する。
Cを、d(P,C)=d(Q,C)=Lとなるように取れる。
題意より、d(f(P),f(C))=d(f(Q),f(C))=Lだが、三角不等式より
L×2.524…=d(f(P),f(Q))<d(P,C)+d(Q,C)=2L となって矛盾。
よって、d(f(P),f(Q))=(√11−√3)L/2=aL ■
>>792 (・3・)工エェー
続きだYo!

補題3 n=2(二次元)の場合、任意の自然数M,mに対し、dは長さ(√3)^M・a^mを保存する。
∵)L=1に対し補題1をM回、補題2をm回適用する。■

補題4 n=2(二次元)の場合、∀P,Q∈E² d(P,Q)=d(f(P),f(Q))
∵)d(P,Q)=L>0とする。∀ε>0を固定する。
m:=max{k∈N|a^k≦ε/2}、M:=max{k∈N|(√3)^k・a^n≦L}とおく。
点Aを、d(P,A)=(√3)^M・a^m、d(Q,A)=a^mとなるように取れる。
三角不等式により、
 d(P,Q)≦d(P,A)+d(Q,A)≦L+ε/2<L+ε
 d(P,Q)≧d(P,A)−d(Q,A)≧(L−ε/2)−ε/2=L−ε
L−ε≦d(P,Q)<L+εで、ε>0は任意だから、d(P,Q)=L ■

命題 ∀P,Q∈E^n d(P,Q)=d(f(P),f(Q))
∵)P,Qを含む平面Lを取る。dをLに制限すると、d「LはLの距離で、距離1を保存する。
LはE²と同型だから、補題4により、d「Lは等長である。
∴ d(P,Q)=(d「L)(P,Q)=(d「L)(f(P),f(Q))=d(f(P),f(Q))■
>>811 (・3・)工エェー
申し訳ないYo。
補題4の証明に間違いがあったYo。
折を見て修正するYo。
最後の命題で平面が平面に移ることを仮定してる。
814132人目の素数さん:04/01/27 20:05
ある整数iが存在する
今、iのdケタ目の数字をランダムに変更した
これを2乗したものをxとする

この時、i^2とxのdケタ目は常に同じ数字だった
この場合、iは5の倍数である
i^2とxのdケタ目は常に同じ数字ではなかった
この場合、iは5の倍数ではない

逆に言うと
iが5の倍数であれば、dケタ目をいじってもその2乗した数字はi^2とdケタまで一致する
iが5の倍数でなければ、dケタ目をいじるとその2乗した数字はi^2とd-1ケタ目までしか一致する保障がない

この事を示せ
815132人目の素数さん:04/01/27 20:05
>>814
わかりにくかったらスマソ
題意がわからなかったら無視してね
そんなに難しくないかもしれないけど
わかりにくいんじゃなくてわからない
複数通りに読めてしま文章書くな
817132人目の素数さん:04/01/28 00:52
>複数通りに読めてしま文章書くな

複数ではなく一通りにも読めない文章は(ry
818132人目の素数さん:04/01/28 00:56
日本の財政の負債1000兆円を問題無く解消する方法を示せ。(15点)
819132人目の素数さん:04/01/28 01:08
法案X:「指名者」(∈{個人,国})は「指名者」でなくなることと引き換えに
    日本国内の任意の個人1人を「指名」することができる。
    「指名」された個人は,
   1)Aに対し1000兆円を払う義務を負う。
   2)「指名者」となる。
    また日本国内に「指名者」が居なくなった場合,国が「指名者」となる。

「法案X」は明らかに題意を満たすので,
「法案X」を執行すればよい。//         ・・・(答)
820132人目の素数さん:04/01/28 01:09
居なくなった場合→居ない場合
821ぼるじょあ:04/01/28 09:21
>>792 (・3・)
アルェー。アクセス規制で書き込めないYo
>>812の修正・追加だYo

補題2−1 n=2(二次元)の場合、dが長さLを保存すれば、任意の自然数mに対し、長さmLも保存する。
∵)d(P,Q)=mLとする。点の二系列P=A(0),A(1),…,A(m−1),A(m)=Q
およびB(1),B(2),…,B(m)を取って、1≦j≦mに対し、
△A(j−1)A(j)B(j)も△A(j−1)B(j−1)B(j)も
一辺の長さがLの正三角形にできる。
題意より、△f(A(j−1))f(A(j))f(B(j))も
△f(A(j−1))f(B(j−1))f(B(j))も一辺の長さがLの正三角形。
補題1により、1≦j≦m−1に対し、
 d(A(j−1),f(B(j+1))=d(f(B(j),f(A(j+1))=√3L
よって、fは点の二系列の位置関係を合同に移すから、
 d(f(P),f(Q))=d(f(A(0)),f(A(m)))=mL ■

補題3 n=2(二次元)の場合、任意の自然数M,mに対し、dは長さM・a^mを保存する。
∵)L=1に対し補題2をm回適用してL=a^mとし、Mに対して補題2−1を適用する。■

補題4 n=2(二次元)の場合、∀P,Q∈E² d(P,Q)=d(f(P),f(Q))
∵)d(P,Q)=L>0とする。∀ε>0を固定する。
m:=max{k∈N|a^k≦ε/2}、M:=max{k∈N|k・a^m≦L}とおく。
点Aを、d(P,A)=M・a^m、d(Q,A)=a^mとなるように取れる。
三角不等式により、
 d(P,Q)≦d(P,A)+d(Q,A)≦L+ε/2<L+ε
 d(P,Q)≧d(P,A)−d(Q,A)>(L−ε/2)−ε/2=L−ε
L−ε<d(P,Q)<L+εで、ε>0は任意だから、d(P,Q)=L ■

>>813さん指摘のとおり、命題の前掲証明はダメダメだYo。
三次元でこれを言うためには、例えば補題1を正三角形でなく、正四面体にする必要があるYo。
一般のn次元に拡張するためには、さらに複雑な拡張が必要だYo。
とても面倒くさいYo!
822関数:04/01/28 10:23
a=0.02の時、x=1450,y=166.883、x=910,y=123.721、x=630,y=71.3792、
x=93,y=6.5291、
a=0.04の時、x=1450,y=140.343、x=910,y=99.8933、x=630,y=60.9578、
x=93,y=5.7882
の時、aを未知数として線形代数を用いて一つの式として
表わすにはどうしたらいいでしょうか?
よろしくお願いします。
今、日本で一番数学の出来るひとと言ったら誰ですか?
要するに最高の数学者といったら誰でしょう?
824132人目の素数さん:04/01/28 13:08
>>823
ぼるじょあ
佐藤幹夫という人が有名だが、やはりあの人がトップと言えるかな?
>>821
>>813さん指摘のとおり、命題の前掲証明はダメダメだYo。
>三次元でこれを言うためには、例えば補題1を正三角形でなく、正四面体にする必要があるYo。
>一般のn次元に拡張するためには、さらに複雑な拡張が必要だYo。
 
単に正四面体にするだけじゃダメじゃないの?たとえば補題2−1なんかは
そのままは成立しない。n=2の場合任意に直線lとその上の点P1・・・Pmを
PiP(i+1)=Lととったときそれらを辺の一部として含む正三角形からなるタイル張りがあるけど
3次元以上だとそんなものは一般にはないからこの方針ではダメだとおもうよ。
もう一工夫必要な気がする。
ぼるじょあの中の人はダメダメだな
L/2 P L/2
←--------→↓←---------→
--------------------------- 
l B (2) C l ↑
l l  l
l l  l
l (1) (3)l  l
l l  l H
l l  l
l l  l
l A (4) D l  l
--------------------------- ↓
△                  △
  --

κ1=κ3=2 κ2=κ4=1 のとき、
上図のラーメンの曲げモーメント図(B.M.D)、せん断力図(S.F.D)を描け。
図中の(1)〜(4)は部材の番号を表しており、κは部材の剛比とする。

-----
l l
l l
-----

B C

A D

2

1 3
4


ごめん、ずれた。これで勘弁して。高さはHで荷重はL/2地点にP
ttp://no.m78.com/up/data/u000915.jpg

三度目の正直。
κ1=κ3=2 κ2=κ4=1 のとき、
このラーメンの曲げモーメント図(B.M.D)、せん断力図(S.F.D)を描け。
図中の(1)〜(4)は部材の番号を表しており、κは部材の剛比とする。

答えかきます。一部解答でてるけど全部書きます。
以下f:E^k→E^kに対して
I(f)={u>0 | ∀P,Q∈E^k d(P,Q)=u⇒d(f(P),f(Q))=u}
とおく。c=(k-1)/(2k)とおく。
補題1
P1〜Pkをd(Pi,Pj)=u (i≠j)ととるときv≧uにたいしてd(Q,Pi)=vとなる点が
ちょうど2つ存在してその間の距離は2√(v^2-cu^2)である。
(∵)計算するだけ。□
補題2
u,v∈I(f)、v>(√c)u⇒w=2√(v^2-cu^2)∈I(f)
(∵)d(Q1,Q2)=wなるQiをとる。補題1よりP1〜Pkをd(Pi,Pj)=u (i≠j)、
d(Pi,Qj)=vととれる。補題1よりd(f(Q1),f(Q2))=0 or wである。
さらにQ3をd(Q1,Q3)=w、d(Q2,Q3)=uととる。
同様にしてd(f(Q1),f(Q3))=0 or wであるがd(Q2,f(Q3))=uより
d(f(Q1),f(Q2))=d(f(Q1),f(Q3))=wである。□
系3
I(f)≠φ⇒sup(I(f))=∞
補題4
P1〜Pkをd(Pi,Pj)=u (i≠j)ととる。HをすべてのPiを通る超平面とする。
v,w∈I(f)かつv,w>(√c)uとする。
Q,Rをd(Pi,Q)=v、d(Pi,R)=wとなる点でE^k\Hのおなじ連結成分に属するとする。
Pi,Q,R,Hのfによる像をPi',Q',R',H'とするときQ',R'は
E^k\H'のおなじ連結成分に属する。
(∵)(i)w>u+vのとき。n=-[-d(Q,R)/u]とおく。つまりd(Q,R)/uの小数以下をきりあげた
自然数をnとおく。d(Q,R)≦nu<d(Q,R)+uである。
このとき仮定よりn≧2となるのでS0〜SnをS0=Q、Sn=R、d(Si,S(i+1))=uととれる。
よって特にd(Q',R')≦nuとなる。
一方でd(Pi,R'')=wなる点をE^k\H'の連結成分のうちQ'をふくまない側にとるとき
d(Q',R'')
=√(w^2-cu^2)+√(v^2-cu^2)
>√(w^2-cu^2)-√(v^2-cu^2)+2√(v^2-cu^2)
=d(Q,R)+u (仮定w>u+wよりd(Q,R)=√(w^2-cu^2)-√(v^2-cu^2)に注意!)
>nu
からR''≠R'。よってQ'とR'は同じ連結成分にはいる。
(ii)一般のとき。系3よりx∈I(f)をx>u+v、u+wととれる。そこでSを
d(Pi,S)=xとなる点でQ,RとE^k\Hのおなじ連結成分に属する点とする。
S'をSのfによる像とすれば前段の議論よりQ'とS'はE^k\Hのおなじ連結成分に属し
R'とS'はE^k\Hのおなじ連結成分に属する。よってこの場合もよい。□
系5
u,v,w∈I(f)、v,w>(√c)u⇒|√(v^2-cu^2)-√(w^2-cu^2)|∈I(f)
補題6
u∈I(f)⇒√(3/2-5/(2n))-√(1/2-1/(2n))u∈I(f)。とくにI(f)≠φ⇒infI(f)=0
(∵)補題1をv=uに適用して2(√(1-c))u=(√(2-2/n))u∈I(f)。
さらに系5をv=(√(2-2/n))u、w=uに適用して前段を得る。
√(3/2-5/(2n))-√(1/2-1/(2n))<(√11-√3)/2=0.792・・・より後段も成立。□
補題7
I(f)≠φ⇒fは縮小写像、すなわちd(f(P),f(Q))≦d(P,Q) (∀P、Q)。
(∵)P,Qを任意にとる。正数e>0をe<d(P,Q)の範囲で任意にとるとき
u∈I(f)をu<eととる。このときn=-[-d(P,Q)/u]とおけばd(P,Q)/u>1より
n≧2となるのでR0・・・RnをR0=P、Rn=Q、d(Ri,R(i+1))=uととれる。
よってd(f(P),f(Q))≦nu≦d(P,Q)+u<d(P,Q)+e。
eは任意であったのでd(f(P),f(Q))≦d(P,Q)である。□
定理8
I(f)≠φ⇒fは等長写像、すなわちd(f(P),f(Q))=d(P,Q) (∀P、Q)。
(∵)P,Qを任意にとる。Rを直線PQ上の点でQが線分PR上、かつd(P,R)∈I(f)となる点にとる。
このとき
d(f(P),f(Q))≧d(f(P),f(R))-d(f(Q),f(R))≧d(P,R)-d(Q,R)=d(P,Q)
よって補題7とあわせて主張をえる。
834132人目の素数さん:04/01/31 22:45
RSA-1024
http://www.rsasecurity.com/rsalabs/challenges/factoring/numbers.html

この数を素因数分解できたら賞金10万ドル。

13506641086599522334960321627880596993888147560566
70275244851438515265106048595338339402871505719094
41798207282164471551373680419703964191743046496589
27425623934102086438320211037295872576235850964311
05640735015081875106765946292055636855294752135008
52879416377328533906109750544334999811150056977236
890927563
面白そうやん。おそらく今見つかってる最大の素数を抜く可能性を
探っているのかな。今のとこ、わかってる範囲で最大の素数って
いくつだっけ?
>>834
禿しく素数の予感
RSAってかいてあるんだから合成数なんじゃなないの?
>>836
じゃあ素数だってことを示して10万ドルもらってこい
>>836-837
合成数であることは確認しますた

N=1350664108659952233496032162788059699388814756056670275244851438515265

2^(N-1)≡ 1209390944320336158676505953529569968675400984635889512389028083675567339322020
59338533485341471166628419681241072885123739040710771394053528488357104984091930
03137847878952260296151232848795137981274063004726939255003314975191034799510966
3412317772521248297950196643140069546889855131459759160570963857373851 (mod N)
840解けますか?:04/02/01 21:44
複素変数べき級数納∞,n=1]z^n/n
の収束、発散について議論せよ。
841132人目の素数さん:04/02/01 23:21
test
842132人目の素数さん:04/02/01 23:41
さて問題。
2004年4月1日の日経株価平均終値は?
843132人目の素数さん:04/02/02 00:18
このスレって質問ばっかり
844132人目の素数さん:04/02/02 00:20
ごめんなさい。宿題でもなんでもないのだけど、
本当にわからないです。

Nを正の整数とする時、
Nの正の約数の個数の2倍がNに等しい.

このようなNは、8と12以外に存在するかどうかを示せ。

お願いしますm(__)m
>>844
d(N)=Nの約数の個数とする。

N=p_1^e_1 p_2^e_2 ... p_k^e_k, d(N)/N=1/2とする。
d(N)/N=Π{i=1}^{k} (e_i+1)/p_i^e_i。
(e_i+1)/p_i^e_i≦(e_i+1)/2^e_i≦1より

(e_i+1)/p_i^e_i≧d(N)/N=1/2。

よって、(p_i, e_i)は(2, e)(e≦3), (3, e)(e≦1)のいずれかである。

d(2^3)/2^3=1/2,
d(2^2)/2^2=3/4,
d(2)/2=1,
d(3)/3=2/3
より、N=2^3=8またはN=2^2*3=12である。
846132人目の素数さん:04/02/02 00:45
>>845

なるほどなるほど。thx。
まあ代数系の人間にはそんなに難しくないかもしれないが出題
―問題―――――――――――――――――――――――
f(x)∈Z[x]としてその因数分解をf(x)=Π[i=1,n](x-αi)とする。
D=Π[i<j](αi-αj)^2とおく。整係数多項式g(x)、h(x)∈Z[x]で
D=g(x)f(x)+h(x)f'(x)
となるものが存在することを示せ。
――――――――――――――――――――――――――
848 ◆BhMath2chk :04/02/02 01:00
N=2mとするとm以下でNの約数でないのは一個。
m−1がNの約数ならm−1は2の約数。
m−2がNの約数ならm−2は4の約数。
>>847
どこでの因数分解なのかくらい書けよ(Q上なのかR上なのかC上なのかそれ以外なのか)
最高次の係数も無条件で1になってるし

850132人目の素数さん:04/02/02 15:34
n^sqrt(ln(ln(n))/ln(n))
=(ln(n))^sqrt(ln(n)/ln(ln(n)))
の導出が、どうしてもわかりません。

どなたかご指導ください。
よろしくお願いしますm(_ _)m
851数学〜:04/02/02 15:50
Sin(Z) = i (複素数)
を満たす複素関数Zを求めよ
sinx=(e^(ix)+e^(-ix))/2=i
e^(ix)+e^(-ix)=2i
e^(2ix)-2ie^(ix)+1=0
tan(x)=tanh(x)に同質なものを感じる。
待てよ。間違えた。
sinx=(e^(ix)+e^(-ix))/(2i)=i
e^(2ix)+2e^(ix)+1=0
(e^(ix)-1)^2=0
e^(ix)=1
x=2nπ
おお、だとしたら、どこが難解なのであろうか?
sinz=i
e^(iz)=cosz+isinz
e^(-iz)=cosz-isinz
(e^(iz)-e^(-iz))/(2i)=sinz=i
e^(iz)-e^(-iz)=-2
e^(iz)+2-e^(-iz)=0
e^(2iz)+2e^(iz)-1=0
maybe, this is correct.
>>849
αiは複素数でつ。(実数とはかぎらないという意味で。実数かもしれないし整数かも知れない。)
f(x)はモニックという仮定をいてといてくらはい。というわけで再掲
―問題―――――――――――――――――――――――
f(x)∈Z[x]をモニック多項式としてその因数分解を
f(x)=Π[i=1,n](x-αi)とする。(αi∈C)
D=Π[i<j](αi-αj)^2とおく。整係数多項式g(x)、h(x)∈Z[x]で
D=g(x)f(x)+h(x)f'(x)
となるものが存在することを示せ。
――――――――――――――――――――――――――
e^(iz)=(-2+-sqr(4-(-4)))/2=-1+-sqr(2)
iz=ln(+-sqr(2)-1)
z=-i*ln(+-sqr(2)-1)
合ってるか?
sin(z)=iになるか?
857132人目の素数さん:04/02/02 20:25
負数同士の乗算が正数になる証明。

俺、中学の時どう習ったんやろ・・・
>>720の三行目だけでも、誰かわかった人いない?
暇を見ては考えてるんだけどダメだ。

p=q=1 のときなら、最大値1、下限が-1ということはわかった。
(-1が最小値になるかどうかはまだ不明)
>>858
p,qに具体的な値入れるよりも
nに具体的な値入れて実験したほうがいいんじゃね?
やってないんで知らないけど
>>858
その問題は別スレで解決済み
861数学〜:04/02/03 11:07
>>853
>>854
>>856
複素関数の問題はこう解くんですよ

sinZ = i
e^(iZ) = cosZ+isinZ
e^(-iZ) = cosZ-isinZ
(e^(iZ)-e^(-iZ))/(2i) = sinZ = i
e^(iZ)-e^(-iZ) = -2
e^(iZ)+2-e^(-iZ) = 0
e^(2iZ)+2e^(iZ)-1 = 0
e^(iZ) = -1±√2

iZ = Log{-1±√2} = log|-1±√2| + i*arg(-1±√2)
Z = -log|-1+√2| + arg(-1+√2),
= -log|-1-√2| + arg(-1-√2)

-1+√2 > 0より arg(-1+√2) = π
-1-√2 < 0より arg(-1-√2) = -π
Z = π - i*log(-1+√2),
= -π - i*log(-1-√2),

まぁ、こんなとき方があるんだってくらい思ってくれれば
難解でしたよね・・・
862数学〜:04/02/03 11:17
あ、ごめんなさい
Z = π - i*log(√2-1),
= -π - i*log(√2+1),
でした
863数学〜:04/02/03 12:20
arg(-1+√2) = n*π
arg(-1-√2) = -n*π
Z = n*π - i*log(√2-1),
= -n*π - i*log(√2+1)
( n : 自然数 )
でつね
間違い。
出題者がアホだとどうにもならない
e^(iz)=-1±√2
ここでzは複素数なのでz=x+iy x,y:実数と置くと
e^(-y+ix)=e^(-y)*e^(ix)だから

e^(-y)=-1+√2 , e^(ix)=1 or e^(-y)=1+√2 , e^(ix)=-1
すなわち
x=0 , y=log(√2+1)
x=π , y=log(√2-1)

よって
z=i*log(√2+1) , π+i*log(√2-1)
0<x<2πとか勝手に解釈してた。逝ってきます

z=2nπ+i*log(√2+1) , (2n+1)π+i*log(√2-1) n;整数
868132人目の素数さん:04/02/03 23:16
俺が問題出したらお前ら鼻血もんだぜ?
869132人目の素数さん:04/02/03 23:17
xy平面上の単位円を底面とし、A(0,0,√3)を頂点とする円錐をx=1/2で切ったときの切り口にできる曲線上にPをとる。この時の線分APの通過する円錐の側面の面積が√3/2になることを示せ

onegaisimatu!
343 名前:大学への名無しさん 投稿日:04/02/03 21:05 ID:6TtD+7jk
84で数学できるならこれくらい出来るだろ?
xy平面上の単位円を底面とし、A(0,0,3)を頂点とする円錐をx=1/2で切ったときの切り口にできる曲線上にPをとる。この時の線分APの通過する円錐の側面の面積が√3/2になることを示せ

onegaisimasu!!

350 名前:大学への名無しさん 投稿日:04/02/03 21:46 ID:6TtD+7jk
すんません
A(0,0,3)を頂点とする円錐 でなくて
A(0,0,√3)を頂点とする円錐です

http://school2.2ch.net/test/read.cgi/kouri/1075254162/
871858:04/02/04 00:01
>>860
解決したスレに誘導おながいします。
数学〜

↑なんだこの馬鹿は?
873132人目の素数さん:04/02/04 03:04
地図に色を塗るとき、隣り合った国に同じ色を使わないようにすると
最低4色必要であることを数式によって示せ
>>873
国が1つしかない場合1色しか必要ないよ。
地図に関する条件が足りない。
f(x) = 1(-π<x<π), 0(その他)を満たす関数f(x)の
離散フーリエ変換F(α)を求めてくらさい
>>871
ちょっと前のさくらスレか分からんスレだったと思うけどどこかわからん。
方針は以下の通り (注意:計算まちがってるかも)
(1)p=1,q=1のとき最大値は1。
(∵)容易
(2)p=0,q=1-1/nのとき最大値1-3/n+2/n^2、最小値-1+3/n-2/n^2。
(∵)p=0だから値域はある実数Mによって-M≦Σ[1≦i≦n](ai)^3≦Mとかける。
一方でbi=ai+1/nとおくとΣ[1≦i≦n](bi)^3=(ai)^3+(3/n)(1-1/n)+1/n^2=(ai)^3+3/n-2/n^2
であるからΣ[1≦i≦n](bi)^3の最大値はM+3/n-2/n^2である。
ここでΣ[1≦i≦n]b_i=1、Σ[1≦i≦n](b_i)^2=1だから(1)よりM+3/n-2/n^2=1。
(3)一般のとき最大値A+B、最小値A-Bである。ここに
A=3p^2/n-3pq/n^2+p^3/n^2、B=((q-p^2/n)/(1-1/n))^(3/2)
(∵)bi=(ai-p/n)((1-1/n)/(q-p^2/n))^(3/2)とおくとΣ[1≦i≦n]b_i=0、Σ[1≦i≦n](b_i)^2=1-1/n
から(2)をつかう。以下ry
877 ◆BhMath2chk :04/02/04 22:00
>>855
fが重根を持つときはD=0なのでg(x)=0,h(x)=0とすればいい。
fが重根を持たないときfの根をa(i)とする。
p(i,x)=(Π_{j<k,j≠i,k≠i}(a(j)−a(k))^2)(Π_{j≠i}(x−a(j)))。
h(x)=Σ_{i}p(i,x)とすると
h(x)の係数はfの根の整数係数多項式で対称式なので
h(x)は整数係数多項式。
D−h(x)(df/dx)(x)をf(x)で割るとfの最高次の係数は1なので
商は整数係数多項式でD−h(x)(df/dx)(x)に根を代入すると0になるので余りは0。
g(x)=(D−h(x)(df/dx)(x))/f(x)とすると
g(x),h(x)は整数係数多項式で
g(x)f(x)+h(x)(df/dx)(x)=D。
878855:04/02/05 01:08
>>877
おお、正解。想定してた解答とちがうけど証明問題だから当然か。
判別式(共役差積)
>>876
ちょっと違うような気もするな。A=nq^2-pとおいて、
{p^3+3pA±((n-2)A√((n-1)A))/(n-1)}/n^2 かな。

俺は普通に未定乗数法で解いた。
未定乗数法で、極値はaiの構成要素が2種類未満の時ってことが分かって、
そのうち最大or最小はn-1個が同じ数で1個だけ違う場合ってのが分かる。
(n個が同じ場合はA=0の時)
881132人目の素数さん:04/02/06 13:23
>>880
オレも未定乗数法でやろうとして2種類しかないとこまでいってそこからの計算で
つまった。この
 
>そのうち最大or最小はn-1個が同じ数で1個だけ違う場合ってのが分かる。
 
これはどうやったの?
>>881
かたっぽがk種類として2種類の値を求めて、Σai^3をkで表してkについての増減
を見ればできたと思う。力技だが…。
883132人目の素数さん:04/02/06 20:07
>>882
具体的にやってみてちょ。
>>883
片方がk個なら、A=np-q^2とおいて、A>0の場合k>=1で、
Σai^3=(p^3+3pA)/n^2±{A^(3/2)/n^2}*(n-2k)/√(k(n-k))
となる。(ここまでが結構計算は大変。普通に連立方程式をといたりするだけだけど。)

で、結局kに関係のある(n-2k)/√(k(n-k))の最大、最小がどういう場合かを調べりゃいい。
|n-2k|/√(k(n-k))=(k/(n-k)+(n-k)/k-2)^(1/2)
と変形するのがわかりやすいかな。
ごめん、>>880>>884もAがまちがってる。A=nq-p^2。失敬失敬。
複素関数の問題はこう解くんですよ
まぁ、こんなとき方があるんだってくらい思ってくれれば
難解でしたよね・・・
887132人目の素数さん:04/02/06 22:41
>>885
結局この方針ではできてないの?
>>885
>ごめん、>>880>>884もAがまちがってる。A=nq-p^2。失敬失敬。
 
A=nq-p^2としてもおかしいんじゃない?
>>887
いや、できてると思うんだけど…。
>>888
どこがおかしいかな?
890888:04/02/06 23:17
>>889
いや、よみまちがってた。あってるかどうかわからん。
k個がα、n-k個がβとして
kα+(n-k)β=p、kα^2+(n-k)β^2=qを解いてkα^3+(n-k)β^3を計算すれば
いいと思うんだけどこっからなんか手がとまってしまう。どうやったの?
>>890
普通に2次方程式解いて、
α=p/n±√(k(n-k)A)/nk、β=p/n±√(k(n-k)A)/n(n-k) (複号逆順)

これからkα^3+(n-k)β^3を計算。なんも技巧的なことはしてないよ。
多分あってるはず。
>>891
すげーな・・・むしろ技巧的でないとこに感心するな。
 
>α=p/n±√(k(n-k)A)/nk、β=p/n±√(k(n-k)A)/n(n-k) (複号逆順)
 
pの分母おかしくね?
>>891
一応未定乗数λから求めようともしたが、どっちも似たような計算量になりそうだったから。

p/nでおかしくないよ。2次方程式だけでも解いてみろやw
>>893
だってkα+(n-k)βがpにならないよ。
>>894
このスレでそんなこと言うなw
ちゃんと計算汁。
このスレもずいぶんレベル下がったなあ…
>>895
ほんとになる?もっかい計算してみる。
 
>α=p/n±√(k(n-k)A)/nk、β=p/n±√(k(n-k)A)/n(n-k) (複号逆順)
 
この√(k(n-k)A)/nkと√(k(n-k)A)/n(n-k)の分母ルートの外だよね?
うん、分母の外。
√(k(n-k)A)/(nk)と√(k(n-k)A)/(n(n-k))
分母の()が足りなかったな。すまない。
899894:04/02/06 23:44
すまん・・・pになった・・・吊ってくる
√(k(n-k)A)/(nk)

√{(k(n-k)A)}/(nk)
土日おひまならどぞ
―問題―――――――――――――――――――――――――――――――
m,nを自然数とする。m行n列の複素係数の行列の組A,Bが
dim{(P,Q)∈Mm(C)×Mn(C)|PA=AQ、PB=BQ}=1
をみたすとき|m-n|=0 or 1であることを示せ。
――――――――――――――――――――――――――――――――――
902901:04/02/07 00:26
ごめん。ちょっと訂正。>>901のままでも問題としては成立してるけどより精密には次のほがいい。
―問題―――――――――――――――――――――――――――――――
m,nを自然数とする。m行n列の複素係数の行列の組A,Bが
dim{(P,Q)∈Mm(C)×Mn(C)|PA=AQ、PB=BQ}=1
をみたすとき|m-n|=1であることを示せ。
――――――――――――――――――――――――――――――――――
903 ◆BhMath2chk :04/02/07 01:10
m=1,n=1でいいような気がする。
>>903
ああ、すまん。やっぱりm=nの場合があるな。m=nになるのはm=n=1だけのハズ。
というわけで修正
―問題―――――――――――――――――――――――――――――――
m,nを自然数とする。m行n列の複素係数の行列の組A,Bが
dim{(P,Q)∈Mm(C)×Mn(C)|PA=AQ、PB=BQ}=1
をみたすとき|m-n|=1 or m=n=1であることを示せ。
――――――――――――――――――――――――――――――――――
905132人目の素数さん:04/02/07 11:23
>>720
∇G=∇f+a∇q+b∇p=0
3ai^2+a2ai+b=0
3q+2ap+bn=0
b=-(3q+2ap)/n
ai=(-a+/-(a^2-3b)^.5)/3=(-a+/-(a^2-3(3q+2ap)/n)^.5)/3
=(-a+/-(a^2-6ap/n-9q/n)^.5)/3
3p=-an+/-n(a^2-6ap/n-9q/n)^.5
(3p+an)^2=n^2(a^2-6ap/n-9q/n)=n^2a^2-6apn-9qn
9p^2+6pan+a^2n^2=n^2a^2-6apn-9qn
p^2+12pan+9qn=0
a=-(p^2+9qn)/12pn
...
どうも>>720の問題の一発解答出ないな。
みんなそれぞれに解いてるようだけど、
ちゃんと順序に沿って、綺麗な解答はないな。
難問なのかね?俺には無理だけど
907132人目の素数さん:04/02/07 13:00
>>905
訂正
9p^2+12pan+9qn=0
a=-3(p^2+qn)/4pn
これをaiにほり込んで3乗してnかければいい。




908132人目の素数さん:04/02/07 15:42
>>720
Σx=p
Σx^2=q
f=Σx^3
∇g=∇f+a∇q+b∇p=0
3x^2+2ax+b=0
3Σx^2+2aΣx+nb=0
b=-(3q+2p)/n
x=(-a+/-(a^2-3b)^.5)/3
Σx=p=n(-a+/-(a^2-3b)^.5)/3
x=p/n
Σx^2=q=np^2/n^2=p^2/n
Σx^3=np^3/n^3=pq?

909132人目の素数さん:04/02/07 16:32
>>720
Σx=p
Σx^2=q
f=Σx^3
∇g=∇f+a∇q+b∇p=0
3x^2+2ax+b=0
3Σx^2+2aΣx+nb=0
b=-(3q+2p)/n
x=(-a+/-(a^2-3b)^.5)/3
Σx=p=n(-a+/-(a^2-3b)^.5)/3
x=p/n
Σx^2=q=np^2/n^2=p^2/n
Σx^3=np^3/n^3=pq/n?



910132人目の素数さん :04/02/07 23:36
誰も答えられねぇか・・・
難関過ぎたニカ??
はっはっは

>>875
f(x) = 1(-π<x<π), 0(その他)を満たす関数f(x)の
離散フーリエ変換F(α)を求めてくらさい
911132人目の素数さん:04/02/07 23:45
凸n角形の重心が凸n角形の内部(周上を含まない)にあることを
証明してください。
>>911
先ずは、重心の定義を書いて下さい。

たとえば
 Aが重心⇔Aを通る任意の直線により図形を二分したとき、その双方の面積が等しい
が定義ならば、重心が凸角形の内部にあることは自明でしょう。
913132人目の素数さん:04/02/08 00:08
>>912
Oを原点,↑をベクトルとすると、
OX_G↑=OX_1↑+OX_2↑+・・・+OX_n↑
914132人目の素数さん:04/02/08 00:09
>>912
右辺をnで割っといてね。なははは
凸n角形の重心じゃないの。
916132人目の素数さん:04/02/08 00:23
>>915
そうやで。これって重心の公式じゃなかった?
数Bのベクトルとかで出てくる。
◆ わからない問題はここに書いてね 138 ◆
http://science2.2ch.net/test/read.cgi/math/1075025294/
恐ろしく難解ではないな。教科書の練習問題にでてきそう。
919132人目の素数さん:04/02/08 01:41
>>918
まあまあ、そう批評ばかりせずに解いてくださいな。
ちなみに教科書の練習問題に出てくるほどやさしいもんではないよ。
n角形の重心とn点の重心は違う。
>>905
ai=(-a+/-(a^2-3b)^.5)/3=(-a+/-(a^2-3(3q+2ap)/n)^.5)/3
=(-a+/-(a^2-6ap/n-9q/n)^.5)/3
3p=-an+/-n(a^2-6ap/n-9q/n)^.5

が間違い。各aiは±どちらかわからんでしょ?
各aiが同じ2次方程式の解、ってことだから、2種類から構成されるのが分かるだけ。
922132人目の素数さん:04/02/08 11:48
半径rの球面上で半径の円弧の長さがa<πrの3つの
円で挟まれた部分の面積はいくらか?
↑まず日本語から学ぼう
924132人目の素数さん :04/02/08 16:40
Cos2tをラプラス変換せよ
925132人目の素数さん:04/02/08 21:42
半径rの球面上の半径bの円で囲まれた小さいほうの
半球の表面積は?
926132人目の素数さん:04/02/08 23:09
>>922>>925か?
やヴぁいよキミ
>>922 恐ろしく難解だ!題意すらわからないよ
>>924 教科書に書いてあることは、恐ろしく難解にあらず
>>925 半球なので、b=rとなり、半球の表面積=球の表面積/2=2πr^2
    じゃないよな?922の問題だけに、何か引っ掛けられている悪寒
928132人目の素数さん:04/02/10 21:55
>>924
宿題は質問スレで聞け!
929132人目の素数さん:04/02/10 22:03
四択問題が100問

試験A
回答配分A:B:C:D=40:30:20:10
つまり全てAを選択すれば40点が取れる

試験B
回答配分A:B:C:D=25:25:25:25

ただし回答に関して何の手がかりもないとする

1:どちらの試験を受けるべきか
2:合格点が50点の時どちらの試験を受けてどのような戦略を取るか
>>929
マルチされてもなぁ
なんかつまらなくねぇ?
932 ◆BhMath2chk :04/02/11 16:00
>>904
R={(P,Q)|PA=AQ,PB=BQ}とする。
PA=AQ,PB=BQを各成分ごとに表すと
変数の個数m^2+n^2方程式の個数2mnなので
m^2+n^2−2mn≦dim(R)=1からm=nまたは|m−n|=1。
m=nとする。
Aはn次の正則な正方行列S,Tと対角行列Cを使って
A=SCTと表されPA=AQから(S^(−1)PS)C=C(TQT^(−1))。
Cの対角成分の0の個数をcとすると方程式の個数はn^2−c^2。
Bも同じで0の個数をdとすると方程式の個数はn^2−d^2。
よって2n^2−(n^2−c^2)−(n^2−d^2)≦dim(R)=1から
c^2+d^2≦1となるのでc=0またはd=0。
c=0ならAは正則行列でd=0ならBは正則行列。
Aが正則行列であるとするとQ=A^(−1)PAなので
Pが決まればQは一つに決まる。
PB=BQ=BA^(−1)PA。
P(BA^(−1))=(BA^(−1))P。
BA^(−1)が単位行列のスカラー倍でないときはEを単位行列として
P=xE+yBA^(−1)は条件を満たすので1<dim(R)。
BA^(−1)が単位行列のスカラー倍のときは
任意のPが条件を満たすのでdim(R)=n^2となりn=1。
933132人目の素数さん:04/02/12 15:58
pを素数とし、F(a)=a^(p-1)/pとする。

このとき
#{a| 1≦a≦p, F(a)≡0(mod p)}=O(p^(1/2+1/loglogp))
を示せ。
また、aを素数に限定したときはどうなるか。
934考へるヒント:04/02/14 00:33
>911
一辺ABを延長した直線を考える。
A,B以外のn-2個の頂点がABの片側に集まっていれば重心Gも同じ側にある。
935904:04/02/14 20:11
>>932
正解でつね。ちなみに次が成立しまつ。
―定理―
m行n列の行列の組A,Bにたいして
V={(P,Q)|PA=AQ、PB=BQ}
W={(P,Q)∈V|A,Bはべき零}
とおくときdimV-dimW=1⇒|m-n|=0,1でありm=nのときdimV=nとなる。
――――
初等的な証明はしらん。だれぞ挑戦してみそ。
936934:04/02/14 20:17
おながいしまつ。
【問題】
(1)平面上に5点があって、どの3点も共線でないならば、
その中に凸4角形をなすような4点が存在することを示せ。(エステ・クライン,1931)

(2)平面上に9点があって、どの3点も共線でないならば、
その中に凸5角形をなすような5点が存在することを示せ。

(3)凸6角形をなすような6点が存在する為には、最低で何点必要か。
937132人目の素数さん:04/02/14 21:09
>>933
ヒントおながいしまつ
938132人目の素数さん:04/02/14 21:23
>933
F(a) = [a^(p-1)-1]/p ?
647 名前:132人目の素数さん[sage] 投稿日:04/02/14 22:37
私からの挑戦状です。私の全能力を用いて作成された問題です。
この問題を簡単に解かれると私という存在が全否定されてしまうのではないか、という
恐怖もあります。しかし、あえて挑戦してみることにしました。

∫[0→x] √(cosx・x^2-(sinx/e^x)) dx = f(x) とするとき、
極限 lim[x→∞] f(x)/x^n  が0でない定数に収束するような自然数nの値を定めよ。
>>939
cosx・x^2-(sinx/e^x)って負になるとおもうんだが
941132人目の素人さん:04/02/15 12:16
>939
g(x)=(x^2)・(e^x)>0 は単調増加だから、
cos(x)・g(x)-sin(x)って
0.6195605246284・・・ < x < 1.4630501758863・・・
のような切れぎれの区間で正になるとおもうんだが
942凸包ジージョ:04/02/15 13:45
>936
(1) 共線でないから、外から輪ゴムをかけると3角形以上になる。4角形以上なら成立。
3角形になったときは、内部にある残りの2点を考える...
943池田 進:04/02/15 14:07
初めまして、池田 進です。
さて、面白い身近な問題を発見したので興味をもったら考えてみてください。

最近コンビニでおまけ付きお菓子が売られてヒットしています。おまけにはいろいろ種類が
あって、お気に入りフィギュアをゲットしたい。
いや、全種類ゲットしたい。しかし、パッケージの中は見えないので期待をしつつポケット
マネー出して買うわけです。

そこで、いったい何個くらい買えば何種類くらいのおまけをゲットできるか?

購買数をx、取得種類数期待値をyとして、y=f(x)、種類数をNとすると
f(0)=0
f(1)=1
f(2)=f(1)+{N-f(1)}/N
・・・・・・・・・・・
f(x)=f(x-1)+{N-f(x-1)}/N

-> f(x)=(1-1/N)f(x-1)+1

こんな漸化式になるでしょうか?

さて、これどう解く?
>>941
x→0での話をしてるのだから0の近傍で正にならないとダメだろ。
0の近傍で(cosx)(x^2)〜x^2、sinx/e^x〜xなんだから|x|<<1で
cosx・x^2-(sinx/e^x)<0。
>>933
ヒントくれくれ第2版
946132人目の素数さん:04/02/15 18:45
>>933
ヒントくれアゲ
>>945-946
1≦a, b≦pでF(a)≡F(b)≡0(mod p)ならば…
948132人目の素数さん:04/02/15 18:55
>>947
さっぱりわからん。もうちょっとおながいします。
F(a)=(a^(p-1))/p ?
F(a)=a^((p-1)/p) ?
F(a)=(a^(p-1)-1)/pでは?
951132人目の素数さん:04/02/16 00:29
>>933
ヒントくれ――――――――――――――!!!!!!!!!!!!!!!!!!!!!!!!!!!!!!!!!!!!!!!!!!!!!!!!!!!!!!!!!!!!!!!!!!!!!!!!!!!!!!!!
1/pをmodで見るのってどうするの
953132人目の素数さん:04/02/16 04:24
初めて御邪魔します。ここのスレの住人なら簡単かも知れない問題を一つ。もし、既出の場合にはご容赦。少し長く
なりますが、御許しを。

数直線上の0と1との間を一定のスピードで実数を指しながら動くポインターが有るとします。ご承知の通り、この
ポインターがどの有理数でもいいから有理数を指している時間はゼロのはずです。なぜなら、有理数は稠密 (dense)
ではなく、測度がゼロだからです。さて問題は、このことを「0と1との数直線上で有理数を発見する確率はゼロ。」
と言い換えても良いかということです。あるいは、命題を少し変えて、「0と1との間の実数を任意に抽出した場合、
それが有理数である確率はゼロで、無理数である確率は1。」と解釈しても良いのでしょうか? しかし、もし本当
にそうだとすると、有理数が0と1との間に存在している確率自体もゼロ、つまり「有理数は0と1との間には存在
しない。」となり、矛盾してきませんか? 

この疑問のそもそもの動機は、数学の面白い応用を趣味にしている私の知人が、犯罪者が人質を取った状況を言い出
したことに始まります。彼のアイデアは、犯罪者が警察に対して人質の生存に関するヒントを与える状況で、「もし、
この5次方程式のゼロに一番近い根が有理数であれば、人質は生きていて,無理数であれば、もう死亡している。」
というものでした。根を厳密に探る場合には、コンピューターもあまり助けにはならない所がみそなのだということ
です。これに対して、私の結論の一つは「有理数は稠密でないのだから、人質の生存確率はゼロ。」としたのですが、
一方で、与えられた代数方程式そのものの係数が有理数の場合が多いので,根が有理数の確率もゼロでなくなる気も
します。(尤も、この文脈で「確率」を論ずるのも少し変だという見方も有りますが。)

この人質云々の問題は動機を示すためだけで、あまり重要ではないのですが、要点は、実数閉区間の中から測度が
ゼロの有理数を抽出する確率はゼロと考えて良いか、また、そうだとすると、有理数は存在していないのと同じに
なって矛盾しないか、ということです。どなたか、機知にとんだ回答をお願いします。
954132人目の素数さん:04/02/16 06:32
有理数は稠密でないのだから
有理数は稠密でないのだから
有理数は稠密でないのだから
有理数は稠密でないのだから
有理数は稠密でないのだから
有理数は稠密でないのだから
有理数は稠密でないのだから
有理数は稠密でないのだから
>>944
>>x→0での話をしてるのだから

へ?
測度論学べとか言おうと思ったけど
どう見てもそれ以前の問題だなこりゃ
>>952
a^(p-1)-1は常にpで割り切れる。よって、(a^(p-1)-1)/pは整数。
問題文の訂正をしないままヒントなんか出してるから
訂正無しかと思ったよ。何か特別なルールでもあるのかと思った
959はは:04/02/16 21:25
問題・素数は無限個あることを証明せよ。
960132人目の素数さん:04/02/16 23:10
代数体 F上の non-CM 楕円曲線 Eと素数 pに対してLを Fに、Eの総ての pべき分点を添加した体とする。
Mordel-Weil群 E(L)の非p-torsion部分が、有限群となることを簡潔に解説せよ。
また、この有限群の位数に現れる可能性がある素数は、Eに対して定まるある有限個の素数であることを証明せよ。

どなたか自信有る方おねがいします。
(4^4−1)/5−(3^4−1)/5=(4^4−3^4)/5=175/5=35。
962132人目の素数さん:04/02/17 00:22
>>955
ああ、スマンx→∞のときか。しかしいづれにせよ積分区間が0〜xなんだから
∫[0→x] √(cosx・x^2-(sinx/e^x)) dxは定義不能。原点の近傍で積分核が定義不能だから。
963132人目の素数さん:04/02/17 00:23
>>961
なにそれ?
964132人目の素数さん:04/02/17 00:51
ab=bc=ca=1
ba=cb=ac=-1
aa=bb=cc=0
で、
abc=bca=cab=acb=bac=cba=0
のとき、
aaa=?
aab=?
abb=?
bba=?
965132人目の素数さん:04/02/17 00:58
>>961>>933の解答なの?解説きぼんage
>>960
>非p-torsion部分
ってtorsion部分のorderがpと互いに素の元のなす部分群って意味?
967223,241:04/02/17 01:18
【再掲】0≦a,b,c≦1のとき、次の不等式を示してくださいです。
F(a,b,c) ≡ a/(b+c+1) + b/(c+a+1) + c/(a+b+1) + (1-a)(1-b)(1-c) ≧ 7/8.

a+b+c=s とおき、F(a,b,c) ≧ F(s/3,s/3,s/3) ≧ 7/8 と分ける。
右側は F(m,m,m)=3m/(2m+1)+(1-m)^3=7/8+(m-1/2)^2・{1/2+m(3-2m)}/(2m+1)≧7/8.(0≦m≦1)
等号成立は m=1/2のとき.
幸運にして?左側も成り立つ。これが本問のポイント。

基本対称式を a+b+c=s, bc+ca+ab=t, abc=u とおくと、
F(a,b,c)−F(s/3,s/3,s/3)=(s+1){(7s/3+2)(s*s/3-t)-3(s^3/27-u)}/[(2s/3+1){(2s/3+1)^2-(s+1)(s*s/3-t)+(s^3/27-u)}]
-(s*s/3-t) + (s^3/27-u).
968132人目の素数さん:04/02/17 02:51
家族で晩飯を食べいてるときにテレビに小倉優子がでていた。

俺「小倉ありすって昔シノラーだったって知ってる?」

妹「優子でしょ、あれ。小倉ありすって誰?」

俺「あー優子じゃん。そうそう、小倉優子、、、。」

死ぬかと思った。
>>967
これ未定定数法でとけるんじゃないの?
970132人目の素数さん:04/02/17 02:57
家族で朝飯を食べいてるときにテレビに小倉智昭がでていた。

俺「小倉ありすってズラだったって知ってる?」

妹「智昭でしょ、あれ。小倉ありすって誰?」

俺「あー智昭じゃん。そうそう、小倉智昭、、、。」

死ぬかと思った。
>>953
>実数閉区間の中から測度がゼロの有理数を
>抽出する確率はゼロと考えて良いか

よい。

>そうだとすると、有理数は存在していないのと同じに
>なって矛盾しないか

あくまで「確率論的・測度論的には、存在しないのと同等に扱える」
だけであって、実際に存在しないと主張しているわけではないので
何も矛盾は起こらない。
972132人目の素数さん:04/02/17 09:44
>>971さん、有難う御座います。私は理論物理屋ですが、私も大体似たことを考えていました。
ところで、2chの欠点は回答者の身分や立場があまりはっきりしない点だと思われます。
と申しますのは、数学者か物理屋かによっても数学の「解釈」が大分変わってくるという話をよく耳にしますから。
但し、質問や回答が公表されるために、あまりいい加減なことは書けないのが2chの利点だと思われます。 
>>953は大体似たことを考えていた人間の書く文章とは思えんが・・・w
974132人目の素数さん:04/02/17 11:56
要するに、>>953は「実数の集合は100%有理数からできている。」という命題のどこがおかしいのか
と訊きたかったんだろう? それで、お前さんの答えは「どこもおかしくない。」でいいんだね! 
それだけのことだ! 
975132人目の素数さん:04/02/17 12:19
ああ、それで、無理数のことを英語では「不合理な」という形容詞を使って 'irrational numbers' と言うんだ! 
ところで、>>974の「実数の集合は100%有理数からできている。」は
「実数の集合は100%無理数からできている。」の間違いだと思います。 
>>953
回答もらっててマルチかよ。

恐ろしく難解な問題をだせ!
http://science2.2ch.net/test/read.cgi/math/1049652059/
953  132人目の素数さん  Date:04/02/16 04:24

分からない問題はここに書いてね154
http://science2.2ch.net/test/read.cgi/math/1076862490/
49  132人目の素数さん  Date:04/02/16 14:30
977132人目の素数さん:04/02/17 12:33
>>972
理論物理屋ってのは、おまえさんみたいな馬鹿でも勤まるんだな・・・
978132人目の素数さん:04/02/17 13:02
>>977 
威張ってるようだが、おまえさんは、理論物理屋さえ勤まらないんなら、
自分が>>972にすら及ばないことを自分で証明したな!
不快用語を頻発させるのは無教養の象徴だぞ! 
979132人目の素数さん:04/02/17 13:09
有理数は稠密 (dense)ではない
という理論物理屋がいるだろうか?
かなり似非臭いっつーか、今井並。

5次方程式関連も・・・なぁ・・
恐らく、学部生とかでそういう研究室で卒研ということになった人とかでは?
981132人目の素数さん:04/02/17 13:29
理論物理屋があまり(純粋)数学を知らないのは、全然驚きではないね! 
年配の理論物理屋で、大体どれ位の人間達が、「多様体」の定義をちゃんと言えることか? 
嘘だと思ったら、50歳以上の理論物理屋の教授に質問してごらん! 
俺は,自称微分方程式が専門という、40歳ぐらいの数学科の教授で Painleve 方程式すら知らない奴に
出会ったことがある位だ。念のため繰り返すが、彼は数学科の教授だった! 
>>981
スレ違い。
983132人目の素数さん:04/02/17 13:35
陰毛が絡まってすごい絡み目を作っています。
どんな場合でも通用するほどき方を教えてください。
984132人目の素数さん:04/02/17 13:41
女に潮を吹かせるにはどうしたらいいか?
>>981
それで?
986132人目の素数さん:04/02/17 14:21
物理屋で多様体の定義を知ってる人はほとんどいない。理論でも。
987132人目の素数さん:04/02/17 14:22
>>984 Gスポットを刺激するといいでしょう。
988132人目の素数さん:04/02/17 14:35
定義を知らずとも、結果くらい正しいの使おうよ。
角の三等分屋と変わらねぇじゃん。
>>988
そう。
角の三等分屋⊆理論物理屋
 ↓
>953∈角の三等分屋⊆理論物理屋
991132人目の素数さん:04/02/17 15:40
つーか、物理屋と数学屋では正しさに対する認識が違う。
残り8レス。後1問くらいは出してくださいよぅ
>>991
[0,1]区間内で有理数が稠密で無い
ということに関して
正しいと認識している物理屋がいるということカ?
>>992
次スレを立てないとこのまま落ちるぞ
995132人目の素数さん:04/02/17 15:46
うんこ姫
996132人目の素数さん:04/02/17 15:47
エレガントなシコリ方教えてください。
997132人目の素数さん:04/02/17 15:49
もうオナニーは面倒です。やめてもいいですか?
998132人目の素数さん:04/02/17 15:51
スピノールって何ですか? 本を読んでたら2次元複素ベクトル空間
の元って書いてあったんだけど(読み間違いかもしれませんが)、何
のことかよく分りません。
999132人目の素数さん:04/02/17 16:10
>>993
数学の定理が正しいことは、物理屋も判断できる(証明を理解する
力があるかどうかは別にして)。
ただ物理屋は研究になると間違ったことを平気で言うようになる。
1000132人目の素数さん:04/02/17 16:10
次スレ
10011001
このスレッドは1000を超えました。
もう書けないので、新しいスレッドを立ててくださいです。。。